i hÅc quÈc gia tp. hcm · 2020. 10. 18. · ngo i b i vi¸t cõa anh ph¤m tu§n huy tø n«m...

172

Upload: others

Post on 24-Jan-2021

3 views

Category:

Documents


0 download

TRANSCRIPT

Page 1: I HÅC QUÈC GIA TP. HCM · 2020. 10. 18. · Ngo i b i vi¸t cõa anh Ph¤m Tu§n Huy tø n«m 2013, c¡c b¤n s³ th§y r§t nhi·u b i vi¸t m t¡c gi£ v¨n cán l håc sinh cõa
Page 2: I HÅC QUÈC GIA TP. HCM · 2020. 10. 18. · Ngo i b i vi¸t cõa anh Ph¤m Tu§n Huy tø n«m 2013, c¡c b¤n s³ th§y r§t nhi·u b i vi¸t m t¡c gi£ v¨n cán l håc sinh cõa
Page 3: I HÅC QUÈC GIA TP. HCM · 2020. 10. 18. · Ngo i b i vi¸t cõa anh Ph¤m Tu§n Huy tø n«m 2013, c¡c b¤n s³ th§y r§t nhi·u b i vi¸t m t¡c gi£ v¨n cán l håc sinh cõa

ĐẠI HỌC QUỐC GIA TP. HCM

TRƯỜNG PHỔ THÔNG NĂNG KHIẾU

Chuyên đềToán học

Số 11

Tháng 12/2018

Page 4: I HÅC QUÈC GIA TP. HCM · 2020. 10. 18. · Ngo i b i vi¸t cõa anh Ph¤m Tu§n Huy tø n«m 2013, c¡c b¤n s³ th§y r§t nhi·u b i vi¸t m t¡c gi£ v¨n cán l håc sinh cõa
Page 5: I HÅC QUÈC GIA TP. HCM · 2020. 10. 18. · Ngo i b i vi¸t cõa anh Ph¤m Tu§n Huy tø n«m 2013, c¡c b¤n s³ th§y r§t nhi·u b i vi¸t m t¡c gi£ v¨n cán l håc sinh cõa

MỤC LỤC

Ban biên tậpLời nói đầu . . . . . . . . . . . . . . . . . . . . . . . . . . . . . . . . . . . . . . . . . . . . . . . 7

Nguyễn Tăng VũPhép vị tự quay cùng tâm . . . . . . . . . . . . . . . . . . . . . . . . . . . . . . . . . . . . . . . . 9

Phạm Tuấn HuySuy luận phản chứng trong hình học . . . . . . . . . . . . . . . . . . . . . . . . . . . . . . . . . . 21

Nguyễn NguyễnCác bài toán về hệ số nhị thức . . . . . . . . . . . . . . . . . . . . . . . . . . . . . . . . . . . . . 49

Nguyễn Tiến HoàngMột số định hướng tiếp cận lời giải đối với các bài toán Olympic . . . . . . . . . . . . . . . . . . . 67

Phan Quốc VượngVề bài toán tổ hợp trong đề thi chọn đội tuyển 2017 . . . . . . . . . . . . . . . . . . . . . . . . . . 85

Nguyễn Mạc Nam TrungVề bài toán xác định hàm số trong giải phương trình hàm . . . . . . . . . . . . . . . . . . . . . . . 91

Võ Kế Huy - Bùi Duy KhangVề phương pháp quy nạp trong giải toán hình học tổ hợp . . . . . . . . . . . . . . . . . . . . . . . 101

Trần Nguyễn Nam HưngBài toán chia kẹo Euler . . . . . . . . . . . . . . . . . . . . . . . . . . . . . . . . . . . . . . . . . 113

Ban biên tậpLời giải đề chọn đội dự tuyển 2017 . . . . . . . . . . . . . . . . . . . . . . . . . . . . . . . . . . . 125

Nguyễn Tăng Vũ - Nguyễn Ngọc Duy - Vương Trung Dũng - Lê Phúc LữLời giải để chọn đội tuyển các năm 2016 - 2017 - 2018 . . . . . . . . . . . . . . . . . . . . . . . . 131

Ban biên tậpGiới thiệu một số bài toán hay . . . . . . . . . . . . . . . . . . . . . . . . . . . . . . . . . . . . . 169

5

Page 6: I HÅC QUÈC GIA TP. HCM · 2020. 10. 18. · Ngo i b i vi¸t cõa anh Ph¤m Tu§n Huy tø n«m 2013, c¡c b¤n s³ th§y r§t nhi·u b i vi¸t m t¡c gi£ v¨n cán l håc sinh cõa
Page 7: I HÅC QUÈC GIA TP. HCM · 2020. 10. 18. · Ngo i b i vi¸t cõa anh Ph¤m Tu§n Huy tø n«m 2013, c¡c b¤n s³ th§y r§t nhi·u b i vi¸t m t¡c gi£ v¨n cán l håc sinh cõa

LỜI NÓI ĐẦU

Các bạn đang cầm trên tay Chuyên đề Toán học số 11 của trường Phổ thông Năng khiếu.

Đây là một dự án được bắt đầu từ cuối năm 2016 sau thành công của Chuyên đề Toán học số10. Quyển số 10 đã đánh dấu kỷ niệm 20 năm thành lập trường Phổ thông Năng khiếu. Nhưngquan trọng hơn, đó còn là đánh dấu của một sự chuyển giao giữa các thế hệ với những gươngmặt mới xuất hiện trong đội ngũ tác giả và biên tập viên. Và quyển số 11 này không nằm ngoàidòng chảy đó. Ngoài bài viết của anh Phạm Tuấn Huy từ năm 2013, các bạn sẽ thấy rất nhiều bàiviết mà tác giả vẫn còn là học sinh của trường. Nội dung các bài viết vẫn được trải dài ở cả 4 phânmôn Hình học - Đại số - Số học - Tổ hợp, trong đó có nhiều vấn đề thú vị được tiếp cận bởi các gócnhìn khác nhau nhưng vẫn với lối suy nghĩ tự nhiên khi giải toán của học sinh THPT. Tin rằngbạn đọc sẽ tìm thấy nhiều điều thú vị.

Dự án này không chỉ có những thành công. Bởi đội ngũ trẻ còn thiếu kinh nghiệm mà quátrình ra mắt của quyển chuyên đề đã gặp không ít khó khăn. Mặc dù một seminar giới thiệu đãđược tổ chức vào cuối tháng 7/2017 khi mọi việc gần hoàn tất, mọi thứ đã phải dừng lại vào 1tháng sau đó bởi nhiều lý do khách quan. Đến tận tháng 10/2018, những phần việc còn lại mớiđược bắt đầu và ban biên tập đã nhận được nhiều sự giúp đỡ hơn.

Để hoàn thành được quyển chuyên đề, không thể phủ nhận được sự đóng góp của nhiều bênvới những vai trò khác nhau. Xin gửi lời cảm ơn đến thầy Nguyễn Tăng Vũ đã gợi ý và hỗ trợ rấtnhiều về mặt chuyên môn khi dự án chỉ mới là ý tưởng cho đến khi gần hoàn tất. Cảm ơn thầyTrần Nam Dũng đã hỗ trợ nhiệt tình những công đoạn cuối cùng, đặc biệt là khâu in ấn, và lắngnghe rất cầu thị để các tác giả và biên tập viên có thể yên tâm sáng tạo và hoàn thiện chuyên đề.Xin gửi lời cảm ơn đến các anh Lâm Bảo Chánh, Nguyễn Văn Huyện, Lương Văn Khải, Lê Viết Ânvà Phạm Hoàng Minh đã tham gia đánh máy và hỗ trợ về kỹ thuật. Ban biên tập cũng gửi lời cảmơn đến bạn Nguyễn Lê Duy, học sinh khối chuyên Anh niên khoá 2016-2019 đã thiết kế một mẫubìa đẹp, rất đơn giản mà hiện đại cho chuyên đề số này.

Những số đếm cơ bản là từ 1 đến 10 nên số 11 thường gợi đến sự khởi đầu mới trên một nềntảng lâu dài sẵn có. Ban biên tập cũng tin như vậy. Mong rằng đây là bước đầu một sự phát triểnmới của Chuyên đề Toán học với những hoạt động liên tục hơn nhằm duy trì sự say mê tìm tòicủa học sinh khối Toán. Đó chính là cơ sở để họ còn đi xa hơn nữa.

Hãy mở sách ra đi, rồi cùng nhau khám phá xem “thầy trò Năng Khiếu viết gì".

7

Page 8: I HÅC QUÈC GIA TP. HCM · 2020. 10. 18. · Ngo i b i vi¸t cõa anh Ph¤m Tu§n Huy tø n«m 2013, c¡c b¤n s³ th§y r§t nhi·u b i vi¸t m t¡c gi£ v¨n cán l håc sinh cõa
Page 9: I HÅC QUÈC GIA TP. HCM · 2020. 10. 18. · Ngo i b i vi¸t cõa anh Ph¤m Tu§n Huy tø n«m 2013, c¡c b¤n s³ th§y r§t nhi·u b i vi¸t m t¡c gi£ v¨n cán l håc sinh cõa

PHÉP VỊ TỰ QUAY CÙNG TÂM

Nguyễn Tăng Vũ

Giáo viên trường Phổ thông Năng khiếu

1. ĐỊNH NGHĨA VÀ TÍNH CHẤT

Định nghĩa 1. Cho điểm O và số thực dương k và góc lượng giác α. Phép biến hình biếnđiểm M thành điểm M′ sao cho SM′ = k.SM,∠(OM, OM′) = α được gọi là phép vị tựquay cùng tâm (hay còn được gọi là Spiral similarity). Kí hiệu là S(O; k; α).

Định lý 1. Cho phép biến hình S(O; k; α). Khi đó:

1. Ảnh của đường thẳng d là đường thẳng d′ sao cho ∠(d, d′) = α.2. Ảnh của tam giác ABC là tam giác A′B′C′ đồng dạng cùng hướng với ABC với tỉ số

đồng dạng là k.

Định lý 2. Cho phép vị tự quay cùng tâm S(O; k; α) biến A thành A′ và B thành B′. Khi đó

1. ∆OAB ∼ ∆OA′B′.2. ∆OAA′ ∼ OBB′.

Định nghĩa 2. Ta nói hai tam giác ABC và A′B′C′ là đồng dạng cùng hướng nếu

∠(AB; AC) = ∠(A′B′; A′C′) (mod 2π) vàAB

A′B′=

ACA′C′

.

Định lý 3. Hai tam giác OAB và OA′B′ đồng dạng cùng hướng khi và chỉ khi có một phépvị tự quay biến AB 7→ A′B′.

Nhận xét.

1. Nếu có một phép vị tự quay tâm O biến AB thành A′B′ thì cũng có một phép vị tự quaytâm O biến AA′ thành BB′.

2. Nếu có ∆OAB ∼ ∆OA′B′ và ∆OAA′ ∼ ∆OBB′ thì có một phép vị tự quay tâm Obiến A 7→ A′, B 7→ B′.

9

Page 10: I HÅC QUÈC GIA TP. HCM · 2020. 10. 18. · Ngo i b i vi¸t cõa anh Ph¤m Tu§n Huy tø n«m 2013, c¡c b¤n s³ th§y r§t nhi·u b i vi¸t m t¡c gi£ v¨n cán l håc sinh cõa

A

B

O

A′

B′

Định lý 4. Cho các điểm A, A′, B, B′ sao cho không có 3 điểm nào thẳng hàng.

1. Tồn tại duy nhất một phép vị tự quay S(O; k; α) biến A thành A′ và B thành B′.2. E là giao điểm của AB và A′B′; F là giao điểm của AA′ và BB′. Khi đó O, A, A′, E và

O, B, B′, E đồng viên; O, A, B, F và O, A′, B′, F đồng viên.

A

B

O

A′

B′

E

F

Định lý 5. Cho phép vị tự quay S(O; k; α) biến A thành A′ và B thành B′. Khi đó giao điểmthứ hai của đường tròn ngoại tiếp tam giác OAB và OA′B′ là giao điểm của AA′ và BB′.

2. VÍ DỤ

Ví dụ 1 (USAMO 2006). Cho tứ giác ABCD. Các điểm E, F thuộc các cạnh AD và BC sao

choAEAD

=BFBC

. Đường thẳng EF cắt các đường thẳng AB và CD lần lượt tại S và T. Chứng

minh rằng đường tròn ngoại tiếp các tam giác SAE, SBF, TDE, TCF cùng đi qua một điểm.

10

Page 11: I HÅC QUÈC GIA TP. HCM · 2020. 10. 18. · Ngo i b i vi¸t cõa anh Ph¤m Tu§n Huy tø n«m 2013, c¡c b¤n s³ th§y r§t nhi·u b i vi¸t m t¡c gi£ v¨n cán l håc sinh cõa

A

B

CD

E

F

S

T

P

Lời giải. Xét phép vị tự quay cùng tâm S(P, k, α = (AD; BC)) biến A 7→ B, D 7→ C.

MàAEAD

=BFBC

nên S(P; k; α) : E 7→ F.

Vậy S(P; k; α) : AE 7→ BF, ED 7→ FC.Ta có S = EF∩ AB, T = EF∩CD nên theo định lý 3 thì các tứ giác PAES, EBFS, PFCTvà PEDT nội tiếp.

Ví dụ 2. Cho tam giác ABC nhọn có các đường cao AD, BE và CF. Chứng minh rằngđường thẳng euler của các tam giác AEF, BDF và CEF đồng quy.

Lời giải. Xét phép vị tự quay S(E; k; α = (EA; ED) biến A 7→ D, F 7→ C. Khi đó trựctâm tam giác AEF biến thành trực tâm tam giác ECD; tâm ngoại tiếp tam giác AEFbiến thành tâm ngoại tiếp tam giác ECD. Hay Ha 7→ Hc và Ma 7→ Mc.Gọi P là giao điểm của HaMa và HcMc, ta có P thuộc (EMaMc). Mà EMaMc là đườngtròn Euler của tam giác ABC. Nên P thuộc đường tròn Euler của tam giác ABC.

11

Page 12: I HÅC QUÈC GIA TP. HCM · 2020. 10. 18. · Ngo i b i vi¸t cõa anh Ph¤m Tu§n Huy tø n«m 2013, c¡c b¤n s³ th§y r§t nhi·u b i vi¸t m t¡c gi£ v¨n cán l håc sinh cõa

A

B

C

D

E

F

H

Ha

Ma

McHc

P

Chứng minh tương tự ta có giao điểm của các đường MbHb, McHc; MbHb, MaHa cũngthuộc đường tròn Euler của tam giác nên các giao điểm này trùng nhau. Vậy cácđường thẳng Euler của các tam giác AEF, BDF và CDE đồng quy tại một điểm thuộcđường tròn Euler của tam giác ABC.

Ví dụ 3. Cho hai đường tròn (O) và (I) cắt nhau tại hai điểm A và B. Một đường thẳngthay đổi qua A cắt (O) tại C và cắt (I) tại D (A nằm giữa C và D). Tiếp tuyến tại C của (O)

và tiếp tuyến tại D của (I) cắt nhau tại điểm E. Chứng minh rằng đường trung trực của BEluôn tiếp xúc với một đường tròn cố định.

Lời giải. Dễ thấy tứ giác BCED nội tiếp. Ta có ∆BCD ∼ ∆BOI và ∆BCO ∼ ∆BDC.Xét phép vị tự quay tâm B biến C 7→ O, I 7→ D và E 7→ K.Khi đó ta cũng có ∆BKO ∼ ∆BEC, suy ra ∠BKO = ∠BEC = ∠BDC = ∠BIO. Do đótứ giác BOKI nội tiếp.

12

Page 13: I HÅC QUÈC GIA TP. HCM · 2020. 10. 18. · Ngo i b i vi¸t cõa anh Ph¤m Tu§n Huy tø n«m 2013, c¡c b¤n s³ th§y r§t nhi·u b i vi¸t m t¡c gi£ v¨n cán l håc sinh cõa

OI

A

B

C

D

E

K

Hơn nữa KE = KO và ∠EKO = ∠DIB = 2∠EBD. Suy ra K là tâm ngoại tiếp tứ giácBCED.Gọi N là trung điểm BE ta có ∠NKB = ∠EIB, suy ra NK là tiếp tuyến của (BOI) màNK là trung trực của BE nên ta có điều cần chứng minh.

Ví dụ 4 (PTNK 2013). Tam giác ABC nhọn có H là trực tâm và P là điểm di động bên trongtam giác ABC sao cho ∠BPC = ∠BHC. Đường thẳng qua B và vuông góc với AB cắt PC tạiM, đường thẳng qua C và vuông góc với AC cắt PB tại N. Chứng minh trung điểm I của MNluôn thuộc một đường thẳng cố định.

Lời giải. Dễ thấy tứ giác BCED nội tiếp. Ta có ∆BCD ∼ ∆BOI và ∆BCO ∼ ∆BDC.Xét phép vị tự quay tâm B biến C 7→ O, I 7→ D và E 7→ K.Khi đó ta cũng có ∆BKO ∼ ∆BEC, suy ra ∠BKO = ∠BEC = ∠BDC = ∠BIO. Do đótứ giác BOKI nội tiếp.

13

Page 14: I HÅC QUÈC GIA TP. HCM · 2020. 10. 18. · Ngo i b i vi¸t cõa anh Ph¤m Tu§n Huy tø n«m 2013, c¡c b¤n s³ th§y r§t nhi·u b i vi¸t m t¡c gi£ v¨n cán l håc sinh cõa

A

B C

H

P

M

N

D1

D

I

X

Hơn nữa KE = KO và ∠EKO = ∠DIB = 2∠EBD. Suy ra K là tâm ngoại tiếp tứ giácBCED.Gọi N là trung điểm BE ta có ∠NKB = ∠EIB, suy ra NK là tiếp tuyến của (BOI) màNK là trung trực của BE nên ta có điều cần chứng minh.

Ví dụ 5 (PTNK 2013). Tam giác ABC nhọn có H là trực tâm và P là điểm di động bên trongtam giác ABC sao cho ∠BPC = ∠BHC. Đường thẳng qua B và vuông góc với AB cắt PC tạiM, đường thẳng qua C và vuông góc với AC cắt PB tại N. Chứng minh trung điểm I của MNluôn thuộc một đường thẳng cố định.

Lời giải. Ta có tứ giác PDMN nội tiếp, gọi X là giao điểm của đường tròn ngoại tiếptứ giác và đường tròn ngoại tiếp tam giác BHC, X khác phía P đối với đường thẳngBC. Ta chứng minh H, D, X thẳng hàng.

14

Page 15: I HÅC QUÈC GIA TP. HCM · 2020. 10. 18. · Ngo i b i vi¸t cõa anh Ph¤m Tu§n Huy tø n«m 2013, c¡c b¤n s³ th§y r§t nhi·u b i vi¸t m t¡c gi£ v¨n cán l håc sinh cõa

A

B C

H

P

M

N

D1

D

I

X

Ta có: ∠(XD; XP) = ∠(ND; NP) = ∠(BH; BP) = ∠(XH; XP). Suy ra X, H, P thẳnghàng. Do đó X cố định. Khi đó ∠(NX; NM) = ∠(DX; DM) = ∠(DH; DB) không đổivà ∠(XN; XM) = ∠(PM; PN) = ∠(AC; AB) không đổi. Vậy tam giác XMN đồng

dạng với tam giác XCB, suy ra ∠(XM; XI) = ∠(XC; XK) vàXMXI

=XCXK

không đổi.

Xét phép vị tự quay tâm X góc quay ∠(XC; EK) tỉ sốXCXK

biến M thành I, mà M thuộc

đường thẳng DB cố định nên I cũng thuộc một đường thẳng cố định. Kết thúc chứngminh.

Ví dụ 6 (Việt Nam TST 2013). Cho tứ giác ABCD lồi có các cặp cạnh đối không song songnội tiếp đường tròn (O; R). Gọi E là giao điểm của AC, BD. Đường phân giác góc ∠AEBcắt các đường thẳng AB, BC, CD, DA lần lượt tại M, N, P, Q.

1. Chứng minh rằng các đường tròn (AQM), (BMN), (CPN), (DPQ) cùng đi qua mộtđiểm suy nhất. Gọi điểm đó là K.

2. Đặt min{AC, BD} = m. Chứng minh rằng OK ≤ 2R2√

4R2 −m2.

Lời giải. Gọi V là giao điểm của AB và CD, U là giao điểm của AD và BC.

1. Xét phép vị tự quay biến AB thành DC. Gọi K là tâm của phép vị tự quay đó, tacó K thuộc đường tròn ngoại tiếp các tam giác BCV, ADV, ABU, CDU. (1)

15

Page 16: I HÅC QUÈC GIA TP. HCM · 2020. 10. 18. · Ngo i b i vi¸t cõa anh Ph¤m Tu§n Huy tø n«m 2013, c¡c b¤n s³ th§y r§t nhi·u b i vi¸t m t¡c gi£ v¨n cán l håc sinh cõa

Mặt khác do M thuộc AB, P thuộc AC vàMBPC

=AMDP

nên phép vị tự quay trên

cũng biến AM thành DP và MB thành PC. Từ đó ta cũng có K thuộc đườngtròn ngoại tiếp các tam giác AMQ, DPQ, MBP, CPN. (2)Từ (1) và (2) ta có các đường tròn ngoại tiếp các tam giácAQM, BMN, CNP, DPQ cùng đi qua một điểm K.

2. Ta có ∠AKU = ∠ABU = ∠ADV mà ∠ADV + ∠AKV = 1800 nên ∠AKU +

∠AKV = 180o. Do đó U, K , V thẳng hàng.Hơn nữa ∠UKA + ∠VKC = ∠ABU + ∠CBV = 2∠ADC = ∠AOC. .Do đóAOCK nội tiếp, tương tự thì DOBK nội tiếp. Mà EA.EC = EB.ED nên E thuộctrục đẳng phương của (DOBK) và (AOCK), từ đó O, E, K thẳng hàng và

OE.OK = OA2 = R2, suy ra OK =R2

OE.

Gọi H, T là hình chiếu của O trên AC và BD. Ta có OE ≥ OH, OT, mà OH =√R2 − AC2

4, OK =

√R2 − BD2

4, suy ra OE ≥

√R2 − m2

4=

12

√4R2 −m2.

Bài toán kết thúc.

3. BÀI TẬP

Bài 1. Cho hai đường tròn w1 và w2 cắt nhau tại P và K. XY là tiếp tuyến chung của haiđường tròn với X thuộcw1 và Y thuộc w2 và gần P hơn K. XP cắt w2 tại C và YP cắt w1 tạiB. BX và CY cắt nhau tại A. Đường tròn ngoại tiếp tam giác AXY và ABC cắt nhau tại Q.Chứng minh rằng ∠QXA = ∠QKP.

Bài 2. Cho hai đường tròn w và w′ cắt nhau tại A và B, đường thẳng qua B cắt w, w′ lầnlượt tại C và D. Tiếp tuyến tại C của w và tiếp tuyến tại D của w′ cắt nhau tại P. Chứngminh rằng đường trung trực của AP luôn tiếp xúc với một đường tròn cố định.

Bài 3. Cho góc ∠xAy và điểm P cố định nằm bên trong góc. Đường tròn thay đổi qua A vàP cắt Ax và Ay lần lượt tại D và E. Chứng minh rằng trọng tâm của tam giác ADE thuộcmột đường thẳng cố định.

Bài 4. Cho tam giác ABC nội tiếp đường tròn w, X là một điểm thuộc miền trong tam giác.AX, BX và CX lần lượt cắt w tại A′, B′, C′. Đường trung trực của XA′ cắt BC tại A1; cácđiểm B1, C1 được xác định tương tự. Chứng minh rằng A1, B1, C1 thẳng hàng.

Bài 5. Cho tam giác ABC. Về phía ngoài tam giác dựng các tam giác ABD, ACE cân tại Dvà E sao cho ∠ADB = 120◦, ∠AEC = 60◦. Gọi M là trung điểm của BC. Tính số đo cácgóc của tam giác DME.

Bài 6. Cho tam giác ABC. Các điểm D và E lần lượt thay đổi trên các cạnh AB, AC sao choAD = CE. Chứng minh rằng đường tròn ngoại tiếp tam giác ADE luôn đi qua một điểm cốđịnh khác A.

16

Page 17: I HÅC QUÈC GIA TP. HCM · 2020. 10. 18. · Ngo i b i vi¸t cõa anh Ph¤m Tu§n Huy tø n«m 2013, c¡c b¤n s³ th§y r§t nhi·u b i vi¸t m t¡c gi£ v¨n cán l håc sinh cõa

Bài 7. Cho tứ giác ABCD với BC = DA và BC không song song với DA. Cho hai điểmthay đổi F, E lần lượt thuộc BC và DA sao cho BF = DE. Gọi P là giao điểm của AC vàBD. EF cắt BD và AC lần lượt tại Q và R. Chứng minh rằng đường tròn ngoại tiếp tam giácPQR luôn đi qua một điểm cố định khác P.

Bài 8. Cho tam giác ABC nhọn. Tìm điểm M trong tam giác sao cho MA + MB + MC đạtgiá trị nhỏ nhất.

Bài 9. Cho hai hình vuông ABCD và AB′C′D′ cùng hướng (B 6= B′). Chứng minh rằngcác đường thẳng BB′, CC′và DD′ đồng quy.

Bài 10. Cho lục giác ABCDEF nội tiếp đường tròn sao cho AB = CD = EF. Gọi M, N, Plà trung điểm của AB, CD, EF. Chứng minh tam giác MNP đều.

Bài 11. Cho tam giác ABC và đường tròn tâm I nội tiếp tam giác tiếp xúc với các cạnhBC, AC và AB tại D, E, F. Gọi D′, E′, F′ là hình điểm đối xứng của D, E, F qua I. GọiA′, B′, C′ là trung điểm của các đoạn thẳng IA, IB, IC. Chứng minh rằng A′D′, B′E′ vàC′F′ đồng quy tại một điểm.

Bài 12 (PTNK 2011). Cho tam giác ABC nội tiếp đường tròn (O) với B, C cố định còn Athay đổi trên (O). Trung trực d của BC cắt AB, AC tại M, N. Gọi P, Q là các điểm đối xứngcủa M, N qua O và K là giao điểm của BQ và CP.

1. Chứng minh luôn thuộc một đường tròn cố định.2. Kết luận trên còn đúng không nếu thay bằng đường thẳng Euler của tam giác ABC.

Bài 13. Cho tam giác ABC cân tại A, nội tiếp đường tròn (O). Gọi (ω) là đường tròn tiếpxúc với AB, AC tại D và E và tiếp xúc trong với (O) tại K. Chứng minh rằng DE đi qua tâmnội tiếp của tam giác ABC.

Bài 14. Cho hai đường tròn (O1) và (O2) cắt nhau tại A và B. C1C2, D1D2 là tiếp tuyếnchung của (O1) và (O2). I1, I2 là giao điểm của C1D1 và C2D2 với O1O2. Chứng minh rằng∠O1AO2 = ∠I1AI2

Bài 15. Cho tam giác ABC nội tiếp đường tròn (O), gọi A1, B1, C1 lần lượt là các chân cácđường cao của tam giác dựng từ A, B, C. Gọi H1 là trực tâm tam giác A1B1C1. Gọi A2, B2, C2

lần lượt là trung điểm của B1C1, A1C1, A1B1. Chứng minh rằng trung điểm I của OH1 làtâm đường tròn nội tiếp tam giác A2B2C2.

Bài 16. Cho tam giác ABC. Đường tròn tâm I nội tiếp tam giác tiếp xúc với ba cạnh BC, ACvà AB lần lượt tại A1, B1, C1. Gọi H là trực tâm của tam giác A1B1C1, O là tâm đường trònngoại tiếp tam giác ABC. Chứng minh rằng H, I, O thẳng hàng.

17

Page 18: I HÅC QUÈC GIA TP. HCM · 2020. 10. 18. · Ngo i b i vi¸t cõa anh Ph¤m Tu§n Huy tø n«m 2013, c¡c b¤n s³ th§y r§t nhi·u b i vi¸t m t¡c gi£ v¨n cán l håc sinh cõa

Bài 17. Hai đường tròn (O1), (O2) tiếp xúc ngoài nhau tại C và tiếp xúc trong với (O) tạiD và E. Gọi (d) là tiếp tuyến chung của (O1) và (O2) tại C. AB là đường kính của (O) saocho A, D, O1 cùng phía đối với (d). Chứng minh rằng AO1, BO2 và DE đồng quy.

Bài 18 (IMO 2008). Cho tứ giác lồi ABCD (AB khác BC). Gọi đường tròn nội tiếp của cáctam giác ABC và ADC lần lượt là (ω1) và (ω2). Giả sử tồn tại đường tròn (ω) tiếp xúcvới tia BA về hướng A và tia BC về hướng C và tiếp xúc với các đường thẳng AD và CD.Chứng minh rằng tiếp tuyến chung ngoài của các đường tròn (w1) và (w2) cắt nhau tại mộtđiểm thuộc đường tròn (C).

Bài 19 (IMO shortlist 1998). Cho tam giác ABC. Gọi H là trực tâm và O là tâm đườngtròn ngoại tiếp tam giác. Gọi D, E, F lần lượt là điểm đối xứng của A qua BC, B qua CA vàcủa C qua AB. Chứng minh rằng D, E, F thẳng hàng khi và chỉ khi OH = 2R, với R là bánkính đường tròn ngoại tiếp tam giác.

Bài 20. Cho tam giác ABC nội tiếp đường tròn tâm O và ngoại tiếp đường tròn tâm I. GọiA′, B′, C′ lần lượt là giao điểm của AI, BI, CI với (O). (I) tiếp xúc với các cạnh BC, AC vàAB lần lượt tại A1, B1, C1.

1. Chứng minh rằng đường thẳng Euler của tam giác A′B′C′ đi qua điểm I.2. Chứng minh rằng đường thẳng Euler của tam giác A1B1C1 đi qua điểm O.

Bài 21. Cho tam giác ABC nhọn có trực tâm H và tâm đường tròn ngoại tiếp là O. Đườngtrung trực của AH cắt cạnh AB và AC tại D và E. Chứng minh rằng OA là phân giác củagóc ∠DOE.

Bài 22 (PTNK 2009). Cho góc xOy và điểm P cố định nằm trong góc xOy. Đường tròn thayđổi qua O và P cắt Ox và Oy tại M, N.

1. Tìm quỹ tích trọng tâm của tam giác OMN.2. Tìm quỹ tích trực tâm của tam giác OMN.

Bài 23. Cho tam giác nhọn ABC với AB khác AC. Gọi H là trực tâm của tam giác ABC,M là trung điểm cạnh BC. Lấy D trên cạnh AB và E trên cạnh AC sao cho AD = AE vàD, H, E thẳng hàng. Chứng minh rằng đường thẳng MH vuông góc với dây cung chung củađường tròn ngoại tiếp các tam giác ADE và ABC.

Bài 24 (Mối quan hệ giữa phép vị tự và nghịch đảo). Cho tam giác ABC, đường tròntâm I nội tiếp tam giác và tiếp xúc với các cạnh BC, AC và AB tại D, E, F. Chứng minh rằngtrực tâm của tam giác DEF thuộc đường thẳng nối tâm nội tiếp I và tâm ngoại tiếp O củatam giác ABC.

Bài 25. Cho hình bình hành ABCD và hai điểm A1, C1 trên các cạnh AB, BC tương ứng.Các đường thẳng CA1 và AC1 cắt nhau tại P. Giả sử đường tròn ngoại tiếp tam giác AA1Pvà CC1P giao nhau tại P trong tam giác ACD. Chứng minh rằng ∠PDA = ∠QBA.

18

Page 19: I HÅC QUÈC GIA TP. HCM · 2020. 10. 18. · Ngo i b i vi¸t cõa anh Ph¤m Tu§n Huy tø n«m 2013, c¡c b¤n s³ th§y r§t nhi·u b i vi¸t m t¡c gi£ v¨n cán l håc sinh cõa

19

Page 20: I HÅC QUÈC GIA TP. HCM · 2020. 10. 18. · Ngo i b i vi¸t cõa anh Ph¤m Tu§n Huy tø n«m 2013, c¡c b¤n s³ th§y r§t nhi·u b i vi¸t m t¡c gi£ v¨n cán l håc sinh cõa
Page 21: I HÅC QUÈC GIA TP. HCM · 2020. 10. 18. · Ngo i b i vi¸t cõa anh Ph¤m Tu§n Huy tø n«m 2013, c¡c b¤n s³ th§y r§t nhi·u b i vi¸t m t¡c gi£ v¨n cán l håc sinh cõa

SUY LUẬN PHẢN CHỨNG TRONG HÌNH HỌC

Phạm Tuấn Huy

Lớp chuyên Toán khoá 2011 - 2014

GIỚI THIỆU. Bài viết được hoàn thành sau khi tác giả đạt huy chương vàng tạiOlympic Toán Quốc tế năm 2013 và từng được sử dụng để giảng dạy cho đội dựtuyển trường PTNK. Dù được viết đã lâu, BBT vẫn cho rằng đây là bài viết giá trị vềhình học, thể hiện cách tiếp cận vấn đề tự nhiên và lý thú.

1. Cơ sở lý luận

Phản chứng là một phương pháp thú vị và một tư tưởng đẹp. Trong tổ hợp, đạisố và số học, phản chứng cho phép ta tạo thêm được điều kiện và chọn một đườnghướng chứng minh đơn giản hơn so với việc đi theo chiều xuôi từ giả thiết đến kếtluận của một bài toán. Trong hình học, ngoài những bài toán thể hiện rõ phươngpháp của phản chứng (chẳng hạn như bất đẳng thức hình học), thì tư tưởng và hìnhảnh của phép suy luận phản chứng vẫn xuất hiện trong nhiều bài toán với hình thứckhá đa đạng và thú vị. Đặc thù của phương pháp này cho phép ta "ứng biến" linhhoạt giữa giả thiết và kết luận đồng thời có được cái nhìn rộng hơn về những mốiquan hệ đó. Do đó, trong nhiều bài toán hình học, phản chứng có thể không phải làlời giải cuối cùng nhưng là đường lối hỗ trợ trong việc tìm ra lời giải.

Một trong những hình ảnh thường gặp nhất của suy luận phản chứng trong hìnhhọc là việc sử dụng "điểm trùng nhau". Tình huống thường gặp nhất, ta cần chứngminh tính chất hay sự tồn tại của một số đối tượng hình học, chẳng hạn như giaođiểm của một số đường thẳng. Khi đó, gọi hai hay một số giao điểm (dĩ nhiên tồntại) của một số cặp hay một số đối tượng. Sau đó, ta sẽ chứng minh các giao điểm(đối tượng) mà ta vừa dựng là trùng nhau. Đôi khi để thực hiện điều này, ta cũng cầngọi thêm một số đối tượng khác cùng đi qua điểm đang xét rồi xét sự đồng quy củachúng với các đối tượng gọi thêm nhằm có thêm tính chất của các điểm mà ta cầnchứng minh trùng nhau.

Suy luận này còn được gặp ở dạng sau: giả thiết cho biết điểm A là một điểmcó các tính chất (1), (2),... mà từ những điều kiện này ta suy ra duy nhất cách dựngđiểm A. Ngoài ra, từ những tính chất trên, dựa vào kết luận cần chứng minh suy luậnngược về hay qua trực quan hình vẽ, ta đoán được A còn có các tính chất (1′).(2′),...mà từ đó có thể kết nối được giữa (1), (2),... và kết luận. Khi đó, ta bỏ đi một số tínhchất theo cách xác định gốc của điểm A và thay vào đó là một số tính chất ta đoán

21

Page 22: I HÅC QUÈC GIA TP. HCM · 2020. 10. 18. · Ngo i b i vi¸t cõa anh Ph¤m Tu§n Huy tø n«m 2013, c¡c b¤n s³ th§y r§t nhi·u b i vi¸t m t¡c gi£ v¨n cán l håc sinh cõa

được, sao cho từ hệ các tính chất trên xây dựng được duy nhất một điểm A′. Bằng cáclý luận hình học ta sẽ chứng minh A và A′ trùng nhau để có cùng lúc hệ điều kiện(1), (2),... và (1′), (2′),...

Suy luận phản chứng ở đây thực chất biểu hiện ở việc ta xác định một điểm A′

theo cách khác A. Giả sử A′ không trùng A, bằng suy luận hình học thông thường sẽdẫn đến điều vô lý. Phần phản chứng này đôi khi rất quen thuộc hoặc trực quan hiểnnhiên nên ta bỏ qua.

Trong cách làm này, một tiêu chí rất quan trọng là cách dựng hình và dựng điểm:những điều kiện mà ta chọn để xác định điểm A′ phải bảo đảm tính "không thừa",tức là từ đó ta luôn dựng được ít nhất và tốt nhất là đúng một điểm thoả mãn. Đây làmột chi tiết cần được thực hiện cẩn thận vì quá trình dựng điểm mới và phản chứngdễ xảy ra ngộ nhận. Cách dựng hình cũng là một phần quan trọng giúp ta thực hiệndễ dàng và thành công những suy luận hình học, đặc biệt là những suy luận trùngnhau giữa các điểm.

Những tính chất mà ta chọn thêm có thể là những điều kiện ta cần trong kết luận,những điều ta suy ra được bằng phép tương đương từ kết luận, hay là những chi tiếtkhông gắn một cách tương đương với kết luận nhưng lại luôn có và từ đó có thể suyra được kết luận.

Việc đảo liên tục và linh động giữa giả thiết và kết luận của bài toán hay nhữngmệnh đề con cho phép ta thay đổi cách xác định điểm để tìm cách liên kết rõ và dễnhất giữa giả thiết và kết luận. Việc hoán vị các bộ điều kiện ta có, ta cần và ta suyđoán cho ta sự lựa chọn đa dạng về hướng đi và cách làm để có thể chọn bộ điều kiệnphù hợp nhất mà từ đó theo sở trường có thể suy ra tất cả những điều kiện còn lại,biến đổi chi tiết khó liên hệ thành dễ liên hệ thông qua một trung gian. Cách xác địnhmột điểm rõ ràng và cụ thể hơn chỉ ra hướng đi, hoán đổi nhiệm vụ cần chứng minh,ví dụ giữa đồng quy, thẳng hàng, đồng viên,... Đặc biệt trong nhiều bài toán sử dụngphương pháp đại số hoá (tính toán), việc hoán đổi điều kiện là rất quan trọng nhằmtìm một cách xác định và dựng điểm thuận lợi cho tính toán. Chú ý đây là phươngpháp mạnh với những công cụ tỉ lệ, tỉ số kép, các định lý về tỉ lệ như Menelaus, Cevavà phương pháp diện tích, phương pháp vector,...

Việc có thể lý luận được các điểm trùng nhau thường được dựa trên những lýluận cơ bản nhất sau đây:

Định lý 1. Về giao điểm của các đối tượng hình học:

1. Hai đường thẳng có nhiều nhất 1 giao điểm.2. Hai đường tròn có nhiều nhất 2 giao điểm.3. Một đường thẳng và một đường tròn có nhiều nhất 2 giao điểm.4. Một tia có gốc nằm trong đường tròn và đường tròn đó có nhiều nhất 1 giao điểm.

Định lý 2. Về tính duy nhất của một số đối tượng hình học:

1. Tồn tại duy nhất 1 điểm chia trong hay chia ngoài đoạn thẳng theo tỉ số k cho trước.

22

Page 23: I HÅC QUÈC GIA TP. HCM · 2020. 10. 18. · Ngo i b i vi¸t cõa anh Ph¤m Tu§n Huy tø n«m 2013, c¡c b¤n s³ th§y r§t nhi·u b i vi¸t m t¡c gi£ v¨n cán l håc sinh cõa

2. Tồn tại duy nhất 1 điểm thuộc đường thẳng AB mà MA2 −MB2 = k cho trước.3. Tồn tại duy nhất 1 điểm trên đường tròn chia 1 cung cho trước theo tỉ số k cho trước.4. Tồn tại duy nhất 1 điểm nằm trên đường thẳng đi qua 3 điểm A, B, C phân biệt sao cho

tỉ số kép của hàng định ra bởi 4 điểm đó là k 6= 1 cho trước.

Trong tình huống này, có thể mở rộng ra nhiều tính chất phức tạp hơn bằng mộtsố công cụ đại số, chẳng hạn như tồn tại nhiều nhất 1 điểm H nằm trên đường thẳngđi qua 4 điểm A, B, C, D phân biệt sao cho HA.HB = HC.HD. Một phương pháphiệu quả để xác định sự duy nhất này là lắp các điểm vào một trục toạ độ và quy vềsố nghiệm của một phương trình đại số đơn giản.

Việc chứng minh các định lý trên là khá đơn giản, xin dành cho bạn đọc. Ta sẽ đisâu hơn vào một số ví dụ cho phương pháp trên.

2. Một số ví dụ

2.1. Các bài toán liên quan đến sự đồng quy, thẳng hàng, đồng viên hay việc xác địnhgiao điểm một số đối tượng

Ví dụ 1. Cho tam giác ABC có các đường cao AA1, BB1, CC1 và trực tâm H. Chứng minhrằng đường thẳng Euler của các tam giác AB1C1, BC1A1, CA1B1 đồng quy.

Phân tích. Một hướng đi thú vị và thường gặp khi xét đến sự đồng quy của 3 hay nhiềuđường thẳng là việc xét giao điểm của từng cặp đường thẳng trong đó, chẳng hạn ở đây, ta lấymột đường thẳng làm gốc để xét các tỉ lệ trên đó nhằm chứng minh tính đồng quy, các đốitượng xác định khá quen thuộc và thuận lợi cho tính toán nên hướng làm trên khả thi.Ở bài toán này, ý tưởng trên gần như tương tự tuy nhiên hướng tiếp cận lại khá thú vị: ta xétthêm tính chất của giao điểm các cặp đường thẳng nhằm chứng minh các điểm trùng nhau,chẳng hạn xét thêm một đường thẳng hay một đường tròn chứa giao điểm ấy.

Lời giải. Gọi A2, B2, C2 là trung điểm HA, HB, HC và Ha, Hb, Hc là trực tâm các tamgiác AB1C1, BC1A1, CA1B1 thì A2, B2, C2 ∈ (A1B1C1) và A2, B2, C2 lần lượt là tâmđường tròn ngoại tiếp các tam giác AB1C1, BC1A1, CA1B1.

23

Page 24: I HÅC QUÈC GIA TP. HCM · 2020. 10. 18. · Ngo i b i vi¸t cõa anh Ph¤m Tu§n Huy tø n«m 2013, c¡c b¤n s³ th§y r§t nhi·u b i vi¸t m t¡c gi£ v¨n cán l håc sinh cõa

A

B C

C1

B1

A1

B2

A2

C2

Ha

Hb

Hc

H

Mặt khác lại chú ý rằng các tam giác AB1C1, BC1A1, CA1B1, ABC đồng dạng cùnghướng, ta có (Ha A2, HbB2) ≡ (AB1, A1B) ≡ (CA, CB) ≡ (C2A2, C2B2) (mod π)nên Ha A2, HbB2 cắt nhau tại Tc ∈ (A1B1C1). Tương tự thì Ha A2, HcC2 cắt nhau tạiTb ∈ (A1B1C1), hơn nữa Ha A2 và (A1B1C1) chỉ có duy nhất 1 giao điểm khác A2 nênTc ≡ Tb hay Ha A2, HbB2, HcC2 đồng quy. Vậy đường thẳng Euler của các tam giácAB1C1, BC1A1, CA1B1 đồng quy. Bài toán kết thúc.

Ví dụ 2 (USA TST 2013). Cho tam giác ABC nhọn có trực tâm H. HB, HA giao CA, CBlần lượt tại E, F. Gọi N, L là giao điểm của BE và đường tròn đường kính CA sao cho L nằmgiữa B và N; M, K là giao điểm của AF và đường tròn đường kính CB sao cho K nằm giữaA và M. Chứng minh rằng AB, NK, ML đồng quy.

Lời giải. Gọi T là giao điểm của ML và AB. Theo định lý Menelaus thì:

TATB

.LBLH

.MHMA

= 1

Dẫn đếnTATB

=LHLB

.MAMH

. Ta có FH.FA = FB.FC = FM2 nên dễ dàng có được:

MH = FM− FH =√

FH.FA− FH

MA = FA− FM = FA−√

FH.FA

24

Page 25: I HÅC QUÈC GIA TP. HCM · 2020. 10. 18. · Ngo i b i vi¸t cõa anh Ph¤m Tu§n Huy tø n«m 2013, c¡c b¤n s³ th§y r§t nhi·u b i vi¸t m t¡c gi£ v¨n cán l håc sinh cõa

C

A B

F

N

K

ML

T

HE

Từ đó dẫn đếnMAMH

=FA−

√FH.FA√

FH.FA− FH=

√FAFH

.

Ta cũng đồng thời có EA.EC = EH.EB = EL2 nên:

LH = EL− EH =√

EH.EB− EH

LB = EB− EL = EB−√

EH.EB

Do đóLHLB

=

√EH.EB− EH

EB−√

EH.EB=

√EHEB

, nghĩa là

TATB

=LHLB

.MAMH

=

√HE.FAHF.EB

.

Chú ý rằng A, B có vai trò như nhau đối với biểu thức ở vế phải. Tương tự, nếu gọi

T′ là giao điểm của KN, AB ta cũng cóT′AT′B

=

√HE.FAHF.EB

=TATB

. Vậy T ≡ T′ hay

ML, NK, AB đồng quy.

Ví dụ 3. (Vietnam TST 2013) Cho đường tròn (O, R) và điểm A cố định nằm trên (O).B, C là các điểm thay đổi trên (O) sao cho ∠BAC = α không đổi. Trên các tia BA, CA lầnlượt lấy E, F sao cho BE = BC = CF. Gọi P, Q lần lượt là trung điểm cung AB không chứaC và cung AC không chứa B của (O). Đường thẳng qua O vuông góc với EF cắt AB, AC tạiM, N. Chứng minh rằng giao điểm của PN và QM thuộc đường tròn cố định.

Phân tích. Các bài toán liên quan đến định lý Pascal và sự đồng quy trên đường tròn sửdụng trong nhiều suy luận khi ta có kết luận và cần chứng minh giả thiết. Ta đã biết định lýPascal với 6 điểm trên đường tròn tạo ra các giao điểm thẳng hàng. Nhưng nếu ta có các giaođiểm thẳng hàng thì sẽ có thể dẫn đến sự đồng viên của các điểm.

25

Page 26: I HÅC QUÈC GIA TP. HCM · 2020. 10. 18. · Ngo i b i vi¸t cõa anh Ph¤m Tu§n Huy tø n«m 2013, c¡c b¤n s³ th§y r§t nhi·u b i vi¸t m t¡c gi£ v¨n cán l håc sinh cõa

Cụ thể, nếu ta có AC cắt BF tại M, CE cắt FD tại N, AD cắt BE tại P, 3 điểm M, N, Pthẳng hàng và A, B, C, D, F đồng viên thì E ∈ (ABCDF). Gọi E′ là giao điểm của BP và(ABCDF), CE′ cắt FD tại N′ thì M, N′, P thẳng hàng dẫn đến N′ ≡ N, E′ ≡ E.

O

A

B C

F

Q

PE

N

M

T

I

Y

X

Lời giải: Gọi I là tâm nội tiếp tam giác ABC. Ta sẽ chứng minh rằng OI ⊥ EF hay làOE2 −OF2 = IE2 − IF2. Gọi (I) tiếp xúc với CA, AB tại X, Y thì:

IE2 − IF2 = EY2 − FX2 = (a− p + b)2 − (a− p + c)2 = (b− c)a

Lại có EA.EB = R2 −OE2 và FA.FC = R2 −OF2 nên:

OE2 −OF2 = FA.FC− EA.EB = (b− a)a− (c− a)a = (b− c)a

Do đó IE2 − IF2 = OE2 −OF2 hay OI ⊥ EF. Từ đó M, I, N thẳng hàng. Gọi PM cắtQN tại T. Ta có BQ cắt CP tại I, AB cắt PT tại M, AC cắt QT tại N và M, I, N thẳnghàng nên T ∈ (O). Vậy T luôn thuộc một đường tròn cố định.

Ví dụ 4. Cho tam giác ABC nhọn nội tiếp (O) có trực tâm H và đường cao AD, BE, CF. Pbất kỳ trên đường thẳng Euler của tam giác ABC. PA, PB, PC cắt (O) tại M, N, K. Gọi A′

đối xứng với M qua D, B′ đối xứng với N qua E, C′ đối xứng với K qua F.

26

Page 27: I HÅC QUÈC GIA TP. HCM · 2020. 10. 18. · Ngo i b i vi¸t cõa anh Ph¤m Tu§n Huy tø n«m 2013, c¡c b¤n s³ th§y r§t nhi·u b i vi¸t m t¡c gi£ v¨n cán l håc sinh cõa

a) Chứng minh rằng H, A′, B′, C′ thuộc cùng một đường tròn.b) Đường thẳng qua C′ vuông góc HC′ và đường thẳng qua B′ vuông góc HB′ lần lượt

cắt HK, HN tại V, T. Chứng minh rằng VT ‖ NK.

O

A

B C

C1

A1

H

B1

P

M

D

N

E

F

K

A′

C′

B′

A′1

B′1

C′1

X

O′1

O′3

Lời giải. Gọi A1, B1, C1 là giao điểm thứ hai của HA, HB, HC với (O).

• Ta chứng minh rằng đường thẳng qua M vuông góc với MA1, gọi là m, và cácđường thẳng xác định tương tự đồng quy trên đường thẳng OH. Thật vậy, gọiA′1, B′1, C′1 là các điểm xuyên tâm đối của A1, B1, C1 với (O). Theo định lý Pascal,gọi NB′1 cắt KC′1 tại O′1 và BC′1 cắt CB′1 tại X thì P, X, O′1 thẳng hàng. Cũng theođịnh lý Pascal thì X, O, H thẳng hàng nên X, O, O′1, H, P thẳng hàng hay n, k cắtnhau tại O′1 ∈ OH. Tương tự thì m, n cắt nhau tại O′2 ∈ OH nên O′1 ≡ O′2. Do đóm, n, k đồng quy trên đường thẳng Euler của tam giác ABC.

• Bây giờ, ta chứng minh rằng đường thẳng qua A1 vuông góc với MA1 và cácđường thẳng xác định tương tự đồng quy trên đường thẳng OH. Thật vây, quaphép đối xứng tâm O thì 3 đường thẳng trên biến thành m, n, k đồng quy tạiO′1 ∈ OH nên 3 đường thẳng ban đầu đồng quy tại O′3 đối xứng với O′1 qua O.

• Xét phép vị tự tâm H tỉ số12

, ta có đường thẳng qua D, E, F lần lượt vuông góc

với MA1, NB1, PC1, gọi là d, e, f , đồng quy tại O′′1 trên OH. Dễ thấy rằng đường

27

Page 28: I HÅC QUÈC GIA TP. HCM · 2020. 10. 18. · Ngo i b i vi¸t cõa anh Ph¤m Tu§n Huy tø n«m 2013, c¡c b¤n s³ th§y r§t nhi·u b i vi¸t m t¡c gi£ v¨n cán l håc sinh cõa

thẳng qua A′ vuông góc với HA′ và các đường thẳng xác định tương tự đồngquy tại O1 đối xứng với O′1 qua O′′1 nên ∠HA′O1 = ∠HB′O1 = ∠HC′O1 = 90◦.

Qua phép vị tự tâm H tỉ số12

thì O1 biến thành O2 là tâm của (HA′B′C′).

• Gọi f cắt HK tại F′ và k cắt HN tại E′ thì F′, E′ là trung điểm HV, HT. Do đó đểchứng minh rằng VT//KN thì ta chỉ cần E′F′//KN. Nhưng thật vậy, điều này

đúng theo định lý Thales đảo do tỉ lệE′HE′N

=F′HF′K

=HO′′1O′′1 O′1

.

Bài toán được chứng minh hoàn toàn.

Nhận xét. Để liên hệ được các góc trong tam giác A′B′C′ là một việc khá phức tạp, cùng vớiviệc phát hiện tâm của đường tròn (HA′B′C′) nằm trên OH nên ta nghĩ đến một cách chứngminh nội tiếp khá đặc biệt: chứng minh tâm hoặc chân đường kính của các đường tròn chứamột số trong các điểm H, A′, B′, C′ trùng nhau. Từ đó ta quy về việc sử dụng định lý Pascalđể chứng minh các đường thẳng đồng quy. Việc xét thêm đường thẳng OH trong sự đồngquy là một ý tương tự với ví dụ 1 nhằm giới hạn số giao điểm.

Ví dụ 5 (IMO 2013). Đường tròn bàng tiếp góc A của tam giác ABC tiếp xúc với đoạn BCtại A1. Các điểm B1 trên CA và điểm C1 trên AB được xác định tương tự, bằng cách lầnlượt xét đường tròn bàng tiếp góc B và góc C, tương ứng. Giả sử tâm đường tròn ngoại tiếptam giác A1B1C1 nằm trên đường tròn ngoại tiếp tam giác ABC. Chứng minh rằng tam giácABC là tam giác vuông.

Phân tích. Thông thường, tâm đường tròn ngoại tiếp của tam giác sẽ được xác định bởigiao điểm đường trung trực của các cạnh. Tuy nhiên ở đây, việc xác định trung trực củaA1B1, B1C1, C1A1 là khá phức tạp. Dù vậy, khi giải quyết chiều thuận của bài toán, ta thấytâm O1 của (A1B1C1) là điểm chính giữa cung chứa góc vuông. Từ đó, với trực quan, tacó thể suy đoán tâm của (A1B1C1), nếu thuộc (ABC) thì là điểm chính giữa của 1 trong 3

cung_

BAC,_

ACB,_

CBA, giả sử là_

BAC. Sau khi lấy M là điểm chính giữa cung này, ta có thểchứng minh đơn giản rằng MB1 = MC1, từ đó M thuộc trung trực B1C1. Từ điều này, tachỉ cần lí luận sao cho M và O1 thuộc cùng một cung chắn BC thì sẽ có ngay M ≡ O1.

28

Page 29: I HÅC QUÈC GIA TP. HCM · 2020. 10. 18. · Ngo i b i vi¸t cõa anh Ph¤m Tu§n Huy tø n«m 2013, c¡c b¤n s³ th§y r§t nhi·u b i vi¸t m t¡c gi£ v¨n cán l håc sinh cõa

O

O1

B C

A

B1

C1

A1

M

Lời giải. Gọi O là tâm ngoại tiếp tam giác ABC và M là trung điểm cung BC chứaA của (O). Từ giả thiết ta có BC1 = B1C, MB = MC và ∠MCB1 = ∠MBC1 nên∆MBC1 = ∆MCB1 (c.g.c). Từ đây MB1 = MC1 hay M thuộc trung trực B1C1. Do đóM là một trong hai giao điểm của trung trực B1C1 và (O).Theo phân tích trên, ta sẽ lí luận để M ≡ O1. Thật vậy, do A1, B1, C1 nằm trong cáccạnh của tam giác ABC, còn theo giả thiết thì O1 ∈ (ABC) nằm ở miền ngoài tamgiác ABC nên O1 nằm ở miền ngoài tam giác A1B1C1 hay tam giác A1B1C1 tù. Có thểgiả sử rằng ∠B1A1C1 > 90◦ để ta có O1 thuộc miền trong góc B1A1C1. Điều này dẫn

đến O1 thuộc cung_

BAC. Hệ quả là ta có M ≡ O1. Đến đây, ta có thể xử lí phần cònlại theo nhiều cách khác nhau, chẳng hạn so sánh O1A1 và O1B1 như sau:Đặt BC = a, CA = b, AB = c và 2p = a + b + c. Gọi A3 là trung điểm BC thì:

O1A21 = O1A2

3 + A1A23 = O1B2 − a2

4+

(b− c)2

4.

Nhận thấy rằng ∆MB1C1 = ∆MCB (c.g.c) nên ta cũng đồng thời có được:

O1B21 = O1B2 · B1C2

1BC2 =

O1B2

a2 ·[(p− b)2 + (p− c)2 − 2(p− b).(p− c) cos A

]

Từ các đẳng thức trên, nhờ O là tâm ngoại tiếp tam giác ABC, ta có được:

O1B2 =a2

4 sin2 A2

=a2

2(1− cos A)

29

Page 30: I HÅC QUÈC GIA TP. HCM · 2020. 10. 18. · Ngo i b i vi¸t cõa anh Ph¤m Tu§n Huy tø n«m 2013, c¡c b¤n s³ th§y r§t nhi·u b i vi¸t m t¡c gi£ v¨n cán l håc sinh cõa

Kết hợp với cos A =b+c2 − a2

2bcvà O1A1 = O1B1, bằng biến đổi tương đương, ta nhận

được a2 = b2 + c2. Theo định lý Pythagore, tam giác ABC vuông tại A.

2.2. Các bài toán liên quan đến tính duy nhất của đối tượng: dựng đối tượng thoảmãn trước, sau đó chứng minh tính duy nhất của đối tượng này

Ví dụ 6. (Sharygin 2013) Đường tròn k đi qua hai đỉnh B, C của tam giác nhọn (không cân)ABC. k cắt các tia AB, AC kéo dài về phía B, C theo thứ tự tại P, Q. Gọi A1 là chân đườngcao kẻ từ A đến BC. Giả sử A1P = A1Q. Chứng minh rằng ∠PA1Q = 2∠BAC.

Phân tích. Rõ ràng việc khai thác "giả sử A1P = A1Q" là phức tạp vì ta chưa thể xác địnhrõ P, Q từ đó. Ta sẽ đi ngược lại từ kết luận để xác định P, Q thỏa mãn. Sau đó chứng minhtrường hợp của đề bài cũng phải trùng với trường hợp đó.

A

B C

A1

P1

Q1

k

Lời giải. Dựng (A1; A1A) cắt các tia AB, AC theo thứ tự tại P1, Q1. Ta có ∠AQ1P1 =

90◦ −∠A1AB = ∠ABC nên B, C, P1, Q1 đồng viên, hơn nữa có A1P1 = A1Q1.Do B, C, P1, Q1 đồng viên, ta có PQ ‖ P1Q1 hoặc PQ ≡ P1Q1. Giả sử PQ 6≡ P1Q1.

A1P, A1Q cắt P1Q1 thứ tự tại P2, Q2. Do PQ ‖ P1Q1, A1P = A1Q nên tam giácA1P2Q2 cân tại A1, dẫn đến A1P1 = A1Q1. Từ đây thì ∠P1A1P = ∠Q1A1Q. Do đó∠A1Q1Q = ∠A1P1P, vô lý bởi vì tam giác ABC không cân.

Vậy PQ ≡ P1Q1. Do đó ∠PA1Q = ∠P1A1Q1 = 2∠BAC.

Ví dụ 7. (Vietnam TST 2008) Cho tam giác ABC nội tiếp đường tròn (O) có phân giácAD, BE, CF. Gọi K, M, N lần lượt chia trong AD, BE, CF theo cùng một tỉ số k. Kí hiệu (X)

là đường tròn qua K, A và tiếp xúc với OA. Các đường tròn (Y), (Z) định nghĩa tương tự.

30

Page 31: I HÅC QUÈC GIA TP. HCM · 2020. 10. 18. · Ngo i b i vi¸t cõa anh Ph¤m Tu§n Huy tø n«m 2013, c¡c b¤n s³ th§y r§t nhi·u b i vi¸t m t¡c gi£ v¨n cán l håc sinh cõa

a) Nếu k = 12 , chứng minh rằng (X), (Y), (Z) có 2 điểm chung và trọng tâm G của tam

giác ABC thuộc đường nối hai điểm này.b) Tìm mọi k sao cho (X), (Y), (Z) có 2 điểm chung.

O

A

B

C

F

D

EK

NM

Oa

Ha

HHc

Hb

Oc

Ob

A′

Lời giải. a) Gọi K′ là trung điểm của AK, A′ là giao điểm của tiếp tuyến tại A với BCvà Oa là tâm của đường tròn qua A, K và tiếp xúc với OA. Các điểm khác xác địnhtương tự. Ta có Oa thuộc trung trực của AK vì Oa A = OaK, và cũng có Oa ∈ A′A.

Do ∠A′DA = ∠BCA + ∠BAC2 = ∠A′AD nên A′D = A′A. Lại có Oa A = OaK

nên OaK ‖ BC. Do đó Oa là trung điểm của A′A. Tương tự thì (Oa), (Ob), (Oc) theothứ tự là các đường tròn đường kính AA′, BB′, CC′.

Gọi Ha, Hb, Hc theo thứ tự là chân các đường cao hạ từ các đỉnh A, B, C xuốngcạnh đối diện của tam giác ABC thì Ha, Hb, Hc theo thứ tự thuộc (Oa), (Ob), (Oc). MàHHa.HA = HHb.HB = HHc.HC và OA2 = OB2 = OC2 nên (Oa), (Ob), (Oc) có trụcđẳng phương chung là OH, nhưng OH chính là đường thẳng Euler của tam giác ABCnên đi qua trọng tâm G. Từ đây thì OaObOc ⊥ OG.

b) Tiếp theo ý tưởng của phần trên, gọi O′a, O′b, O′c lần lượt là các điểm chia trongAA′, BB′, CC′ với cùng một tỉ lệ k 6= 1

2 . Suy ra O′a, O′b, O′c chia trong (hay ngoài)

31

Page 32: I HÅC QUÈC GIA TP. HCM · 2020. 10. 18. · Ngo i b i vi¸t cõa anh Ph¤m Tu§n Huy tø n«m 2013, c¡c b¤n s³ th§y r§t nhi·u b i vi¸t m t¡c gi£ v¨n cán l håc sinh cõa

Oa A′, ObB′, OcC′ theo cùng một tỉ lệ k′ (k′ 6= 0). Một kết quả quen thuộc là A′, B′, C′

thẳng hàng. Để thuận tiện, ta sẽ ký hiệu là A′, B′, C′. Ta có:

−−→O′aO′b =

(k′−−→OaOb +

−−→A′B′

)· 1

k′ + 1

Và cũng đồng thời có được:

−−→O′bO′c =

(k′−−→ObOc +

−−→B′C′

)· 1

k′ + 1.

Vì O′a, O′b, O′c nên ∃µ, k′ để

−−→OaOb +

−−→A′B′ = µ

(k′−−→ObOc +

−−→B′C′

)

Hay là−−→OaOb = µk′

−−→ObOc và

−−→A′B′ = µ

−−→B′C′. Từ đây dẫn đến OaOb

ObOc= k′ A′B′

B′C′. Tuy nhiên,

nếu điều này đúng thì với k 6= 12 , ta có A, B, C thẳng hàng, vô lý.

Vậy k = 12 là giá trị duy nhất để cho (Oa), (Ob), (Oc) có hai điểm chung.

Nhận xét. Ý tưởng của câu b) là việc chứng minh k = 12 là giá trị duy nhất thỏa mãn điều

kiện, để có điều này, ta thông qua giả sử một trường hợp thỏa mãn khác tồn tại và chứng minhvô lý (hình ảnh của bổ đề ERIQ gợi nhớ đến phương pháp vector). Việc chứng minh sự tồn tạiduy nhất bằng đại số cũng là phương pháp thú vị trong hình học.

Ví dụ 8. (Iran TST 2013) Trên đường thẳng ` cho A, B, C, D phân biệt theo thứ tự đó. Cácđường tròn (O1), (O2) qua A, B và (O3), (O4) qua C, D sao cho (O1) tiếp xúc với (O3) và(O2) tiếp xúc với (O4) (các tiếp điểm ở cùng phía so với `). Chứng minh rằng tiếp tuyếnchung ngoài của (O1) và (O4), tiếp tuyến chung ngoài của (O2) và (O3) và ` đồng quy.

32

Page 33: I HÅC QUÈC GIA TP. HCM · 2020. 10. 18. · Ngo i b i vi¸t cõa anh Ph¤m Tu§n Huy tø n«m 2013, c¡c b¤n s³ th§y r§t nhi·u b i vi¸t m t¡c gi£ v¨n cán l håc sinh cõa

A DB C

O1

O3

k1

k3

MO2

k2

O4

k4

N

P

S TQ′

Q

Lời giải. Gọi N là tiếp điểm của (O2) và (O4), M là giao điểm của (O1) và (O3). GọiP′ là giao điểm của tiếp tuyến tại N của (O2), (O4) và `.

Xét trục ` với A(a), B(b), C(c), D(d), P′(p′). Ta có P′A.P′B = P′C.P′D nên khiquy về toạ độ, ta được (a− p′)(b− p′) = (c− p′)(d− p′). Sau khi khai triển, đây làphương trình bậc nhất theo p′ nên có nghiệm duy nhất, dẫn đến tồn tại duy nhấtP′ ∈ ` mà P′A.P′B = P′C.P′D. Tuy nhiên, nếu gọi P là giao điểm của tiếp tuyến tạiM của (O1), (O3) và ` thì ta cũng có PA.PB = PC.PD. Do đó P′ ≡ P.

Gọi S, T là trung điểm AB, CD. Đặt khoảng cách từ O1, O2, O3, O4 đến ` lần lượt làh1, h2, h3, h4. Trước hết, ta chứng minh đoạn mà tiếp tuyến chung chắn trên (O1), (O4)

và (O2), (O3) là bằng nhau. Điều này tương đương với:

O1O24 − (R4 − R1)

2 = O2O23 − (R2 − R3)

2

Tuy nhiên dễ dàng thấy rằng:

O1O24 − (h1 − h4)

2 = O2O23 − (h2 − h3)

2 = (d(O1O2, O3O4))2

Từ đó ta cần (h1 − h4)2 − (h2 − h3)

2 = (R1 − R4)2 − (R2 − R3)

2. Khai triển, ta cần có:

R1R4 − R2R3 = h1h4 − h2h3 (1)

Dễ dàng chứng minh được rằng:

• x2 + h2i = R2

i + h2 (1 ≤ i ≤ 4) với x = PA+PB2 , y = PC+PD

2 và h = PM = PN.• h1h3 = h2h4 = xy, R1R3 = R2R4 = h2.

33

Page 34: I HÅC QUÈC GIA TP. HCM · 2020. 10. 18. · Ngo i b i vi¸t cõa anh Ph¤m Tu§n Huy tø n«m 2013, c¡c b¤n s³ th§y r§t nhi·u b i vi¸t m t¡c gi£ v¨n cán l håc sinh cõa

Bình phương hai vế của (1), ta cần chứng minh rằng:

(x2 + h21− h2)(y2 + h2

4− h2)+ (x2 + h22− h2)(y2 + h2

3− h2)− 2h4 = h21h2

4 + h22h2

3− 2x2y2

Chuyển vế và thu gọn, ta được:

4x2y2 = 2h2(x2 + y2) + h2(h21 + h2

2 + h23 + h2

4)− x2(h24 + h2

3)− y2(h21 + h2

2) (2)

Ta có h4 = R21R2

3 = (h21 + x2 − h2)(h2

3 + y2 − h2). Khai triển và thu gọn, ta được

h21h2 + h2

3h2 + h2(x2 + y2)− y2h21 − x2h2

3 = 2x2y2.

Lập các đẳng thức tương tự và cộng lại, ta có (2) đúng, dẫn đến (1) đúng. Như vậy, độdài đoạn tiếp tuyến chung chắn trên (O1), (O4) và (O2), (O3) bằng nhau, gọi độ dàiđó là k. Gọi Q1 là giao điểm tiếp tuyến chung của (O1), (O4) và ` thì

√Q1A.Q1B−

√Q1C.Q1D = k.

Gọi Q′1 là giao điểm tiếp tuyến chung của (O2), (O3) và ` thì

√Q′1A.Q′1B−

√Q′1C.Q′1D = k.

Trên trục `, gọi toạ độ Q1(q1) và Q′1(q′1). Xét hàm số:

f (q) =√(a− q)(b− q)−

√(c− q)(d− q)− k (q < a)

Trong đó ta chọn gốc tọa độ thỏa mãn ad = bc. Ta có

f ′(q) =12

(c + d− 2q√(c− q)(d− q)

− a + b− 2q√(a− q)(b− q)

).

Đặt a′ =√

a− q, b′ =√

b− q, c′ =√

c− q, d′ =√

d− q. Ta sẽ chứng minh dấu củaf ′(q) luôn không đổi khi q < a, tức là (a′c′ − b′d′)(b′c′ − a′d′) không đổi dấu. Chú ýrằng do a < b < c < d nên a′c′ < b′d′, hơn nữa ta có:

(b′c′)2 − (a′d′)2 = (b− q)(c− q)− (a− q)(d− q) = (a + d− b− c)q

Do q < a < 0 nên (b′c′)2 − (a′d′)2 không đổi dấu, từ đó f ′(q) không đổi dấu. Nhưngrõ ràng f (q) có nghiệm nên nghiệm đó phải duy nhất. Vậy Q′1 ≡ Q1, hay tiếp tuyếnchung ngoài của (O1) và (O4), của (O2) và (O3) cắt nhau trên `.

Nhận xét. Ở bài toán này, tuy việc xây dựng các điểm mới tự nhiên nhưng việc chứng minhtrùng nhau khá phức tạp. Việc sử dụng công cụ đại số mà ở đây là tọa độ trục giúp việc chứngminh sự tồn tại duy nhất của các điểm thỏa mãn hệ thức đại số chặt chẽ hơn.

34

Page 35: I HÅC QUÈC GIA TP. HCM · 2020. 10. 18. · Ngo i b i vi¸t cõa anh Ph¤m Tu§n Huy tø n«m 2013, c¡c b¤n s³ th§y r§t nhi·u b i vi¸t m t¡c gi£ v¨n cán l håc sinh cõa

2.3. Các bài toán liên quan đến việc đảo cách xác định đối tượng, thay bằng nhữngđối tượng có cách xác định đặc biệt

Ví dụ 9. (Canada MO 2013) Gọi O là tâm đường tròn ngoại tiếp tam giác ABC nhọn.Lấy P trên AB là điểm thỏa mãn ∠BOP = ∠ABC, lấy Q trên AC là điểm thỏa mãn∠COQ = ∠ACB. Chứng minh rằng đường thẳng đối xứng với BC qua PQ tiếp xúc vớiđường tròn ngoại tiếp tam giác APQ.

Phân tích. Qua quan sát, có thể giao điểm của d (đường đối xứng với BC qua PQ) và(APQ) đồng thời là giao điểm thứ hai của (ABC) và (APQ). Tiếp điểm này có nhiều tínhchất đặc biệt dễ khai thác, ta sẽ bắt đầu từ đây. Gọi giao điểm thứ hai của (ABC) và (APQ)

là K. Kẻ tiếp tuyến từ K của (APQ). Hơn nữa, ta có thể thấy AK ‖ BC. Tính chất này gợimở cho ta nhiều điều, đồng thời khai thác được d dễ dàng hơn.

O

A

B C

P

Q

K

O′

MK′

Lời giải. Gọi O′ là tâm của (APQ). Ta có ∠POQ = ∠POA + ∠AOQ = 2∠BCA−∠ABC + 2∠ABC−∠BCA = 180◦ −∠BAC nên O ∈ (APQ). Lại có:

∠O′OA = 90◦ −∠AQO = 90◦ − (90◦ −∠ABC +∠BCA) = ∠ABC−∠BCA

Gọi E là giao điểm của AO và BC và M là trung điểm BC. Ta có:

∠MOE = 90◦ − (180◦ − 2∠ABC + 90◦ −∠BAC) = ∠ABC−∠BCA

Từ đó M, O, O′ thẳng hàng. Do đó K đối xứng với A qua OO′ hay AK ‖ BC. Gọi K′

đối xứng với K qua PQ thì ∠KQK′ = 2∠KQP = 2∠KCB, lại có QK = QK′ nên Q

35

Page 36: I HÅC QUÈC GIA TP. HCM · 2020. 10. 18. · Ngo i b i vi¸t cõa anh Ph¤m Tu§n Huy tø n«m 2013, c¡c b¤n s³ th§y r§t nhi·u b i vi¸t m t¡c gi£ v¨n cán l håc sinh cõa

là tâm của (KK′C). Từ đó ∠QK′C = ∠QCK′. Tương tự thì ∠PK′B = ∠BPK′. Do đó∠BK′P +∠PK′Q +∠QK′C = 180◦ hay K′ ∈ BC.

Do ∠QK′C = ∠BCA = ∠KPQ = ∠K′PQ nên BC là tiếp tuyến tại K′ của (K′PQ).Hơn nữa d đối xứng với BC qua PQ nên d trùng với tiếp tuyến tại K của (KPQ).

Ví dụ 10. (Gặp gỡ Toán học V) Cho tam giác ABC cân tại A, D là trung điểm BC. Lấy Enằm ngoài tam giác ABC sao cho CE ⊥ AB và BE = BD. Gọi M là trung điểm BE. F nằmtrên cung nhỏ AD của (ABD) sao cho MF ⊥ BE. Chứng minh rằng ED ⊥ FD.

B C

A

D

E

M

FT

L

K

Lời giải. Lấy F′ sao cho DE ⊥ DF′ và MF′ ⊥ BE. Khi đó tứ giác MEF′D nội tiếp.Gọi K là giao điểm của ME và DF′ thì KD.KF′ = KM.KE. Gọi T là giao điểm củaF′M và DE, T và L là các giao điểm của (TDF′) và (TMB). Khi đó L ∈ BF′ do∠TLF′ = ∠TLB = 90◦. Do MEF′D nội tiếp nên ∆TDF′ ∼ ∆TME. Mặt khác ta có∆TME = ∆TMB nên ∠TLM = ∠TBM = ∠TFD. Sử dụng nhận xét về tính duy nhấtthì L ∈ DM.

Chú ý rằng ∠F′TD = ∠MTE = 90◦ − ∠MET = 90◦ − ∠BDE = ∠BDK nên(F′TD) tiếp xúc BC và tâm O′ của (F′TD) thuộc AD. Dẫn đến ∠BDL = ∠BF′D. Dođó ∠BF′D = ∠BDM = ∠BAD nên F′ ∈ (BAD). Áp dụng tính chất về tính duy nhất

của giao điểm giữa đường thẳng qua M và vuông góc với BE và cung_

ADB, ta phảicó F′ ≡ F. Vậy FD ⊥ DE.

36

Page 37: I HÅC QUÈC GIA TP. HCM · 2020. 10. 18. · Ngo i b i vi¸t cõa anh Ph¤m Tu§n Huy tø n«m 2013, c¡c b¤n s³ th§y r§t nhi·u b i vi¸t m t¡c gi£ v¨n cán l håc sinh cõa

Nhận xét. 3 mối liên hệ chính của bài toán là

MF ⊥ ME, (3)

AF ⊥ FB, (4)

DE ⊥ DF. (5)

Do đó, ở trường hợp gốc, ta có (3), (4) và cần chứng minh (5). Tuy nhiên việc kết hợp (5), (3)lại mang đến nhiều tính chất thuận lợi hơn và vì vậy, ta có thể chuyển sang sử dụng bộ điềukiện (5), (3) để chứng minh (4) như một hướng đi tự nhiên thông qua những suy luận về sốhữu hạn các giao điểm giữa đường thẳng và đường thẳng, đường thẳng và đường tròn.

Ví dụ 11. (Gặp gỡ Toán học V) Cho tứ giác ABCD nội tiếp đường tròn (O). Giả sử AB cắtCD tại E, AD cắt BC tại F. Gọi M, N lần lượt là trung điểm của AC, BD. Gọi d là trungtrực của MN. Giả sử P, Q là các điểm thuộc d sao cho PE ⊥ FM và FQ ⊥ EM. Gọi K làtâm đường tròn ngoại tiếp tam giác NEF. Chứng minh rằng tam giác KPQ cân.

D

A

B

C E

F

M

N

Q

K

PT

Lời giải. Gọi Q′ là giao điểm của đường thẳng qua M vuông góc với MF và FQ. Xácđịnh P′ tương tự. Gọi T là giao điểm của FQ′ và EP′. Do tứ giác ABCD nội tiếp nêndễ dàng có được ∆FBN ∼ ∆FAM và ∆EMA ∼ ∆END, dẫn đến:

∠FNE = ∠FNB +∠BNE = ∠EMA +∠EMC = 180◦ −∠FME

37

Page 38: I HÅC QUÈC GIA TP. HCM · 2020. 10. 18. · Ngo i b i vi¸t cõa anh Ph¤m Tu§n Huy tø n«m 2013, c¡c b¤n s³ th§y r§t nhi·u b i vi¸t m t¡c gi£ v¨n cán l håc sinh cõa

Ta có ∠FTE = 180◦−∠FME = ∠FNE nên T ∈ (ENF). Để ý rằng ∆Q′MF ∼ ∆P′ME(g.g) và Q′MP′T là hình bình hành. Mặt khác do

MFNF

=FCFD

=EAED

=MENE

Từ đó dẫn đếnQ′FP′E

=MFME

=NFNE

hay ∆NQ′F ∼ ∆NP′E (c.g.c). Do đó

NQ′

NP′=

FQ′

EP′=

MQ′

MP′=

TP′

TQ′. (6)

Mặt khác 4Q′NP′ ∼ 4ENF (c.g.c) nên ∠Q′NP′ = ∠ENF = ∠ETF = ∠Q′TP′. Dođó N ∈ (Q′TP′). Kết hợp với (6) thì TNQ′P′ là hình thang cân, dẫn đến

P′N = QT = MP′

Từ đó P′ thuộc trung trực của MN. Áp dụng tính chất về tính duy nhất, ta có P′ ≡ P.Tương tự thì Q ≡ Q′. Do đó:

PEQF

=PMQM

=QTPT

Vậy PE.PT = QT.QF hay KP2 = KQ2 và ta thu được KP = KQ.

Nhận xét. Xuất phát từ tính chất quen thuộc ∠EMF + ∠ENF = 180◦, EP ⊥ FM, vàFQ ⊥ EM ta liên tưởng đến việc xác định T là giao điểm FQ, EP. Từ đó, qua trực quan, có thểthấy MQTP là hình bình hành hay MQ ⊥ MF, MP ⊥ ME. Với các điều kiện MP ⊥ ME,P thuộc trung trực MN, EP ⊥ FM, sau khi đảo lại và sử dụng bộ tính chất vuông góc, ta cóthể khai thác sự kiện đồng dạng một cách hiệu quả. Rõ ràng tính chất thẳng hàng hoặc đồngviên liên hệ với sự đồng dạng là một kết quả khá thú vị.

Ví dụ 12. (Serbia 2013) Cho tam giác ABC nhọn có O là tâm đường tròn ngoại tiếp. Đườngtròn Euler (ω) và đường tròn ngoại tiếp tam giác OBC cắt nhau tại P, Q. Chứng minh rằngAP, AQ là hai tia đối xứng đẳng giác trong ∠BAC.

Phân tích. Dựa vào trực quan, có thể nhận xét AP.AQ = 12 AB.AC. Rõ ràng với tính chất

trên ta thu được nhiều cặp tam giác đồng dạng, tạo dựng liên hệ rõ ràng hơn giữa hai điểm.Vì vậy, xuất phát từ đây có thể ta sẽ thu được nhiều kết quả hơn. Như vậy, ta sẽ chọn điểm Q′

có tính chất đẳng giác và hệ thức, chỉ cần chứng minh Q′ lần lượt thuộc (ω) và (BOC).

38

Page 39: I HÅC QUÈC GIA TP. HCM · 2020. 10. 18. · Ngo i b i vi¸t cõa anh Ph¤m Tu§n Huy tø n«m 2013, c¡c b¤n s³ th§y r§t nhi·u b i vi¸t m t¡c gi£ v¨n cán l håc sinh cõa

O

A

B C

K N

M

T

LP

Q

Lời giải. Gọi Q′ là điểm thuộc tia đẳng giác với AP trong ∠BAC để

AP.AQ′ =12

AB.AC

Gọi M, N, K lần lượt là trung điểm của BC, CA, AB. Từ cách dựng Q′ thì ta có được∆ANQ′ ∼ ∆APB, ∆AQ′B ∼ ∆ANP, ∆AKQ′ ∼ ∆APC, ∆AQ′C ∼ ∆AKP (c.g.c). Kếthợp với các cặp góc tương ứng thì:

∠BQ′C = 360◦ −∠AQ′C−∠AQ′B = 360◦ −∠AKP−∠ANP

Từ đó ∠BQ′C = ∠BAC +∠KMN = 2∠BAC = ∠BOC, dẫn đến Q′ ∈ (BOC). GọiT, L lần lượt là các giao điểm của (BOC) với AB, AC thì:

∠NQ′K = ∠NQ′A +∠AQ′K = ∠PBA +∠ACP = ∠TCL = ∠BAC = ∠MNK

Do đó Q′ ∈ (MNK). Hơn nữa (MNK) và (BOC) có nhiều nhất hai giao điểm, trongđó có P, nên Q′ ≡ Q. Vậy AP, AQ là hai đường đối xứng đẳng giác trong ∠BAC.

Ví dụ 13. Cho tam giác ABC nhọn nội tiếp đường tròn (O). Tiếp tuyến tại A của (O) cắtBC tại D. E là điểm đối xứng của A qua D. Đường thẳng qua B vuông góc với AB cắt trungtrực của BC tại K. Chứng minh rằng BE ⊥ AK.

39

Page 40: I HÅC QUÈC GIA TP. HCM · 2020. 10. 18. · Ngo i b i vi¸t cõa anh Ph¤m Tu§n Huy tø n«m 2013, c¡c b¤n s³ th§y r§t nhi·u b i vi¸t m t¡c gi£ v¨n cán l håc sinh cõa

O

A

B CD

E

K

Lời giải. Gọi K′ là điểm sao cho hai tam giác CAB và COK′ đồng dạng cùng hướng.Từ đó dễ thấy K′ thuộc trung trực của BC và:

OK′

R=

BAAC

=DBDA

=DBDE

Lại có DE ⊥ OA và DB ⊥ OK nên ∠EDB = ∠AOK′ hay ∆AOK′ ∼ ∆EDB (c.g.c).Do đó ∠DEB = ∠OAK′ = 90◦ −∠EAK′ nên EB ⊥ AK′. Hơn nữa có được biến đổi∠K′BC = ∠OAC = 90◦ −∠ABC nên AB ⊥ BK′. Vậy K′ ≡ K hay BE ⊥ AK.

Nhận xét. Đi ngược từ kết luận có thể thu được một tính chất rất đẹp đẽ của điểm K là∆COA ∼ ∆CAB. Từ đây, ta sử dụng việc đảo các điều kiện trong hệ ∆COA ∼ ∆CAB (vaOK ⊥ BC), BK ⊥ BA (và OK ⊥ BC). Trong bài toán hình học, khác với các mảng khác, tanhiều lúc không thể xác định được đường đi hay thu được nhiều chi tiết từ giả thiết, hoặc từđó chỉ có một số dữ kiện phục vụ được cho kết luận. Nhu cầu đi từ kết luận và đối chiếu vớigiả thiết để có định hướng là khá tự nhiên.

Ví dụ 14. (IMO Shortlist 2011) Cho tam giác ABC cân tại A. D là trung điểm của AC.Đường phân giác ∠BAC cắt đường tròn ngoại tiếp tam giác BDC tại E (E nằm trong tamgiác ABC). BD cắt đường tròn ngoại tiếp tam giác AEB tại điểm thứ hai F. Gọi I là giaođiểm của AF và BE. K là giao điểm của CI và BD. Chứng minh rằng I là tâm đường trònnội tiếp của tam giác KAB.

40

Page 41: I HÅC QUÈC GIA TP. HCM · 2020. 10. 18. · Ngo i b i vi¸t cõa anh Ph¤m Tu§n Huy tø n«m 2013, c¡c b¤n s³ th§y r§t nhi·u b i vi¸t m t¡c gi£ v¨n cán l håc sinh cõa

B C

A

DE

F

I

K

D′

G

L

Lời giải. Gọi K′ là điểm thuộc đoạn BD sao cho DK′.DB = DA2, D′ là trung điểmcủa AB. L là giao điểm của BE và AC. Vì ED = ED′ nên BE là phân giác ∠DBA. Ápdụng định lí Menelaus cho tam giác LBD, ta có

ILIB· FB

FD· AD

AL= 1

Từ đóILIB

=ALAD· FD

FB. Lại có DK′ =

DA2

DB, K′B = DB− DK′ =

DB2 − DA2

DBnên:

K′BK′D

=DB2 − DA2

DA2

Dễ dàng có đượcCDCL

=DA

DA + DL. Xét biến đổi:

ILIB· K′B

K′D· CD

CL=

ALAD· FD

FB· DB2 − DA2

DA2 · DADA + DL

Ta cóAL

DA + DL=

ALAL + 2DL

=1

1 + 2 · DB2DA

=DA

DA + DB. Dẫn đến:

ALAD· AD

AD + DL· DB2 − DA2

DA2 =DB− DA

DA

41

Page 42: I HÅC QUÈC GIA TP. HCM · 2020. 10. 18. · Ngo i b i vi¸t cõa anh Ph¤m Tu§n Huy tø n«m 2013, c¡c b¤n s³ th§y r§t nhi·u b i vi¸t m t¡c gi£ v¨n cán l håc sinh cõa

Gọi G là trọng tâm của tam giác ABC thì GF.GB = GE.GA. Chú ýEGEA

=BGBA

nên ta

có đượcEGAG

=BG

BA + BG. Do đó kết hợp lại thì GF =

GA2

BA + BG. Ta có:

FD = GD + GF = GD +GA2

AB + BG

Tương tự thì FB = GB− GF = GB− GA2

AB + BG. Lưu ý rằng:

94

GA2 =14

(4AB2 − BC2

),

94

GB2 =14

(2BC2 + AB2

)

Từ đó có được:

FBFD

=GB2 − GA2 + GB.GA

12

GB.BG +12

GB.AB + GA2=

13(

BC2 − AB2)+ GB.AB

AB2

2+

12

GB.GA

Ta chứng minh rằngFBFD

=

32

GB− 12

AB

12

AB. Thật vậy, điều này tương đương với:

AB(

13

(BC2 − AB2

)+ AB.GB

)=

12

(AB2 + AB.GB

)(3GB− AB)

Viết lại thành:

2(

13

(BC2 − AB2

)+ AB.GB

)= (3GB− AB) (GB + AB)

Đẳng thức cuối cùng đúng sau khi khai triển và thu gọn. Theo định lý Menelaus chotam giác BLD thì K′, I, C thẳng hàng. Theo tính chất duy nhất, ta có K′ ≡ K. Từ đây∠KAD = ∠DBA = 2∠EBA = 2∠FAE, do đó AI là phân giác ∠BAK. Vậy I là tâmđường tròn nội tiếp tam giác KAB. Bài toán kết thúc.

Nhận xét. Điểm K xác định bởi CI ∩ BD khá rời rạc so với các quan hệ khác. Tuy nhiên, nếuđi ngược từ kết luận (cụ thể là biến đổi tương đương), ta lại có tính chất thú vị DK.DB = DA2

và có lẽ đây là một khởi đầu thuận lợi hơn, vì sau đó để chứng minh C, I, K thẳng hàng, ta đãcó định lý Menelaus và tính toán đại số hỗ trợ; các quan hệ tỉ lệ độ dài trong bài khá rõ ràng.

Ví dụ 15. (USATST 2013) Cho tam giác ABC vuông tại C có đường cao CH. Lấy X thuộcCH. L, K lần lượt thuộc AX, BX sao cho AK = AC và BL = BC. Chứng minh rằng khi Xthay đổi trên CH thì (HKL) luôn đi qua một điểm cố định khác H.

42

Page 43: I HÅC QUÈC GIA TP. HCM · 2020. 10. 18. · Ngo i b i vi¸t cõa anh Ph¤m Tu§n Huy tø n«m 2013, c¡c b¤n s³ th§y r§t nhi·u b i vi¸t m t¡c gi£ v¨n cán l håc sinh cõa

A

C

B

X

LK

Q

P

V MH

Q′

C1

C′

C2

Lời giải. Ký hiệu (B) là (B; BC) và (A) là (A; AC). Gọi P, Q lần lượt là giao điểmthứ hai của AX và BX với (B) và (A). V là giao điểm của PQ và AB. Ta có đượcBK.BQ = BC2 = BA.BH và BC tiếp xúc (A) nên (XB, QK) = −1. Gọi M là giao điểmPK, AB thì (AB, VM) = P(XB, QK) = −1. Tương tự, gọi M′ là giao điểm QL, AB thì(AB, VM′) = −1, dẫn đến M′ ≡ M hay KP, LQ, AB đồng quy. Do CX là trục đẳngphương của (A), (B) nên XR.XQ = XL.XP, từ đó QLKP là tứ giác nội tiếp.

Ta chứng minh kết quả sau: Cho (A) và (B) là hai đường tròn trực giao với C vàC′ là hai giao điểm. Gọi V′ là tâm vị tự ngoài của (A), (B). Lấy Q ∈ (A) sao cho đoạnVQ không cắt (A). Đường thẳng VQ cắt (B) tại P′, P sao cho P′ thuộc đoạn PQ. Khiđó các đường thẳng AP, BQ, CC′ đồng quy.

Thật vậy, gọi X′ là giao điểm của AP và CC′. Bởi (A), (B) trực giao nên AC, AC′

là tiếp tuyến của (B), dẫn đến PX′ là đường đối trung của tam giác PCC′. Từ đó:

CX′

C′X′=

(PCPC′

)2

Gọi X′′ là giao điểm của BQ và CC′ thì tương tự, ta cũng cóCX′′

C′X′′=

(QCQC′

)2

. Gọi

Q′ là giao điểm của PQ và (A). Xét phép vị tự tâm V′ như sau:

HV′ : (B) 7→ (A), P 7→ Q′, C 7→ C1, C′ 7→ C2

43

Page 44: I HÅC QUÈC GIA TP. HCM · 2020. 10. 18. · Ngo i b i vi¸t cõa anh Ph¤m Tu§n Huy tø n«m 2013, c¡c b¤n s³ th§y r§t nhi·u b i vi¸t m t¡c gi£ v¨n cán l håc sinh cõa

Từ đây thìPCPC′

=Q′C1

Q′C2. Hơn nữa xét (A) cùng các tam giác đồng dạng thì:

QC′

Q′C2=

V′QV′C1

=V′QV′C2

=QC

Q′C1

Kết hợp các điều kiện trên, ta có được:

CX′

C′X′=

(PCPC′

)2

=

(Q′C1

Q′C2

)2

=

(QCQC′

)2

=CX′′

C′X′′

Các điểm X′.X′′ cùng chia trong đoạn CC′ theo cùng một tỉ số nên X′ ≡ X′′ hay làAP, BQ, CC′ đồng quy. Quay trở lại bài toán.

Gọi T là giao điểm của V′Q và (B). Áp dụng bổ đề trên thì AT, BQ, CC′ đồngquy, nhưng điểm đồng quy chính là X nên T ≡ P, dẫn đến V ≡ V′. Chú ý rằng:

VAVB

=R(A)

R(B)=

CACB

Từ đó CV là phân giác ngoài của tam giác ABC. Lại có (AB, VM) = −1 nên CM làphân giác trong của tam giác ABC. Ta có ∠VPC = ∠VQ′C1 = ∠VCQ nên

VP.VQ = VC2 = VH.VM

Hơn nữa Q(VL, KA) = A(VL, KQ) = −1 nên LK đi qua V. Do QLKP nội tiếp nênVL.VK = VP.VQ = VH.VM. Vậy (HKL) luôn đi qua điểm M cố định.

Nhận xét. Bài toán này hầu như giống bài toán 5 tại IMO 2012 về cách xác định các đốitượng nên ta có thể nghĩ đến việc dựng (A), (B) cùng các giao điểm. Từ sau đó, những suyluận và quan sát giúp biến đổi linh hoạt trong việc sử dụng các điều kiện và điểm trùng nhau.chẳng hạn như PK, QL, (HLK), AB đồng quy hay V là chân đường phân giác ngoài - tâm vịtự ngoài (rõ ràng tính chất này cụ thể hơn việc xác định V = PQ ∩ AB). Bài toán cho thấy rõviệc chứng minh điểm trùng nhau phối hợp giữa các tỉ lệ đại số, hàng điểm điều hòa,...

Ví dụ 16. (Bulgaria 1998 - Vietnam TST 2013) Cho tứ giác ABCD có các cặp cạnh khôngsong song nội tiếp đường tròn (O; R). Gọi E là giao điểm của AC, BD. Đường phân giáctrong của ∠AEB lần lượt cắt các đường thẳng AB, BC, CD, DA lần lượt tại M, N, P, Q.

a) Chứng minh rằng các đường tròn (AQM), (BMN), (CPN), (DPQ) cùng đi qua mộtđiểm. Gọi điểm đó là K.

b) Đặt min{AC, BD} = m. Chứng minh rằng OK ≤ 2R2√

4R2 −m2.

44

Page 45: I HÅC QUÈC GIA TP. HCM · 2020. 10. 18. · Ngo i b i vi¸t cõa anh Ph¤m Tu§n Huy tø n«m 2013, c¡c b¤n s³ th§y r§t nhi·u b i vi¸t m t¡c gi£ v¨n cán l håc sinh cõa

O

A

D

C

B

E

Q

L

N

M

P F

K

Lời giải. Gọi F là giao điểm của AB và CD và L là giao điểm của AD và BC.a) Áp dụng định lí Miquel thì (FMP), (BMN), (CNP), (FBC) có một điểm chung,

(FMP), (QMA), (QPD), (FAD) có một điểm chung; (BMN), (QMA), (LAB) có mộtđiểm chung; (NPC), (QPD), (LCD) có một điểm chung. Gọi K′ là giao điểm của(FBC) và (FAD) thì K nằm trên (LAB), (LCD). Bởi vì (FMP) và (FBC) chỉ có đúngmột giao điểm ngoài F, nếu ta chứng minh được K′ ∈ (FMP) thì ta cũng phải có(BMN), (CNP), (QMA), (QPD) có 1 điểm chung. Ta sẽ chứng minh điều này.

Thật vậy, ∆K′BC, ∆K′AD đồng dạng cùng hướng nên K′ là tâm của phép vị tựquay biến A 7→ D, B 7→ C, AB 7→ DC. Lại có tỉ lệ sau do tứ giác ABCD nội tiếp vàtính chất phân giác:

MAMB

=EAEB

=EDEC

=PDPC

Nghĩa là qua cùng phép vị tự quay trên , ta có M 7→ P. Do đó:

(MK′, MP) ≡ (BK′, BC) ≡ (FK′, FC) ≡ (FK′, FP) (mod π)

Từ đây thì K′ ∈ (FMP). Vậy theo lập luận, ta phải có K′ là điểm chung của các đườngtròn (BMN), (CNP), (QMA), (QPD), dẫn đến K′ ≡ K.

b) Gọi T là điểm thuộc tia OE sao cho OE.OT = R2. Do đó OE.OT = OA2 và∠OAC = ∠OTA. Tương tự thì ∠OCA = ∠OTC và ∠OBD = ∠OTB. Từ đó thì:

∠BTC = ∠OCA−∠OBD = ∠BCD−∠CDA = ∠AFD

45

Page 46: I HÅC QUÈC GIA TP. HCM · 2020. 10. 18. · Ngo i b i vi¸t cõa anh Ph¤m Tu§n Huy tø n«m 2013, c¡c b¤n s³ th§y r§t nhi·u b i vi¸t m t¡c gi£ v¨n cán l håc sinh cõa

Điều này dẫn đến T ∈ (BCF). Tương tự thì T ∈ (ADF). Áp dụng tính chất duy nhấtvề giao điểm, ta phải có T ≡ K. Do đó OE.OK = R2. Vậy ta có:

OK =R2

OE≤ R2√

R2 − m2

4

=2R2

√4R2 −m2

Bài toán được chứng minh hoàn toàn..

Nhận xét. Bài toán trên không quá phức tạp nhưng tích hợp được nhiều tính chất thú vị.Hình ảnh các đường tròn gợi ý đến định lý Miquel, hệ quả là việc xét các giao điểm của cácđường tròn (FBC), (FAD), (FMN). Việc dùng điểm trùng nhau thường hiệu quả trong cácbài toán về sự đồng quy 3 đường tròn hoặc đường thẳng, thông qua việc xét sự đồng quy của 3đường hình học khác. Việc xác định K thông qua giao điểm của các đường tròn là khá phức tạpvà khó liên hệ với định tính. Ý sau tương tự bài toán Bulgaria 1998, khi có OE.OK = OA2

thì ta có thể thu được các tam giác đồng dạng và nhiều tính chất khác.

3. Một số bài tập tham khảo

Bài toán 1. Cho tam giác ABC nội tiếp đường tròn (O). Đường phân giác trong của ∠BACcắt (O) tại D. Gọi I là tâm đường tròn nội tiếp tam giác ABC. K là điểm đối xứng của I quaBC. KD cắt (O) tại M. Chứng minh rằng tứ giác IODM nội tiếp.

Bài toán 2. (Iran 1997) Cho tam giác ABC nội tiếp đường tròn (O). M là điểm thay đổitrên cung BC không chứa A. Gọi I, J lần lượt là tâm đường tròn nội tiếp của các tam giácABM và ACM. Chứng minh rằng đường tròn ngoại tiếp của tam giác MI J luôn đi qua mộtđiểm cố định khi M thay đổi.

Bài toán 3. (Chọn đội tuyển KHTN 2012) Cho tam giác ABC không cân nội tiếp đườngtròn (O). P là điểm bất kì trong tam giác ABC và không trùng với O. AP cắt (O) tại D khácA. Lấy DE, AF là đường kính của (O). EP, FP lần lượt cắt (O) tại G, H khác E, F. Giả sửAH cắt DG tại K. Gọi L là hình chiếu của K lên đường thẳng OP.

a) Chứng minh rằng 4 điểm A, L, K, D cùng thuộc một đường tròn (S).b) Chứng minh rằng đường thẳng OP cắt EF tại điểm T thuộc (S).

Bài toán 4. (Chọn đội tuyển TP.HCM 2013) Cho tam giác ABC cân tại A nội tiếp đườngtròn (O). Điểm M thay đổi trên (O) và M không thuộc AO. Đường thẳng vuông góc vớiAM tại M cắt BC tại N. Đường trung trực của MN cắt AB, AC lần lượt tại E, F. Tìm quỹtích trọng tâm của tam giác AEF khi M thay đổi.

Bài toán 5. (USAMO 2008) Cho tam giác ABC không cân có M, N, P lần lượt là trungđiểm của BC, CA, AB. Đường trung trực của AB và AC cắt AM lần lượt tại D và E. BDcắt CE tại F. Chứng minh rằng tứ giác APFN nội tiếp.

46

Page 47: I HÅC QUÈC GIA TP. HCM · 2020. 10. 18. · Ngo i b i vi¸t cõa anh Ph¤m Tu§n Huy tø n«m 2013, c¡c b¤n s³ th§y r§t nhi·u b i vi¸t m t¡c gi£ v¨n cán l håc sinh cõa

Bài toán 6. (Chọn đội tuyển Hải Phòng 2013) Cho hai đường tròn (O1) và (O2) cắt nhautại hai điểm phân biệt A và B sao cho O1, O2 nằm khác phía với đường thẳng AB. Một đườngthẳng thay đổi qua A cắt (O1), (O2) lần lượt tại C, D khác A (A nằm giữa C và D). Gọi P, Qlần lượt là hình chiếu vuông góc của B xuống tiếp tuyến tại C của (O1) và tiếp tuyến tại Dcủa (O2). Chứng minh rằng đường thẳng PQ luôn tiếp xúc với một đường tròn cố định.

47

Page 48: I HÅC QUÈC GIA TP. HCM · 2020. 10. 18. · Ngo i b i vi¸t cõa anh Ph¤m Tu§n Huy tø n«m 2013, c¡c b¤n s³ th§y r§t nhi·u b i vi¸t m t¡c gi£ v¨n cán l håc sinh cõa
Page 49: I HÅC QUÈC GIA TP. HCM · 2020. 10. 18. · Ngo i b i vi¸t cõa anh Ph¤m Tu§n Huy tø n«m 2013, c¡c b¤n s³ th§y r§t nhi·u b i vi¸t m t¡c gi£ v¨n cán l håc sinh cõa

CÁC BÀI TOÁN VỀ HỆ SỐ NHỊ THỨC

Nguyễn Nguyễn

Lớp chuyên Toán khoá 2016 - 2019

GIỚI THIỆU. Bài viết khai thác các tính chất của hệ số nhị thức và ứng dụng quacác bài toán Olympic.

1. MỘT SỐ ĐỊNH LÝ LIÊN QUAN

Định nghĩa 1. Với k, n ∈ N và n > 0, một tổ hợp chập k của n phần tử là số cách chọn ra k

phần tử mà không kể đến thứ tự giữa chúng từ n phần tử cho trước. Ký hiệu là Ckn hay

(nk

).

Công thức xác định(

nk

)là(

nk

)=

n!k!(n− k)!

. Quy ước rằng với k > n thì(

nk

)= 0.

Để thuận tiện, ký hiệu(

nk

)sẽ được sử dụng trong toàn bộ phạm vi bài viết.

Một số định lý liên quan thường được sử dụng:

Định lý 1 (Khai triển Newton). Với x, y ∈ R và n ∈ N∗: (x + y)n =n

∑i=1

(nk

)xkyn−k.

Định lý 2 (Công thức Pascal). Với n, k ∈ N và n > 0:(

n + 1k + 1

)=

(nk

)+

(n

k + 1

)và(

nk

)=

nk

(n− 1k− 1

).

Định lý 3 (Hệ thức Vandermonde). Với k, m, n ∈ N:k

∑i=0

(mi

)(n

k− i

)=

(m + n

k

).

Chứng minh. Xác định hệ số của xk trong khai triển (x + 1)m+n bằng 2 cách.

Định lý 4. Với m, n ∈ N∗ và m ≥ n thì m là ước của(

mn

)(m, n).

Hệ quả 1. Với p là số nguyên tố, ta luôn có p là ước của(

pk

)với k ∈ {1, 2, ..., p− 1}.

49

Page 50: I HÅC QUÈC GIA TP. HCM · 2020. 10. 18. · Ngo i b i vi¸t cõa anh Ph¤m Tu§n Huy tø n«m 2013, c¡c b¤n s³ th§y r§t nhi·u b i vi¸t m t¡c gi£ v¨n cán l håc sinh cõa

Định lý 5. Với p là số nguyên tố, ta luôn có được(

p− 1k

)≡ (−1)k (mod p) với k ∈

{0, 1, ..., p− 1}.

Định lý 6 (De Polignac, Legendre). Cho n ∈ N∗ và p là số nguyên tố, ta có:

vp(n!) =∞

∑i=1

⌊npi

⌋=

n− sp(n)p− 1

.

Trong đó vp(n) là số mũ lớn nhất của p trong phân tích n ra thừa số nguyên tố, bnc là sốnguyên lớn nhất không vượt quá n và sp(n) là tổng các chữ số của n trong hệ cơ số p.

Chứng minh. Trước hết ta có nhận xét rằng có đúng⌊

npi

⌋−⌊

npi+1

⌋số nguyên

dương t ≤ n mà vp(t) = i với i = 1, 2, .... Hơn nữa khi i đủ lớn thì⌊

npi

⌋= 0 nên từ

đó ta có:

vp(n!) =∞

∑i=1

(⌊npi

⌋−⌊

npi+1

⌋)=

∑i=1

⌊npi

⌋.

Đặt n =s

∑k=0

nk pk với 0 ≤ nk ≤ p− 1. Khi đó với j = 1, 2, ..., ta có⌊

npj

⌋=

s

∑i=j

ni pi−j.

Do đó:

∑j=1

⌊npj

⌋=

s

∑j=1

s

∑i=j

ni pi−j =s

∑i=1

ni

(i−1

∑j=0

pj

)=

s

∑i=1

ni.pi − 1p− 1

=n− sp(n)

p− 1.

Vậy ta đã chứng minh được vp(n!) =∞

∑i=1

⌊npi

⌋=

n− sp(n)p− 1

.

Định lý 7 (Kummer). Cho k, n ∈ N∗ với 1 ≤ k ≤ n và p là số nguyên tố. Khi đó ta có

vp

(nk

)đúng bằng số lần nhớ của phép cộng k và n− k trong hệ cơ số p.

Chứng minh. Phép chứng minh khá dài nên không trình bày lại tại đây, có thể thamkhảo trong bài viết của tác giả Trần Quốc Anh tại [1].

Hệ quả 2. Cho k, n ∈ N∗ với 1 ≤ k ≤ n và p là số nguyên tố. Khi đó ta có vp

(nk

)đúng

bằng số lần nhớ của phép trừ n cho k trong hệ cơ số p.

Định lý 8 (Lucas). Cho p là số nguyên tố và m, n ∈ N∗, m ≤ n. Lần lượt gọi m =

m1m2...mk(p) và n = n1n2...nk(p) là biểu diễn của m và n trong hệ cơ số p. Khi đó ta có:

(nm

)≡(

n1

m1

)(n2

m2

)...(

nkmk

)(mod p).

50

Page 51: I HÅC QUÈC GIA TP. HCM · 2020. 10. 18. · Ngo i b i vi¸t cõa anh Ph¤m Tu§n Huy tø n«m 2013, c¡c b¤n s³ th§y r§t nhi·u b i vi¸t m t¡c gi£ v¨n cán l håc sinh cõa

Chứng minh. Ta dễ dàng có p là ước của(

pi

k

)với i ∈ N∗ và 0 ≤ k ≤ pi − 1. Từ nhận

xét trên thì (1 + x)pi ≡ 1 + xpi(mod p). Do đó:

(1 + x)n = (1 + x)∑ki=0 pini ≡

n

∏i=0

(1 + xpi)ni (mod p)

Ta xác định hệ số của xm ở 2 vế của đồng dư thức trên. Hệ số của xm ở vế trái là(

nm

).

Hệ số của xm ở vế phải là của x∑ki=0 pimi và là

k

∏i=0

(ni

mi

).

Vậy ta có được(

nm

)≡(

n1

m1

)(n2

m2

)...(

nkmk

)(mod p).

Định lý 9 (Wolstenholme). Cho p ≥ 5 là số nguyên tố. Khi đó ta có:(

2pp

)≡ 2 (mod p3) hay

(2p− 1p− 1

)≡ 1 (mod p3).

Chứng minh. Theo đẳng thức Vandermonde, ta có:

(2pp

)=

p

∑i=0

(pi

)(p

p− i

)= 2 +

p−1

∑i=1

(pi

)2

Hơn nữa lại có được

p−1

∑i=1

(pi

)2

=p−1

∑i=1

p2

i2

(p− 1i− 1

)2

= p2p−1

∑i=1

1i2 .

Do đó ta chỉ cần chứng minh rằng:

p−1

∑i=1

1i2 ≡ 0 (mod p).

Nhưng{

1i2

}với i ∈ {1, 2, ..., p− 1} chính là

{12, 22..., (p− 1)2} với p ≥ 5 nên:

p−1

∑i=1

1i2 =

p−1

∑i=1

p2 =p(p− 1)(2p− 1)

6≡ 0 (mod p).

Vậy(

2pp

)≡ 2 (mod p3). Biến đổi dễ có

(2p− 1p− 1

)≡ 1 (mod p3).

51

Page 52: I HÅC QUÈC GIA TP. HCM · 2020. 10. 18. · Ngo i b i vi¸t cõa anh Ph¤m Tu§n Huy tø n«m 2013, c¡c b¤n s³ th§y r§t nhi·u b i vi¸t m t¡c gi£ v¨n cán l håc sinh cõa

Định lý 10 (Babbage). Cho p là số nguyên tố và a, b ∈ N∗. Khi đó(

apbp

)≡(

ab

)(mod p2).

(apbp

)≡(

ab

)(mod p2).

Chứng minh. Có thể tham khảo ở nhiều tài liệu, không chứng minh lại tại đây.

1.1. Chứng minh các đẳng thức tổ hợp

Đối với các bài toán chứng minh các đẳng thức tổ hợp, ngoài việc khai triển trựctiếp thì ta còn một số cách tiếp cận khác như tính bằng 2 cách, xây dựng song ánhgiữa các đối tượng tổ hợp liên quan đến đẳng thức cần chứng minh, so sánh số mũcủa 2 vế hay thậm chí là so sánh từng hạng tử xuất hiện trong 2 vế của đẳng thức...

Bài toán 1. Chứng minh rằng2n

∑i=0

(−1)i(

2ni

)2

= (−1)n(

2nn

).

Lời giải. Ta xác định hệ số của x2n trong khai triển (1 + x)2n(1− x)2n bằng 2 cách.

Ta có (1 + x)2n(1 − x)2n = (1 − x2)2n =2n

∑i=0

(2ni

)(−1)ix2i nên hệ số của x2n là

(−1)n(

2nn

). Mặt khác lại có:

(1 + x)2n(1− x)2n =

(2n

∑i=0

(2ni

)xi

)(2n

∑j=0

(2nj

)(−1)jxj

).

Do đó hệ số của x2n là:

∑i+j=2n

(−1)j(

2ni

)(2nj

)=

2n

∑i=0

(−1)i(

2ni

)(2n

2n− i

)=

2n

∑i=0

(−1)i(

2ni

)2

.

Vậy ta kết luận được rằng2n

∑i=0

(−1)i(

2ni

)2

= (−1)n(

2nn

).

Bài toán 2. Chứng minh rằngn

∑i=0

4n−i(

4n2n + 2i

)(2n + 2i

i

)=

(8n2n

).

Lời giải. Ta xác định hệ số của x4n trong khai triển của(

x +1x

)8nbằng 2 cách.

Do(

x +1x

)8n=

8n

∑i=0

(8ni

)x8n−i

(1x

)i=

8n

∑i=0

(8ni

)x8n−2i nên hệ số của x4n là

(8n2n

).

52

Page 53: I HÅC QUÈC GIA TP. HCM · 2020. 10. 18. · Ngo i b i vi¸t cõa anh Ph¤m Tu§n Huy tø n«m 2013, c¡c b¤n s³ th§y r§t nhi·u b i vi¸t m t¡c gi£ v¨n cán l håc sinh cõa

Mặt khác lại có:

(x +

1x

)8n=

(x2 +

1x2 + 2

)4n=

4n

∑i=0

(x2 +

1x2

)4n−i2i

=4n

∑i=0

4n−i

∑j=0

(4ni

)(4n− i

j

)2ix8n−2i−4j.

Để tính được hệ số của x4n, chọn i = 2n− 2j thì ta có được:

(8n2n

)=

4n

∑i=2n−2j

4n−i

∑j=0

22n−2j(

4n2n− 2j

)(2n + 2j

j

)=

n

∑i=0

4n−i(

4n2n + 2i

)(2n + 2i

i

).

Vậy ta kết luận được rằngn

∑i=0

4n−i(

4n2n + 2i

)(2n + 2i

i

)=

(8n2n

).

Bài toán 3. Chứng minh rằngn

∑i=1

(−1)i−1

i

(ni

)= 1 +

12+

13+ ... +

1n

.

Lời giải. Đặt un =n

∑i=1

(−1)i−1

i

(ni

). Rõ ràng u1 = 1. Ta có:

un+1 =n+1

∑i=1

(−1)i−1

i

(n + 1

i

)=

(−1)n

n + 1+

n

∑i=1

(−1)i−1

in + 1

n + 1− i

(ni

)

=n

∑i=1

(−1)i−1

i

(ni

)+

1n + 1

n+1

∑i=1

(−1)i−1(

n + 1i

)

= un −1

n + 1

(n+1

∑i=0

(−1)k(

n + 1k

)− 1

)

= un −1

n + 1(0− 1) = un +

1n + 1

.

Từ đó bằng phép quy nạp thìn

∑i=1

(−1)i−1

i

(ni

)= 1 +

12+

13+ ... +

1n

.

1.2. Các bài toán liên quan đến tính chia hết

Trong các bài toán sau, việc sử dụng các tính chất số học của hệ số nhị thức để sosánh số mũ đúng giữa 2 vế hoặc sử dụng các đẳng thức tổ hợp để chứng minh các bàitoán chia hết, hơn nữa sử dụng tính chia hết để chứng minh các đẳng thức tỏ ra rấthiệu quả.

Bài toán 4. Tính giá trị của A = gcd((

2n1

),(

2n3

), ...,

(2n

2n− 1

)).

53

Page 54: I HÅC QUÈC GIA TP. HCM · 2020. 10. 18. · Ngo i b i vi¸t cõa anh Ph¤m Tu§n Huy tø n«m 2013, c¡c b¤n s³ th§y r§t nhi·u b i vi¸t m t¡c gi£ v¨n cán l håc sinh cõa

Lời giải. Ta có đẳng thứcn−1

∑i=0

(2n

2i− 1

)= 22n−1 nên 22n−1 ... A hay A = 2m với m ∈ N.

Hơn nữa ta có(

2n1

)= 2n

... 2m nên n... 2m−1 hay m ≤ v2(n) + 1 = p. Do đó A ≤ 2p.

Mặt khác với i ∈ {0, 1, ..., n− 1} thì(

2n2i + 1

)=

2n2i + 1

(2n− 1

2i

)... 2p. Vậy A = 2p.

Bài toán 5 (IMO Shortlist 2012). Tìm tất cả m ∈ N∗ để với n ∈ N∗ thoả mãnm3≤ n ≤ n

2

thì ta luôn có n là ước của(

nm− 2n

).

Lời giải. Ta chứng minh rằng m = p với p là số nguyên tố là tất cả các giá trị cần tìm.Với p = 2 thì ta có n = 1 và dễ dàng kiểm tra được trường hợp này thoả mãn.

Xét p ≥ 3. Khi đó với n ∈ N∗ mà n ≤ p− 12

thì ta luôn có được

(p− 2n)(

np− 2n

)= n

(n− 1

p− 2n− 1

)... n.

Lại có (p− 2n, n) = (p, n) = 1 nên(

np− 2n

)... n. Do đó m = p thoả mãn bài toán.

Ta chứng minh rằng với m là hợp số thì không thoả mãn bài toán. Xét các khả năng:

• Với m = 2k (k ∈ N∗) thì ta chọn n = k, ta có(

nm− 2n

)=

(n0

)= 1 không là

bội của n.• Với m là số lẻ thì m có ước nguyên tố p là số lẻ. Đặt m = p(2k + 1) (k ∈ N∗).

Chọn n = pk thoả mãn m3 ≤ n ≤ n

2 , ta có:

1n

(n

m− 2n

)=

1pk

(pkp

)=

(pk− 1)(pk− 2)...(pk− (p− 1))p!

.

Biểu thức trên không là số nguyên do tử thức không là bội của p.

Vậy tất cả các giá trị m thoả mãn bài toán là m = p với p là số nguyên tố.

Bài toán 6 (IMO Shortlist 2011). Cho p là số nguyên tố lẻ. Với mỗi a ∈ Z, ta đặt Sa =p−1

∑k=1

ak

k. Gọi m, n ∈ Z thoả mãn đẳng thức S3 + S4 − 3S2 =

mn

. Chứng minh rằng p là ước

của m.

Lời giải. Ta có

(−1)k−1

k=

1k(−1)k−1 ≡ 1

k

(p− 1k− 1

)=

1p

(pk

)(mod p).

54

Page 55: I HÅC QUÈC GIA TP. HCM · 2020. 10. 18. · Ngo i b i vi¸t cõa anh Ph¤m Tu§n Huy tø n«m 2013, c¡c b¤n s³ th§y r§t nhi·u b i vi¸t m t¡c gi£ v¨n cán l håc sinh cõa

Từ đó:

Sa =p−1

∑k=1

ak

k

= −p−1

∑k=1

(−a)k(−1)k−1

k≡ −1

p

p−1

∑k=1

(pk

)(−a)k

=−1p

(−1− (−a)p +

p

∑k=0

(−a)k(

pk

))≡ (a− 1)p − ap − 1

p(mod p).

Điều này dẫn đến

S4 +S3− 3S2 ≡3p − 4p − 1 + 2p − 3p − 1− 3(1− 2p − 1)

p=−(2p − 2)2

p≡ 0 (mod p),

trong đó đồng dư thức cuối cùng đúng theo định lý Fermat nhỏ.Vậy m là bội của p.

Bài toán 7 (IMO 1974). Chứng minh rằng f (n) =n

∑i=0

23i(

2n + 12i + 1

)không là bội của 5.

Lời giải. Ta lập hệ thức truy hồi cho f (n) bằng các biến đổi sau:

n

∑i=0

23i(

2n + 12i + 1

)=

n

∑i=0

√8

2i(

2n + 12i + 1

)

=1

2√

8· 2

n

∑i=0

√8

2i+1(

2n + 12i + 1

)

=1

2√

8

(2n+1

∑i=0

√8

i(

2n + 1i

)−

2n+1

∑i=0

(−√

8)i(

2n + 1i

))

=1

2√

8

((1 +√

8)2n+1

−(

1−√

8)2n+1

).

Từ đây ta được hệ thức truy hồi

{f (0) = 1. f (1) = 11

f (n + 2) = 18 f (n + 1)− 49 f (n) ∀n ∈ N.

Bằng tính tuần hoàn của dãy số dư ta dễ dàng có f (n) không là bội của 5, ∀n ∈ N.

Bài toán 8. Cho số nguyên tố p và n = nknk−1...n1n0(p) là biểu diễn trong cơ số p của n ∈

N∗. Chứng minh rằng trong các số(

n0

),(

n1

), ...(

nn

)có đúng (nk + 1)(nk−1 + 1)...(n0 + 1)

số không là bội của p.

55

Page 56: I HÅC QUÈC GIA TP. HCM · 2020. 10. 18. · Ngo i b i vi¸t cõa anh Ph¤m Tu§n Huy tø n«m 2013, c¡c b¤n s³ th§y r§t nhi·u b i vi¸t m t¡c gi£ v¨n cán l håc sinh cõa

Lời giải. Với s ∈ N và s ≤ n, đặt s = sksk−1...s1s0(p). Theo định lý Lucas, ta có:

(ns

)≡(

nksk

)(nk−1

sk−1

)...(

n0

s0

)(mod p).

Do đó(

ns

)không là bội của p khi và chỉ khi 0 ≤ si ≤ ni với 0 ≤ i ≤ k.

Từ đây ta nhận thấy rằng có ni + 1 cách chọn si. Hơn nữa các bước lựa chọn là độc lậpvới nhau nên có đúng (nk + 1)(nk−1 + 1)...(n0 + 1) số không là bội của p.

1.3. Về các bài toán đồng dư theo modulo

Bài toán 9. Chứng minh rằngp

∑k=2

k(

p− 2k− 2

)(p + 2

k

)≡ p + 2 (mod p3).

Lời giải. Ta sẽ chứng minh rằngp

∑k=2

k(

p− 2k− 2

)(p + 2

k

)= (p + 2)

(2p− 1p− 1

). Ta có:

p

∑k=2

k(

p− 2k− 2

)(p + 2

k

)=

p

∑k=2

k(

p− 2k− 2

)(p + 1k− 1

)p + 2

k

= (p + 2)p

∑k=2

(p− 2k− 2

)(p + 1k− 1

).

Do đó ta chỉ cần chứng minh rằngp−2

∑k=0

(p− 2

k

)(p + 1k + 1

)=

(2p− 1p− 1

). Thật vậy, ta có:

p−2

∑k=0

(p− 2

k

)(p + 1k + 1

)=

p−2

∑k=0

(p− 2

k

)(p + 1p− k

)=

(2p− 1p− 1

).

Theo định lý Wolstenholme thì(

2p− 1p− 1

)≡ 1 (mod p3) nên có kết luận bài toán.

Bài toán 10 (Putnam 1991). Cho p là số nguyên tố. Chứng minh rằng:

p

∑i=0

(pi

)(p + i

i

)≡ 2p + 1 (mod p2).

Lời giải. Ta có biến đổip

∑i=0

(pi

)(p + i

i

)=

p−1

∑i=1

(pi

)(p + i

i

)+ 1 +

(2pp

).

Theo định lý Babbage, ta có 1 +(

2pp

)≡ 1 +

(21

)= 3 (mod p2).

56

Page 57: I HÅC QUÈC GIA TP. HCM · 2020. 10. 18. · Ngo i b i vi¸t cõa anh Ph¤m Tu§n Huy tø n«m 2013, c¡c b¤n s³ th§y r§t nhi·u b i vi¸t m t¡c gi£ v¨n cán l håc sinh cõa

Do đó ta chỉ cần chứng minh:

p−1

∑i=1

(pi

)(p + i

i

)≡ 2p − 2 (mod p2).

Với i ∈ {1, 2, ..., p− 1}, ta có:

(p + i

i

)=

(p + 1)(p + 2)...(p + i)i!

=p2.A + p.i!

(1 + 1

2 + ... + 1i

)+ i!

i!(A ∈ Z)

=p2.A

i!+ p

(1 +

12+ ... +

1i

)+ 1

≡ p(

1 +12+ ... +

1i

)+ 1 (mod p2).

Do đó ta có được:

p−1

∑i=1

(pi

)(p + i

i

)≡

p−1

∑i=1

(pi

)(p(

1 +12+ ... +

1i

)+ 1)

≡p−1

∑i=1

p(

pi

)(1 +

12+ ... +

1i

)+

p−1

∑i=1

(pi

)

≡p−1

∑i=1

p2

i

(p− 1i− 1

)Bi!+ 2p − 2 ≡ 2p − 2 (mod p2) (B ∈ Z).

Vậy ta đã chứng minh đượcp

∑i=0

(pi

)(p + i

i

)≡ 2p + 1 (mod p2). Bài toán kết thúc.

Bài toán 11. Cho số nguyên tố p ≥ 5 và k =

⌊2p3

⌋. Chứng minh rằng:

k

∑i=0

(pi

)≡ 1 (mod p2).

Lời giải. Do p ≥ 5 nên ta có p = 6j + 1 hoặc p = 6j− 1 (j ∈ N).Ta chứng minh bài toán với p = 6j + 1, trường hợp còn lại làm tương tự. Ta có:

k

∑i=0

(pi

)=

k

∑i=0

(p− 1i− 1

)pi≡ p

k

∑i=0

(−1)i−1

i(mod p2).

57

Page 58: I HÅC QUÈC GIA TP. HCM · 2020. 10. 18. · Ngo i b i vi¸t cõa anh Ph¤m Tu§n Huy tø n«m 2013, c¡c b¤n s³ th§y r§t nhi·u b i vi¸t m t¡c gi£ v¨n cán l håc sinh cõa

Do đó ta chỉ cần chứng minhk

∑i=0

(−1)i−1

i≡ 0 (mod p). Với p = 6j + 1 thì k = 4j nên:

4j

∑i=0

(−1)i−1

i=

4j

∑i=2j+1

1i=

12

2j

∑i=1

(1

2j + i+

14j + 1− i

)

=12

2j

∑i=1

p(2j + i)(4j + i− 1)

.

Mặt khác lại có gcd(p, (2j + i)(4j + i− 1)) = 1 với i ∈ {1, 2, ..., 2j} nên:

12

2j

∑i=1

p(2j + i)(4j + i− 1)

≡ 0 (mod p).

Vậy ta kết luận đượck

∑i=0

(pi

)≡ 1 (mod p2) với k =

⌊2p3

⌋.

Bài toán 12 (Vietnam 2017). Chứng minh rằng:

1.1008

∑k=1

k(

2017k

)≡ 0

(mod 20172

).

2.504

∑k=1

(−1)k(

2017k

)≡ 3(22016 − 1)

(mod 20172

).

Lời giải. Trong lời giải sau, ta sẽ thay 2017 bởi số nguyên tố p ≡ 1 (mod 4).

1. Với 1 ≤ k ≤ p− 1, ta có k(

pk

)= p

(p− 1k− 1

)≡ p.(−1)k−1 (mod p2). Từ đó:

p−12

∑k=1

k(

pk

)≡ p

p−32

∑k=0

(−1)k ≡ 0 (mod p2).

2. Với k ∈ N và 1 ≤ k ≤ p− 12

, ta có biến đổi sau:

(−1)k(

pk

)= (−1)k.

pk

(p− 1k− 1

)≡ 2.(−1)2k−1 p

2k

≡ 2(

p− 12k− 1

)p

2k= 2

(p

2k

)(mod p2).

58

Page 59: I HÅC QUÈC GIA TP. HCM · 2020. 10. 18. · Ngo i b i vi¸t cõa anh Ph¤m Tu§n Huy tø n«m 2013, c¡c b¤n s³ th§y r§t nhi·u b i vi¸t m t¡c gi£ v¨n cán l håc sinh cõa

Do đó ta có được:

p−14

∑k=1

(−1)k(

pk

)≡

p−14

∑k=1

2(

p2k

)=

p−12

∑k=1

(pk

)+

p−12

∑k=1

(−1)k(

pk

)

≡p−1

2

∑k=1

(pk

)+

p−12

∑k=1

2(

p2k

)

≡p−1

2

∑k=1

(pk

)+

p−1

∑k=1

(pk

)+

p−1

∑k=1

(−1)k(

pk

)

≡ 2p−1 − 1 + 2p − 2 + 0 = 3(2p−1 − 1) (mod p2).

Bài toán kết thúc hoàn toàn.

Bài toán 13 (Vietnam TST 2010). Đặt Sn là tổng các bình phương hệ số trong khai triểncủa (1 + x)n. Chứng minh rằng S2n + 1 không là bội của 3.

Lời giải. Ta có Sn =n

∑k=0

(nk

)2

=

(2nn

). Do đó S2n =

(4n2n

).

Đặt cách viết trong hệ cơ số 3 là 4n = amam−1...a1a0(3) và 2n = bmbm−1...b1b0(3) với0 ≤ ai, bi ≤ 2 và i ∈ {0, 1, ..., m}.Theo định lý Lucas, ta có

(4n2n

)+ 1 ≡

m

∏i=0

(ai

bi

)+ 1 (mod 3). Xét các khả năng:

• Nếu tồn tại j để bj = 2 thì aj = 1 nên(

aj

bj

)=

(12

)= 0. Do đó:

(4n2n

)+ 1 ≡

m

∏i=0

(ai

bi

)+ 1 ≡ 1 (mod 3).

• Nếu ai ∈ {0, 1} (0 ≤ i ≤ m) thì trong các ai có 2k số 1 (2k ≤ m). Khi đó:

(4n2n

)+ 1 ≡ 1m−2k.

(21

)2k+ 1 ≡ 2 (mod 3).

Vậy trong mọi trường hợp, ta luôn có S2n + 1 không là bội của 3. Bài toán được chứngminh.

1.4. Một số bài toán khác

Bài toán 14 (Iran TST 2012). Tìm tất cả n ∈ N∗ sao cho với mọi 0 ≤ i, j ≤ n thì:(

ni

)+

(nj

)≡ i + j (mod 2).

59

Page 60: I HÅC QUÈC GIA TP. HCM · 2020. 10. 18. · Ngo i b i vi¸t cõa anh Ph¤m Tu§n Huy tø n«m 2013, c¡c b¤n s³ th§y r§t nhi·u b i vi¸t m t¡c gi£ v¨n cán l håc sinh cõa

Lời giải. Bài toán tương đương với tìm n ∈ N∗mà(

ni

)+ 1 ≡ i (mod 2) với 0 ≤ i ≤ n.

Đặt n = amam−1...a1a0(2) và i = bmbm−1...b1b0(2). Chọn i chẵn thì(

ni

)≡ 1 (mod 2).

Nếu tồn tại j ∈ N (1 ≤ j ≤ m) sao cho aj = 0, ta chọn i = 2j. Khi đó n− i là phép trừcó nhớ.

Theo định lý Kummer thì(

ni

)≡ 0 (mod 2), vô lý. Do đó phải có bj = 1 với j ∈

{1, 2, ..., m}.Chọn i lẻ, ta có (n

i ) ≡ 0 (mod 2) hay n− i phải là phép trừ có nhớ. Từ đó b0 = 0.Điều này dẫn đến n = 2m− 2 với m ∈ N∗. Thử lại, các số n trên thoả mãn bài toán.

Bài toán 15 (CGMO 2012). Có bao nhiêu số tự nhiên k ∈ {0, 1, ..., 2012} mà(

2012k

)là

bội của 2012 ?

Lời giải. Ta có 2012 = 40(503) nên theo định lý Kummer,(

2012k

)6 ... 503 khi và chỉ khi

k... 503. Ta sẽ đếm các số k để

(2012

k

)không là bội của 4.

Lại có 2012 = 11111011100(2) nên ta đếm k mà phép toán 2012− k có không quá 1 lầnnhớ. Xét các khả năng sau:

• Nếu phép trừ không nhớ: có 28 số k bằng việc ta thay bit 0 trong n bởi bit 1.• Nếu phép trừ có nhớ: có đúng 2 vị trí ở n để thay bit 0 bằng bit 1 và bit ở ngay

phía trước sẽ phải là bit 0; các bit 1 còn lại sẽ không bị ảnh hưởng nên ta có 2.27

số k.

Do đó có 2013 − 2.28 = 1501 số k để(

2012k

)là bội của 4. Hơn nữa

(2012503

),(

20121006

),(

20121539

)là bội của 4 nên tổng cộng có 1501− 3 = 1498 số thoả

mãn bài toán.

Bài toán 16 (Vietnam TST 1996). Với mỗi n ∈ N∗, ta đặt f (n) =

v2

b n−1

2 c∑k=0

3k(

n2k + 1

). Xác định tất cả các giá trị n để ta có f (n) = 1996.

60

Page 61: I HÅC QUÈC GIA TP. HCM · 2020. 10. 18. · Ngo i b i vi¸t cõa anh Ph¤m Tu§n Huy tø n«m 2013, c¡c b¤n s³ th§y r§t nhi·u b i vi¸t m t¡c gi£ v¨n cán l håc sinh cõa

Lời giải. Đặt an =b n−1

2 c∑k=0

3k(

n2k + 1

). Ta có biến đổi sau:

an =b n−1

2 c∑k=0

3k(

n2k + 1

)=

1√3

b n−12 c

∑k=0

(√

3)2k+1(

n2k + 1

)

=1

2√

3

(n

∑k=0

(√

3)k(

nk

)−

n

∑k=0

(−√

3)k(

nk

))

=(1 +

√3)n − (1−

√3)n

2√

3.

Xét các khả năng sau:

1. Nếu n = 2k, ta viết an = 2k.bk với bk =(2+√

3)k−(2−√

3)k

2√

3.

Dễ dàng có được hệ thức truy hồi sau:

{b1 = 1, b2 = 4

bk+2 = 4bk+1 − bk ∀k ∈ N∗

Từ đây bằng phép quy nạp nhận thấy bk lẻ khi và chỉ khi k lẻ. Đặt g(k) = v2(bk)

thì ta có:f (n) = f (2k) = k + g(k).

Đến đây lại có 2 khả năng sau xảy ra:

• Nếu k là số lẻ thì f (n) = f (2k) = k. Để f (n) = 1996 thì k = 1996 là sốchẵn, loại.• Nếu k là số chẵn, đặt k = 2s.l với s ≥ 1 và l lẻ thì ta dễ chứng minh được

điều sau: {g(2m) = g(m) + 1 nếu m là số chẵn

g(2m) = g(m) + 2 nếu m là số lẻ.

Từ đó ta có được f (n) = f (2k) = k + g(2s.l) = k + s + 1.

Do n = 2k = 2s+1.l và kết hợp với giả thiết, ta được 1996 = f (n) =

2s.l + s + 1.

Điều trên tương đương với 2s.l + s = 1995. Nhận xét rằng s phải lẻ và1 ≤ s ≤ 9.

Đến đây thử trực tiếp nhận n ∈ {3984, 3988}.

2. Nếu n = 2k + 1, ta có nhận xét rằng:

{a1 = 1, a2 = 2

an+2 = 2an+1 + 2an ∀n ∈ N∗.

Từ đó kết hợp với việc v2(a2k) ≥ k thì bằng phép quy nạp, ta được f (n) =

v2(a2k+1) = k.

Đến đây do f (n) = 1996 nên ta tìm được n = 3993.

61

Page 62: I HÅC QUÈC GIA TP. HCM · 2020. 10. 18. · Ngo i b i vi¸t cõa anh Ph¤m Tu§n Huy tø n«m 2013, c¡c b¤n s³ th§y r§t nhi·u b i vi¸t m t¡c gi£ v¨n cán l håc sinh cõa

Vậy các giá trị n thoả mãn bài toán là n ∈ {3984, 3993, 3988}.

Bài toán 17 (Vietnam TST 2017). Với mỗi số nguyên dương n, ta đặt xn =

(2nn

).

1. Chứng minh rằng nếu như2017k

2< n < 2017k với k ∈ N∗ thì xn

... 2017.

2. Tìm tất cả h ∈ N∗ để tồn tại các số nguyên dương N, T thoả mãn: ∀n > N thì (xn) làdãy số tuần hoàn theo modulo h với chu kỳ T.

Lời giải. Trong lời giải sau, ta sẽ thay 2017 bằng số nguyên tố p ≥ 3 bất kỳ.

1. Theo giả thiết thì pk−1 <pk

2< n < pk nên n = akak−1...a2a1(p) hay n có đúng k

chữ số trong hệ cơ số p. Giả sử rằng với mỗi i thoả mãn 1 ≤ i ≤ k, ta luôn cóai ≤ p−1

2 thì:

n =k

∑i=1

ai pi−1 ≤ p− 12

k

∑i=1

pi−1 =p− 1

2.pk − 1p− 1

=pk − 1

2<

pk

2.

Điều này trái với giả thiết nên tồn tại chỉ số i thoả mãn 1 ≤ i ≤ k mà ai >p− 1

2.

Do đó ai ≥p + 1

2nên phép cộng n + n trong hệ cơ số p là có nhớ khi ta cộng 2

giá trị ai. Vậy theo định lý Kummer thì xn là bội của p.2. Giả sử rằng số nguyên dương h thoả mãn bài toán có ước nguyên tố lẻ là q.

Ta chọn k đủ lớn sao chopk

2> 2T thì tồn tại T số hạng liên tiếp của xn nằm

trong

(pk

2, pk

)là bội của q hay tồn tại N để xn là bội của q với mọi n > N.

Ta chọn t > k sao cho pt−12 > N. Đến đây chọn n =

pt − 12

thì xn không phải

là bội của q. Nhưng điều này trái với chứng minh trên. Do đó h không có ướcnguyên tố lẻ hay h = 2r.

Giả sử rằng r > 1. Để ý rằng xn =(2n)!n!.n!

và vp(n!) =n− sp(n)

p− 1với sp(n) là tổng

các chữ số của n trong hệ cơ số p, ta có được

v2(xn) = 2n− s2(2n)− 2(n− s2(n)) = 2s2(n)− s2(2n)

Ta sẽ chọn n =r−1

∑i=1

2ai với 1 ≤ a1 < a2 < ... < ar−1 và 2a1 > max {N, T}. Bởi vì

cách chọn, dễ dàng thấy rằng v2(xn) = r− 1 nên xn ≡ 2r−1 (mod h).

Mặt khác do T < 2a1 nên tổng n + T trong hệ cơ số 2 có ít nhất thêm 1 bit 1.

62

Page 63: I HÅC QUÈC GIA TP. HCM · 2020. 10. 18. · Ngo i b i vi¸t cõa anh Ph¤m Tu§n Huy tø n«m 2013, c¡c b¤n s³ th§y r§t nhi·u b i vi¸t m t¡c gi£ v¨n cán l håc sinh cõa

Từ đây dễ dàng nhận thấy v2(xn+T) ≥ r hay xn+T ≡ 0 (mod h).

Nhưng do n > N và dãy tuần hoàn chu kỳ T nên xn ≡ xn+T ≡ 0 (mod h). Điềunày vô lý. Do đó r = 1 hay h = 2. Khi h = 2, dễ dàng kiểm tra rằng xn ≡ 0 (mod2) ∀n ∈ N∗ nên T = 1. Vậy h = 2 là số nguyên dương duy nhất thoả mãn.

Bài toán kết thúc.

2. BÀI TẬP

Bài tập 1. Chứng minh các đẳng thức sau:

1.n

∑i=0

1(n

i )=

n + 12n+1

n+1

∑i=1

2i

i.

2.n

∑i=0

i(

ni

)2

= n(

2n− 1n− 1

).

3.n

∑i=0

(−1)i

i + 1

(ni

)=

1n + 1

.

4.b n

2c∑i=0

((nk

)−(

nk− 1

))2

=1

n + 1

(2nn

).

Bài tập 2. Cho dãy số Fibonacci xác định như sau:

{F0 = F1 = 1

Fn+2 = Fn+1 + Fn ∀n ∈ N. Chứng

minh rằng ta luôn có đẳng thức Fn+1 =n

∑k=0

(⌊n+k2

k

).

Bài tập 3 (China TST 2017). Chứng minh rằng:

58

∑k=0

(2017 + k

58− k

)(2075− k

k

)=

29

∑k=0

(4091− 2k

58− 2k

)

Bài tập 4. Tìm tất cả n ∈ N và n ≥ 3 thoả mãn 22000 là bội của 1 +(

n1

)+

(n2

)+

(n3

).

Bài tập 5. Xác định các giá trị sau:

1. A = gcd((

nk

),(

n + 1k

), ...,

(n + k

k

)).

2. B = gcd((

n1

),(

n2

), ...,

(n

n− 1

)).

Bài tập 6. Chứng minh rằng với n ∈ N∗ thì lcm(1, 2, ..., 2n) là bội của(

2nn

).

Bài tập 7. Tồn tại hay không bộ k số (a1, a2, ..., ak) mà ai ∈ {0, 1} với i ∈ {1, 2, ..., k} mà:

63

Page 64: I HÅC QUÈC GIA TP. HCM · 2020. 10. 18. · Ngo i b i vi¸t cõa anh Ph¤m Tu§n Huy tø n«m 2013, c¡c b¤n s³ th§y r§t nhi·u b i vi¸t m t¡c gi£ v¨n cán l håc sinh cõa

1.k

∑i=0

3i.ai

(ki

)= 4k.

2.k

∑i=0

5i.ai

(ki

)= 6k−1.

Bài tập 8 (Romania TST 2004). Cho m, n ∈ N∗ và m là số lẻ. Chứng minh rằng:

m

∑k=0

(3m3k

)(3n− 1)k ... n.3m.

Bài tập 9 (IMO Shortlist 2007). Chứng minh rằng với k ∈ N và k ≥ 2 thì ta có:

v2

((2k+1

2k

)−(

2k

2k−1

))= 3k.

Bài tập 10 (China TST 2011). Cho trước số nguyên dương n. Gọi k là tổng các chữ số trong

biểu diễn nhị phân của n. Chứng minh rằngn

∑i=−n

(2n

n + i

)i2r ... 22n−k với r ∈ N∗.

Bài tập 11 (China 2015). Tìm tất cả k ∈ N sao cho tồn tại vô hạn n ∈ N để(

2nn

)6 ... n + k.

Bài tập 12. Cho p là số nguyên tố và 0 ≤ k ≤ n ≤ p− 1. Chứng minh rằng:(

nk

)≡ (−1)n+k

(p− k− 1p− n− 1

)(mod p).

Bài tập 13. Cho số nguyên tố p ≥ 5 và n, k ∈ N∗. Chứng mỉnh(

npkp

)≡(

nk

)(mod p3).

Bài tập 14. Cho số nguyên tố p ≥ 5, n ∈ N∗. Chứng minh(

2pn

pn

)≡(

2pn−1

pn−1

)(mod

p3n).

Bài tập 15 (Định lý Morley). Cho số nguyên tố p ≥ 5. Chứng minh rằng:

(−1)p−1

2

(p− 1p−1

2

)≡ 4p−1 (mod p3).

Bài tập 16. Cho p là số nguyên tố. Chứng minh rằng

p−12

∑k=0

3k(

pk

)≡ 2p − 1 (mod p2).

64

Page 65: I HÅC QUÈC GIA TP. HCM · 2020. 10. 18. · Ngo i b i vi¸t cõa anh Ph¤m Tu§n Huy tø n«m 2013, c¡c b¤n s³ th§y r§t nhi·u b i vi¸t m t¡c gi£ v¨n cán l håc sinh cõa

Bài tập 17 (USATST 2010). Liệu có tồn tại hay không số nguyên tố p = 6k + 1 thoả mãn(

3kk

)≡ 1 (mod p).

Bài tập 18. Cho số nguyên tố p ≥ 5. Chứng minh rằng(

p3

p2

)≡(

p2

p

)(mod p8).

Bài tập 19. Có bao nhiêu k ∈ {0, 1, , 2, ..., 2016} để(

2016k

)không là bội của 2016 ?.

Bài tập 20 (Trường xuân 2017). Cho dãy số (xn) xác định bởi công thức:

x0 = 1

xn = −2017 · x0 + x1 + ... + xn−1

n∀n ∈ N, n ≤ 2017

.

Hỏi có bao nhiêu chỉ số i ∈ {0, 1, 2, ..., 2017} sao cho |xi| là bội của 3 ?.

Bài tập 21. Cho p là số nguyên tố và p ≥ 5. Chứng minh rằng tồn tại a ∈ {1, 2, ..., p− 2}sao cho ap−1 − 1 và (a + 1)p−1 − 1 không là bội của p2.

Bài tập 22 (Saudi Arabia TST 2015). Với n ∈ N∗, ta đặt S(n) =n

∑i=0

ri trong đó ri là số dư

của(

ni

)khi chia cho 3. Tìm tất cả n để S(n) ≥ n.

Bài tập 23. Cho trước hằng số k ∈ N∗. Chứng minh rằng tồn tại N ∈ N∗ và chỉ xác định

phụ thuộc vào k sao cho ∀n ≥ N thì(

nk

)có ít nhất k ước nguyên tố phân biệt.

Bài tập 24. Chứng minh rằng không tồn tại n, k ∈ N∗ và n ≥ k+ 3 để các số(

nk

),(

nk + 1

),

(n

k + 2

),(

nk + 3

)lập thành cấp số cộng.

Bài tập 25. Cho tập hợp A =

{(nn

),(

2nn

), ...,

(n2

n

)}với n ∈ N∗ và n là hợp số.

1. Chứng minh rằng tồn tại vô hạn n để A là hệ thặng dư đầy đủ modulo n.2. Chứng minh rằng tồn tại vô hạn n để A không là hệ thặng dư đầy đủ modulo n.

Bài tập 26 (Bulgaria 2011). Tồn tại hay không n, k ∈ N và 1 ≤ k ≤ n− 2 thoả mãn

(nk

)2

+

(n

k + 1

)2

=

(n

k + 2

)4

.

65

Page 66: I HÅC QUÈC GIA TP. HCM · 2020. 10. 18. · Ngo i b i vi¸t cõa anh Ph¤m Tu§n Huy tø n«m 2013, c¡c b¤n s³ th§y r§t nhi·u b i vi¸t m t¡c gi£ v¨n cán l håc sinh cõa

Bài tập 27. Cho p là số nguyên tố và dãy vô hạn các số a0, a1... thoả mãnn

∑i=0

pi(

ni

)ai = 0

với vô số giá trị của n. Chứng minh rằng a0 = a1 = ... = 0.

Bài tập 28 (Romania 2010). Cho dãy số thực dương (an) thoả mãn:

n

∑i=0

(nk

)akan−k = a2

n ∀n ∈ N.

Chứng minh rằng (an) là một cấp số nhân.

Bài tập 29 (IMO Shortlist 2008). Cho n ∈ N∗. Chứng minh rằng các số dư của:

A =

{(2n − 1

1

),(

2n − 13

), ...,

(2n − 12n − 1

)}

khi chia cho 2n là toàn bộ các phần tử của {1, 3, ..., 2n − 1}.

Bài tập 30 (China TST 2012). Với n ∈ N∗, đặt xn =

(2nn

). Chứng minh rằng tồn tại vô

hạn các tập hợp hữu hạn A, B 6= ∅ và A, B ⊂ N∗ thoả mãn:

∏i∈A

xi

∏j∈B

xj= 2012.

TÀI LIỆU THAM KHẢO1. Trần Nam Dũng (chủ biên), Các phương pháp giải toán qua các kỳ thi Olympic, 2014.2. Trần Nam Dũng (chủ biên), Các phương pháp giải toán qua các kỳ thi Olympic, 2015.3. Đề bài và lời giải Việt Nam TST 1989-2004.4. Community - Art of Problem Solving: https://artofproblemsolving.com/community5. Diễn đàn toán học Việt Nam - VMF: https://diendantoanhoc.net

66

Page 67: I HÅC QUÈC GIA TP. HCM · 2020. 10. 18. · Ngo i b i vi¸t cõa anh Ph¤m Tu§n Huy tø n«m 2013, c¡c b¤n s³ th§y r§t nhi·u b i vi¸t m t¡c gi£ v¨n cán l håc sinh cõa

MỘT SỐ ĐỊNH HƯỚNG TIẾP CẬN LỜI GIẢIĐỐI VỚI CÁC BÀI TOÁN OLYMPIC

Nguyễn Tiến Hoàng

Lớp chuyên Toán 2016 - 2019

1. GIỚI THIỆU

Theo quan sát, trong vài năm gần đây, các bài toán khó của kỳ thi HSG Quốc gia(VMO) và chọn đội tuyển tham dự Olympic Toán Quốc tế (Việt Nam TST) đều có mốiliên hệ đến các định lý hoặc dựa trên những bài toán đã có sẵn. Chẳng hạn như bàitoán số 2 của Việt Nam TST 2017:

Bài toán 1 (Việt Nam TST 2017). Với mỗi số nguyên dương n, đặt xn =

(2nn

).

1. Chứng minh rằng nếu tồn tại k ∈ N∗ để2017k

2< n < 2017k thì xn là bội của 2017.

2. Tìm tất cả số nguyên dương h > 1 để tồn tại các số nguyên dương N, T sao cho∀n > N thì {xn} là dãy số tuần hoàn theo modulo h với chu kỳ T.

Đây là một bài toán có sự liên quan mật thiết đến các định lý liên quan đến tínhchất số học của hệ số nhị thức như Legendre hay Babbage. Ví dụ gần đây nhất là bàitoán số 6 của VMO 2018 với tính chất của dãy số Lucas:

Bài toán 2 (Việt Nam 2018). Cho {xn} là dãy số xác định bởi

{x0 = 2, x1 = 1

xn+2 = xn+1 + xn ∀n ∈ N.

1. Với n > 1 là số nguyên dương, chứng minh rằng nếu xn là số nguyên tố thì n là sốnguyên tố hoặc n không có ước nguyên tố lẻ.

2. Tìm tất cả các cặp số nguyên dương (m, n) để xn là bội của xm.

Hai bài toán trên là các ví dụ điển hình cho điều sau: nếu học sinh thiếu kinhnghiệm về những bài toán thuộc về chủ đề liên quan, việc tìm ra lời giải trong điềukiện thời gian hạn chế là một điều rất khó xảy ra. Từ đó dẫn đến câu hỏi: Liệu có thể

67

Page 68: I HÅC QUÈC GIA TP. HCM · 2020. 10. 18. · Ngo i b i vi¸t cõa anh Ph¤m Tu§n Huy tø n«m 2013, c¡c b¤n s³ th§y r§t nhi·u b i vi¸t m t¡c gi£ v¨n cán l håc sinh cõa

giải được trong thời gian ngắn những bài toán ở mức độ trên theo một cách "ngây thơ" và tựnhiên nhất, với việc biết rất ít (hoặc hoàn toàn không biết) các vấn đề liên quan ?

Với tinh thần của câu hỏi trên, bài viết này sẽ quan tâm chủ yếu đến những bàitoán "phi truyền thống", nghĩa là không quá liên quan đến một kết quả nhất địnhnào, đặc biệt là các kết quả khó tiếp cận với số đông. Các bài toán ấy sẽ được tiếp cậntheo góc nhìn tự nhiên nhất có thể, và hơn nữa, lời giải chỉ dựa trên sự quan sát kỹcàng vấn đề, kết hợp với những tư duy quen thuộc.

2. Những định hướng tiếp cận điển hình

2.1. Quy nạp và thác triển từ trường hợp riêng

Nguyên lý quy nạp là một chủ đề quen thuộc với các bài toán Olympic, xin khôngnhắc lại các bước thực hiện cụ thể tại đây. Về bản chất, đây là một cách tiếp cận theohướng xây dựng: nếu mệnh đề A(n) đúng thì các mệnh đề A( f1(n)), A( f2(n)), ...cũng sẽ đúng, với f1, f2, ... là các hàm số theo biến n, có tập xác định và tập giá trị là Z.Sở dĩ lấy tập hợp Z vì trong một số tình huống, ta thực hiện "quy nạp lùi", chẳng hạnnhư giải phương trình hàm Cauchy trên Z. Trong phép quy nạp thông thường thìf1(n) = n + 1. Với phép quy nạp kiểu Cauchy thì ta chọn f1(n) = 2n, f2(n) = n− 1.Và còn nhiều kiểu quy nạp khác nữa tuỳ tình huống cụ thể.

Quy nạp có mối liên hệ mật thiết với thứ tự của các số nguyên. Do tính "rời rạc"của tập hợp số nguyên, ta có thể bắt đầu từ một giá trị khởi tạo nào đó để tiến hànhquy nạp mà yên tâm rằng có thể đi qua được hết các giá trị của yêu cầu bài toán. Từđó dẫn đến hai ý tưởng sau:

• Nên tiến hành quy nạp với các bài toán liên quan đến số nguyên mà việc giảitrực tiếp bài toán gặp nhiều khó khăn (biến đổi đại số cồng kềnh, vấn đề trừutượng,...)• Trong trường hợp bài toán không có dấu hiệu nào rõ ràng, hãy sử dụng ý tưởng

học được của phép quy nạp: giải quyết trường hợp cơ sở, quan sát, sắp xếp cácđại lượng để làm gọn vấn đề, thậm chí là đoán nhận quy luật tổng quát.

Ta bắt đầu với bài toán sau:

Bài toán 3. Chứng minh rằng với n > 3, luôn tồn tại hoán vị (a1, a2, ..., an) của (1, 2, ..., n)sao cho ai + ak 6= 2aj với mọi i, j, k mà 1 ≤ i < j < k ≤ n.

Hướng dẫn. Ta bắt đầu từ những trường hợp nhỏ, chẳng hạn như n = 4, n = 5.Với n = 4, ta có thể có hoán vị (4, 2, 1, 3). Với n = 5 thì ta có một hoán vị là (5, 1, 3, 2, 4).Để ý một chút, hai hoán vị này có gì lạ ? Các số lẻ "nằm ở một bên" và các số chẵn ởbên còn lại. Một hoán vị với đặc điểm này sẽ thoả mãn được điều kiện ai + ak 6= 2aj

nếu ai, ak được lấy ở "hai bên" của hoán vị, vì tổng nhận được luôn là số lẻ trong khi2aj là số chẵn.

68

Page 69: I HÅC QUÈC GIA TP. HCM · 2020. 10. 18. · Ngo i b i vi¸t cõa anh Ph¤m Tu§n Huy tø n«m 2013, c¡c b¤n s³ th§y r§t nhi·u b i vi¸t m t¡c gi£ v¨n cán l håc sinh cõa

Vậy ta chỉ còn phải kiểm tra tính chất của bài toán đối với mỗi bên của hoán vị, mà tasẽ gọi mỗi bên là một hoán vị con. Một nhận xét như sau:

Nhận xét. Nếu (b1, b2, ..., bk) là một hoán vị k phần tử thoả mãn yêu cầu bài toán thì(2b1, 2b2, ..., 2bk) và (2b1 − 1, 2b2 − 1, ..., 2bk − 1) cũng là các hoán vị thoả mãn.

Đây là nhận xét dễ kiểm chứng. Tại sao lại nghĩ đến điều này ? Vì trong mỗi hoán vịcon đều chứa toàn bộ phần tử là số chẵn hoặc số lẻ. Cần có xu hướng giảm độ lớn củacác phần tử trong hoán vị, nên với phần tử chẵn thì ta chia đôi, còn phần tử lẻ thì biếnđổi thành chẵn rồi chia đôi. Từ đó dễ dàng dẫn ra lại được nhận xét trên.Dễ thấy rằng ý tưởng giảm số phần tử như trên chính là một kiểu quy nạp lùi. Hơn nữanếu có hoán vị thoả mãn đề bài của bộ (1, 2, ..., n) thì cũng có hoán vị thoả mãn vớibộ (1, 2, ..., n− 1) như sau: ta chỉ cần loại n là xong. Gọi A(n) là mệnh đề cần chứngminh thì ta có các bước sau:

• Chứng minh rằng A(k) đúng, với k nào đó chọn trước.• Tạo ra hoán vị thoả mãn của (2, 4, ..., 2k) và (1, 3, ..., 2k− 1).• Ghép hoán vị và lần lượt bỏ đi phần tử lớn nhất trong mỗi thao tác.

Việc trình bày lại những phân tích trên thành một lời giải hoàn chỉnh xin dành chobạn đọc.

Bài toán 4. Cho lưới nguyên 100× 100. Cần ít nhất bao nhiêu đường thẳng để mỗi điểmcủa lưới nguyên trên đều nằm trên ít nhất 1 đường thẳng, trừ điểm ở góc dưới cùng bên trái ?

Lời giải. Ta chứng minh bằng quy nạp rằng với lưới nguyên m× n (m, n ∈ N∗) nhưđiều kiện bài toán, số đường thẳng ít nhất là m + n− 2. Thật vậy, trước hết là có cấuhình cho m + n− 2 đường thẳng với m− 1 đường nằm ngang và n− 1 đường nằmdọc.Dĩ nhiên với m = 1, số đường thẳng ít nhất là n − 1 vì không thể có đường nằmngang. Giả sử với m, n > 1, bài toán đúng với mọi lưới nguyên có kích thước nhỏ hơnm× n.

• Nếu có một đường nằm ngang (hoặc nằm dọc), ta loại bỏ đường đó đi để thuđược một lưới nhỏ hơn. Theo giả thiết quy nạp, số đường thẳng cần sử dụng ítnhất cho lưới nhỏ hơn mà ta vừa xây dựng là m + n− 1− 2 đường thẳng, nênkết hợp với đường thẳng mà ta vừa loại thì cần ít nhất m + n− 2 đường thẳng.• Ngược lại, xét 2m + 2n− 5 điểm ở ngoài cùng của lưới nguyên (do đã loại bỏ

điểm ở góc dưới cùng bên trái), mà ta sẽ gọi là "đường bao". Một đường thẳngbất kỳ chỉ cắt tối đa 2 điểm ở đường bao, nên số đường thẳng tối thiểu sẽ là:

⌊2m + 2n− 5

2

⌋+ 1 = m + n− 2.

69

Page 70: I HÅC QUÈC GIA TP. HCM · 2020. 10. 18. · Ngo i b i vi¸t cõa anh Ph¤m Tu§n Huy tø n«m 2013, c¡c b¤n s³ th§y r§t nhi·u b i vi¸t m t¡c gi£ v¨n cán l håc sinh cõa

Theo nguyên lý quy nạp, với m, n ∈ N∗ thì số đường thẳng cần sử dụng ít nhất cholưới nguyên m × n là m + n − 2. Áp dụng với m = n = 100 thì ta cần ít nhất 198đường thẳng.

Đôi khi việc thực hiện quy nạp phải thông qua nhiều biến đổi đại số như hai bàitoán sau:

Bài toán 5. Cho số nguyên dương k. Xét hàm số f : N∗ → N∗ được xác định như sau:

{f (n) = 1 ∀n ≤ k + 1

f (n) = f ( f (n− 1)) + f (n− f (n− 1))

với mọi n > k + 1. Chứng minh rằng ∀n ∈ N∗, { f−1(n)} là tập hữu hạn chỉ chứa các sốnguyên dương liên tiếp.

Lời giải. Trước hết ta chứng minh rằng f (n)− f (n− 1) ∈ {0; 1} ∀n ∈ N∗.Giả sử rằng điều này đúng với mọi số nguyên dương nhỏ hơn n + 1 với n ≥ k. Chú ýrằng điều này dẫn đến f (m) < m với 2 ≤ m ≤ n. Có hai khả năng sau:

• Nếu f (n)− f (n− 1) = 0 thì ta có:

f (n + 1)− f (n) = f ( f (n)) + f (n + 1− f (n))− f ( f (n− 1))− f (n− f (n− 1))

Viết gọn lại thì f (n + 1)− f (n) = f (n− f (n) + 1)− f (n− f (n)) ∈ {0; 1}.• Nếu f (n)− f (n− 1) = 1 thì biến đổi tương tự, ta sẽ có được:

f (n + 1)− f (n) = f ( f (n− 1) + 1)− f ( f (n− 1)) ∈ {0; 1}

Theo nguyên lý quy nạp thì f (n)− f (n− 1) ∈ {0; 1} ∀n ∈ N∗. Từ đó f (n) lập thànhmột dãy không giảm các số tự nhiên "gần liên tiếp". Bây giờ ta chỉ cần chứng minh fkhông bị chặn trên.Thật vậy, giả sử rằng tồn tại a, b ∈ N∗ để f (n) = b ∀n ≥ a ≥ k + 1. Thay n = a + b thì:

f (a+ b) = f ( f (a+ b− 1))+ f (a+ b− f (a+ b+ 1)) = f (b)+ f (a+ b− b) = f (b)+ b > b

Đây là điều vô lý. Từ đó f không bị chặn trên. Phép chứng minh kết thúc.

Bài toán 6 (Trường xuân miền Nam 2016). Chứng minh rằng với mỗi n ∈ N∗, n ≥ 2, tacó thể phân hoạch 2n số tự nhiên đầu tiên thành các tập hợp An, Bn để điều sau được thoảmãn:

∑x∈An

xk = ∑x∈Bn

xk (k ∈ N, k ≤ n− 1)

70

Page 71: I HÅC QUÈC GIA TP. HCM · 2020. 10. 18. · Ngo i b i vi¸t cõa anh Ph¤m Tu§n Huy tø n«m 2013, c¡c b¤n s³ th§y r§t nhi·u b i vi¸t m t¡c gi£ v¨n cán l håc sinh cõa

Lời giải. Rõ ràng tập hợp 2n số tự nhiên đầu tiên chính là Sn = {0; 1, ...; 2n − 1}. Taphân hoạch tập hợp này thành 2 phần là An, Bn sao cho tại An, tổng các chữ số khiviết trong hệ cơ số 2 của mỗi phần tử là số chẵn, còn ở Bn là phần còn lại.Dễ thấy rằng mỗi phần tử của Sn sẽ tương ứng được với một dãy nhị phân có khôngquá n bit. Hơn nữa, tồn tại một song ánh từ An vào Bn bằng cách thay đổi chữ số cuốicùng trong hệ cơ số 2 của mỗi phần tử của An: bit 0 thành bit 1 và ngược lại. Do đó|An| = |Bn| = 2n−1.Ta chứng minh bằng quy nạp theo n rằng cách phân hoạch này thoả mãn yêu cầubài toán. Thật vậy, dễ kiểm tra điều này với n = 2. Giả sử bài toán đã đúng vớin = l (l ≥ 2). Ta chứng minh rằng bài toán cũng đúng với k = l + 1.Để thuận tiện về mặt ký hiệu, ta quy ước với A là một tập hợp số thì A + c =

{a + c | a ∈ A} và cA = {ca | a ∈ A}. Ta nhận thấy được hai điều sau:

{Al+1 = 2Al ∪ (Bl + 2l)

Bl+1 = 2Bl ∪ (Al + 2l)

Từ đây với k ≤ l và 0 ≤ i ≤ k, ta có được:

∑x∈Al+1

xk = ∑x∈Al

(2x)k + ∑x∈Bl

(x + 2l)k = 2k ∑x∈Al

xk + ∑x∈Bl

(ki

)2lixk−i

Viết lại thành ∑x∈Al+1

xk = 2k ∑x∈Al

xk +k

∑i=0

((ki

)2li ∑

x∈Bl

xk−i

). Tương tự, ta có:

∑x∈Bl+1

xk = 2k ∑x∈Bl

xk +k

∑i=0

((ki

)2ni ∑

x∈Al

xk−i

)

Do giả thiết quy nạp mà với k ≤ l − 1 thì ∑x∈Al

xk = ∑x∈Bl

xk. Điều này cũng đúng khi ta

nhân thêm hằng số vào các tổng, nên dẫn đến việc ∑x∈Al+1

xk = ∑x∈Bl+1

xk với k ≤ l − 1.

Chỉ còn trường hợp k = l. Viết lại rõ hơn các đẳng thức trên:

2l ∑x∈Al

xl +l

∑i=0

((li

)2li ∑

x∈Bl

xl−i

)= 2l ∑

x∈Al

xl + 2l ∑x∈Bl

xl +l

∑i=1

((li

)2ni ∑

x∈Bl

xl−i

)

2l ∑x∈Bl

xl +l

∑i=0

((li

)2li ∑

x∈Al

xl−i

)= 2l ∑

x∈Bl

xl + 2l ∑x∈Al

xl +l

∑i=1

((li

)2ni ∑

x∈Bl

xl−i

)

Rõ ràng biểu thức sau cùng nhận được của cả hai đẳng thức là bằng nhau. Từ đó:

∑x∈Al+1

xl = ∑x∈Bl+1

xl

71

Page 72: I HÅC QUÈC GIA TP. HCM · 2020. 10. 18. · Ngo i b i vi¸t cõa anh Ph¤m Tu§n Huy tø n«m 2013, c¡c b¤n s³ th§y r§t nhi·u b i vi¸t m t¡c gi£ v¨n cán l håc sinh cõa

Bài toán đúng với n = l + 1. Theo nguyên lý quy nạp thì bài toán đúng với mọin ∈ N∗.

Để kết thúc, ta xem xét một bài toán từ tạp chí Pi số tháng 1/2018.

Bài toán 7 (Tạp chí Pi, P137). Với mỗi n ∈ N∗, gọi Qn là tập hợp tất cả các số hữu tỉ códạng

mn

với m ∈ Z. Tìm tất cả các hàm số f : Q→ R và tăng ngặt thoả mãn:

∀n ∈ N∗, ∀x, y ∈ Qn, ∃z ∈ Qn : f (z) = f (x)− f (y)

Lời giải. Do f tăng ngặt nên f là đơn ánh. Để ý rằng Q1 = Z ⊂ Qn ∀n ∈ N∗.Theo giả thiết, tồn tại t ∈ Z để f (z) = f (x)− f (x) = 0 với x ∈ Z. Do f tăng ngặt nênta có f (t + 1) > 0, lại nhận thấy nếu f là một hàm số thoả mãn bài toán thì k · f cũngthế (k ∈ R+) nên không mất tính tổng quát, giả sử rằng f (t + 1) = 1.Cũng từ f tăng ngặt mà f (x) > 0 ∀x > t, x ∈ Q. Theo giả thiết, tồn tại t1 ∈ Z sao cho

f (t1) = f (t + 2)− f (t + 1) > 0 = f (t)

Dễ dàng nhận thấy các bất đẳng thức sau:

{f (t + 2) > f (t + 2)− f (t + 1) > 0 = f (t)

f (t + 2) > f (t + 1) > f (t)

Do đó mà f (t + 2)− f (t + 1) = f (t + 1) hay f (t + 2) = 2 f (t + 1) = 2.Bây giờ ta chứng minh bằng quy nạp rằng f (t + n) = n ∀n ∈ N∗. Giả sử rằng điềunày đã đúng đến n = k (k ≥ 2). Theo giả thiết, tồn tại z1, z2, ..., zk ∈ Z sao cho:

f (zi) = f (t + k + 1)− f (t + i) ∀i ∈ {1; 2; ...; k}

Ta có hai dãy bất đẳng thức sau:

{f (t + k + 1) > f (z1) > f (z2) > ... > f (zk) > f (t)

f (t + k + 1) > f (t + k) > f (t + k− 1) > ... > f (t + 1) > f (t)

Do đó mà f (z1) = f (t + k) hay f (t + k + 1)− f (t + 1) = f (t + k), dẫn đến f (t + k +1) = k + 1.Theo nguyên lý quy nạp thì f (t + n) = n ∀n ∈ N∗. Để ý rằng f (t− 1) < f (t) = 0.Bằng việc thiết lập các dãy bất đẳng thức theo chiều ngược lại và quy nạp tương tự,ta có được:

f (t− n) = n f (t− 1) ∀n ∈ N∗

Bây giờ ta xác định f (t− 1). Tồn tại z ∈ Z sao cho:

f (z) = f (t + 1)− f (t− 1) = 1− f (t− 1) > 1

72

Page 73: I HÅC QUÈC GIA TP. HCM · 2020. 10. 18. · Ngo i b i vi¸t cõa anh Ph¤m Tu§n Huy tø n«m 2013, c¡c b¤n s³ th§y r§t nhi·u b i vi¸t m t¡c gi£ v¨n cán l håc sinh cõa

Do đó f (z) ∈ Z, từ đó thì f (t− 1) ∈ Z. Tồn tại l ∈ N∗ sao cho:

l f (t− 1) = f (t− l) = f (t− 1)− f (t + 1) < 0

Từ đây f (t + 1) là bội của f (t− 1), mà f (t− 1) < 0 nên f (t− 1) = −1.

Lập các dãy tương tự, ta sẽ có f (t + 1) = n f(

t +1n

)hay f

(t +

1n

)=

1n∀n ∈ N∗.

Bây giờ lại tiếp tục quy nạp với mỗi n ∈ N∗ thì f (x) = x− t ∀x ∈ Q.Thử lại thì hàm số này thoả mãn. Bởi nhận xét về việc sai khác một hằng số ở phépnhân, tất cả các hàm số cần tìm là f (x) = k(x− t) ∀x ∈ Q (k ∈ R+, t ∈ Z).

2.2. Điều chỉnh địa phương

Điều chỉnh địa phương, đúng như tên gọi, có thể được hiểu rộng là việc xử lýnhững điểm nhỏ, lẻ tẻ mà ta gặp phải trong quá trình giải toán. Đó có thể là việcchứng minh một kết quả bất ngờ phát sinh, hay là cố gắng đưa bài toán về một trườnghợp "gần tốt" nhất có thể - chẳng hạn như việc tìm cực trị của các biểu thức bậc haimà ta vẫn gặp trong các bài toán đếm bằng hai cách.

Nguyên tắc nằm trong việc "sai ở đâu, sửa ở đó". Một ví dụ rất tốt là bài toán 5 ởmục trên. Khi ta thực hiện bước quy nạp, việc giải quyết rẽ nhánh để cuối cùng tất cảcác trường hợp đều thuận lợi chính là minh hoạ của xử lý dữ kiện ở các bước nhỏ đểkết quả tốt nhất.

Ta bắt đầu với các bài toán cực trị.

Bài toán 8. Cho 100 số nguyên dương a1, a2, ..., a100 có tổng bằng 2017. Tìm giá trị nhỏnhất:

A = a21 + a2

2 + ... + a2100.

Lời giải. Không mất tính tổng quát, giả sử a1 ≤ a2 ≤ ... ≤ a100.Ta sẽ chứng minh rằng, để A đạt giá trị nhỏ nhất thì a100 − a1 ≤ 1. Thật vậy, gọi(a1, a2, ..., a100) là bộ số mà a100 − a1 ≥ 2 để A = a2

1 + a22 + ... + a2

100 đạt giá trị nhỏnhất thì xét bộ số sau:

(b1, b2, ..., b100) = (a1 + 1, a2, ..., a100 − 1)

Khi đó thì b1 + b2 + ... + b100 = 2017. Với B = b21 + b2

2 + ... + b2100 thì:

B− A = (a100 − 1)2 + (a1 + 1)2 − a2100 − a2

1 = 2− 2(a100 − a1) < 0

Điều này vô lý với tính nhỏ nhất, nên điều giả sử là sai. Do đó bộ số làm cho A nhỏnhất phải thoả mãn a100 − a1 ≤ 1. Đến đây, kết hợp với việc tổng của 100 số là 2017để có được 20 < a100 < 22, ta sẽ dẫn đến:

A ≥ 83 · 202 + 17 · 212 = 40697

73

Page 74: I HÅC QUÈC GIA TP. HCM · 2020. 10. 18. · Ngo i b i vi¸t cõa anh Ph¤m Tu§n Huy tø n«m 2013, c¡c b¤n s³ th§y r§t nhi·u b i vi¸t m t¡c gi£ v¨n cán l håc sinh cõa

Đẳng thức xảy ra khi a1 = a2 = ... = a83 = 20 và a84 = a85 = ... = a100 = 21.

Bài toán 9. Cho n ≥ 3 là hằng số. Tìm số nguyên dương k nhỏ nhất sao cho với mọi đồ thịG có n đỉnh và ít nhất k cạnh thì G là đồ thị liên thông.

Lời giải. Đặt r =(

n− 12

)+ 1. Ta sẽ chứng minh đây là giá trị k cần tìm.

Trước hết, tồn tại một đồ thị có n đỉnh và r− 1 cạnh mà không liên thông. Thật vậy, taxét G là hợp của một đồ thị đầy đủ có n− 1 đỉnh với một đỉnh treo. Dĩ nhiên G khôngliên thông, từ đó ta phải có k ≥ r. Bây giờ ta chứng minh mọi đồ thị có n đỉnh và ítnhất r cạnh đều liên thông.Giả sử rằng tồn tại một đồ thị n đỉnh và ít nhất r cạnh mà không liên thông. Gọi V làtập hợp các đỉnh, E là tập hợp các cạnh và deg(v) là bậc của đỉnh v ∈ V. Kết quả saulà quen thuộc:

∑v∈V

deg(v) = 2r

Do đó với v ∈ V bất kỳ thì:

max deg(v) ≥ 2rn

=(n− 1)(n− 2) + 1

n> n− 3

Gọi đỉnh có bậc lớn nhất là u thì có các khả năng sau xảy ra:

• Nếu deg(u) = n− 1 thì G liên thông, trái với điều giả sử.• Nếu deg(u) = n− 2, gọi w là đỉnh không kề với u. Rõ ràng tất cả các đỉnh còn

lại của G đều kề với u, và nếu w kề với bất kỳ một đỉnh nào khác thì G sẽ liênthông, trái với điều giả sử. Từ đó w phải là một đỉnh treo, dẫn đến số cạnh tốiđa của G là:

r =(

n− 12

)+ 1 ≤ |E| ≤

(n− 1

2

)

Đây cũng là một điều vô lý.

Tóm lại điều giả sử là sai hay G phải liên thông. Vậy k = r =

(n− 1

2

)là giá trị cần

tìm.

Hai ví dụ tiếp theo sẽ lạ mắt hơn khi các thao tác dựa trên lập luận số học:

Bài toán 10. Với mỗi số nguyên dương m, đặt S(m) là tổng các chữ số của m khi viết tronghệ thập phân. Tìm tất cả n ∈ N∗ để với mỗi k ≥ n, tồn tại bội số m của n mà S(m) = k.

Lời giải. Với mỗi k ≥ n, đặt m = f (k) · n. Ta có:

k = S(m) ≡ m ≡ f (k) · n (mod 9)

Do k nhận mọi giá trị lớn tuỳ ý nên k sẽ đi qua một hệ thặng dư đầy đủ modulo 9. Từđây thấy ngay rằng (n, 9) = 1 hay (n, 3) = 1 chính là điều kiện cần.

74

Page 75: I HÅC QUÈC GIA TP. HCM · 2020. 10. 18. · Ngo i b i vi¸t cõa anh Ph¤m Tu§n Huy tø n«m 2013, c¡c b¤n s³ th§y r§t nhi·u b i vi¸t m t¡c gi£ v¨n cán l håc sinh cõa

Ta sẽ chứng minh rằng đây cũng là điều kiện đủ bằng cách xây dựng m.Trước hết ta giải bài toán với (n, 10) = 1. Theo định lý Euler, ta có:

10sϕ(n) ≡ 1 (mod n) ∀s ∈ N∗

Từ đây dễ có được 10sϕ(n)+1 ≡ 10 (mod n) ∀s ∈ N∗.Ta sẽ xây dựng m có toàn các chữ số 1 sao cho S(m) = k. Nghĩa là ta chọn ra k số hạngphân biệt gồm các dạng 10sϕ(n), 10sϕ(n)+1 để cộng lại với nhau, tạo thành m. Do s cóthể lấy tuỳ ý trên N∗ nên tính phân biệt là hiển nhiên. Ta chỉ cần phải kiểm tra điềukiện chia hết.Gọi số các số hạng mỗi loại lần lượt là a, b ≥ 0 thì ta có hệ phương trình sau:

{a + b = k ≥ n

a + 10b ≡ 0 (mod n)

Nếu k là bội của n thì chọn a = k, b = 0. Ngược lại, ta có 9b ≡ −k (mod n) và(n, 9) = 1 nên b là nghịch đảo của 9 theo modulo n. Dĩ nhiên chọn được b ≤ n nêntồn tại a ≥ 0 thoả mãn.Từ đó ta đã xây dựng được m để S(m) = k và m là bội của n với (n, 10) = 1. Nếu(n, 10) > 1, viết n = 2α · 5β · p với (p, 10) = 1. Xây dựng m thoả mãn với p và cóS(m) = k ≥ n ≥ p, rồi chọn m′ = 10α+βm thì số m′ có S(m′) = S(m) = k ≥ n và m′

là bội của n.Vậy tất cả các số nguyên dương n thoả mãn bài toán là (n, 3) = 1. Bài toán kếtthúc.

Bài toán 11. Trên mặt phẳng toạ độ có một đa giác lồi n đỉnh (n > 2) và toạ độ các đỉnh đềulà số hữu tỉ. Giả sử rằng đa giác có tất cả các cạnh bằng nhau. Chứng minh rằng n là số chẵn.

Lời giải. Đầu tiên, xét bài toán với trường hợp các đỉnh có toạ độ nguyên. Gọi mộtđỉnh nào đó là A1(x1, y1) và gọi các đỉnh lần lượt là A2, A3, ..., An theo chiều kim đồnghồ. Bởi độ dài các cạnh bằng nhau, mà ta sẽ tạm gọi là ` > 0, theo công thức tínhkhoảng cách thì:

`2 = (x1− x2)2 +(y1− y2)

2 = ... = (xn−1− xn)2 +(yn−1− yn)

2 = (xn− x1)2 +(yn− y1)

2

Dĩ nhiên `2 là một số nguyên và là tổng của hai số chính phương.Xét đẳng thức trên theo modulo 4. Quy ước xn+1 = x1, yn+1 = y1. Có 3 khả năng sauxảy ra:

• Nếu `2 ≡ 2 (mod 4) thì ta có xi − xi+1 ≡ yi − yi+1 ≡ 1 (mod 2) (1 ≤ i ≤ n). Từđó:

x1 ≡ x2 + 1 ≡ ... ≡ x1 + n (mod 2)

Điều này dẫn đến n là số chẵn.

75

Page 76: I HÅC QUÈC GIA TP. HCM · 2020. 10. 18. · Ngo i b i vi¸t cõa anh Ph¤m Tu§n Huy tø n«m 2013, c¡c b¤n s³ th§y r§t nhi·u b i vi¸t m t¡c gi£ v¨n cán l håc sinh cõa

• Nếu `2 ≡ 1 (mod 4) thì ta có xi − xi+1 + yi − yi+1 ≡ 1 (mod 2) (1 ≤ i ≤ n). Từđó:

n ≡n

∑i=1

(xi − xi+1) +n

∑i=1

(yi − yi+1) = 0 (mod 2)

• Nếu `2 ≡ 0 (mod 4) thì xi − xi+1 ≡ yi − yi+1 ≡ 0 (mod 2). Điều này nghĩa là cácgiá trị (x1, x2, ..., xn) và (y1, y2, ..., yn) lập thành 2 bộ số mà mỗi bộ đều chẵn hoặclẻ. Bây giờ ta tịnh tiến đa giác theo một vector ~v để x1, x2, .., xn, y1, y2, ..., yn đềuchẵn. Vị tự tâm O là gốc toạ độ để có đa giác mới A′1A′2...A′n với 2x′i = xi, 2y′i = yi

(1 ≤ i ≤ n).

Nếu đa giác mới có toạ độ thoả mãn 1 trong 2 trường hợp đầu thì bài toán sẽxong. Ngược lại, nếu luôn rơi vào trường hợp 3, ta sẽ luôn thu được đa giác mớicó toạ độ nguyên độ dài cạnh mà mỗi lần giảm ít nhất một nửa so với đa giácban đầu. Nhưng điều này là không thể vì khoảng cách nhỏ nhất giữa 2 điểmnguyên luôn là 1 đơn vị.

Tóm lại, nếu các đỉnh có toạ độ nguyên thì n là số chẵn. Với trường hợp các đỉnh cótoạ độ hữu tỉ, vị tự với tâm là gốc toạ độ để thu được đa giác có toạ độ nguyên. Bàitoán kết thúc.

Nhận xét. Số nguyên dương n duy nhất thoả mãn bài toán là n = 4. Điều này có thể đượcchứng minh bởi định lý Pick, nhưng do vượt quá khuôn khổ bài viết nên không trình bày tạiđây.

Rõ ràng ở bài toán trên, ta đã thực hiện được hai thao tác là đưa về trường hợpriêng và điều chỉnh các trường hợp con phát sinh. Cụ thể hơn, việc lùi "vô hạn" sẽ dẫnđến điều vô lý, mà lập luận thường gặp nhất dựa trên một dãy số nguyên dương. Dĩnhiên trong nhiều bài toán khác, ta cũng làm việc với các tình huống tương tự, nhưngsẽ phức tạp hơn hẳn.

Bài toán IMO Shortlist 2017 A7 tiếp theo sẽ cho thấy rõ hơn sự phức tạp đó.

Bài toán 12 (IMO Shortlist 2017). Cho a0, a1, a2, ... là dãy vô hạn các số nguyên vàb0, b1, b2, ... là dãy vô hạn các số nguyên dương, đồng thời thoả mãn điều kiện sau:

a0 = 0, a1 = 1, an+1 =

{anbn + an−1 nếu bn−1 = 1

anbn − an−1 nếu bn−1 > 1∀n ∈ N∗

Chứng minh rằng ít nhất một trong hai số a2017, a2018 không nhỏ hơn 2017.

Lời giải. Với giả thiết, do a0 = 0 nên a2 = a1b1 + a0 = b1 ≥ 1 ∀b0 ∈ N∗.

• Nếu b1 = 1 thì a3 = a2b2 + a1 = a2 + a1 ≥ 2• Nếu b1 ≥ 2 thì a2 = b1 ≥ 2 và a3 = a2b2 − a1 ≥ 2− 1 = 1

Cả hai trường hợp thì ta đều có max{a2, a3} ≥ 2. Cụ thể hơn thì a1, a2, a3 ∈ N∗.

76

Page 77: I HÅC QUÈC GIA TP. HCM · 2020. 10. 18. · Ngo i b i vi¸t cõa anh Ph¤m Tu§n Huy tø n«m 2013, c¡c b¤n s³ th§y r§t nhi·u b i vi¸t m t¡c gi£ v¨n cán l håc sinh cõa

Bây giờ ta chứng minh rằng an ∈ N∗ ∀n ≥ 2 bằng nguyên lý quy nạp. Giả sử rằngđiều này đã đúng đến n = l (n ≥ 3). Có các khả năng sau xảy ra:

• Nếu bl−1 = 1 thì al+1 = albl + al−1 ∈ N∗.• Nếu bl−1 ≥ 2 thì al+1 = albl − al−1. Đến đây ta lại rẽ nhánh thành các khả năng

sau:

– Nếu bl−2 = 1 thì al = al−1bl−1 + al−2 ∈ N∗.Hơn nữa al ≥ al−1 + 1 nên al+1 = albl− al−1 ≥ al− al−1 = 1 hay al+1 ∈ N∗

– Nếu bl−2 ≥ 2 thì al = al−1bl−1 − al−2. Đến đây ta lại rẽ nhánh...

Quá trình rẽ nhánh luôn tạo thành 2 khả năng: một khả năng giải quyết được trọnvẹn, và khả năng còn lại là có chỉ số s để bs ≥ 2 với s giảm ngặt. Dãy chỉ số là một dãynguyên, không âm và giảm ngặt nên phải có lúc dừng. Trường hợp tệ nhất chính làkhi b0 ≥ 2. Lúc đó thì:

min{b0, b1, ..., bl−1} ≥ 2

Chú ý rằng a2 = b1 ≥ 2 ≥ a1 + 1 nên thực hiện quá trình "truy ngược" lên, ta sẽ có:

ai ≥ ai−1bi−1 − ai−2 ≥ 2ai−1 − ai−2 ≥ ai−1 + 1 ∀i ∈ {2; ...; l + 1}

Nói riêng thì al+1 ≥ al + 1 và al+1 ∈ Z nên al+1 ∈ N∗.Từ đây theo nguyên lý quy nạp thì an ∈ N∗ ∀n ≥ 2. Chú ý ở quá trình quy nạp, ta đãcó:

"Nếu có các chỉ số s, l để bs = 1 và bs+1, ..., bs+l ≥ 2 thì as+1 < as+2 < ... <as+l+2"

Bây giờ ta chứng minh rằng max{an+2, an+3} ≥ max{an, an+1}+ 2 ∀n ≥ 1. Có cáckhả năng:

• Nếu bn = bn+1 = 1 thì

{an+2 = an+1bn+1 + an ≥ 1 + max{an, an+1}an+3 = an+2bn+2 + an+1 ≥ 1 + an+2 ≥ 2 + max{an, an+1}

• Nếu bn = 1 và bn+1 ≥ 2 thì:

max{an+2, an+3} ≥ an+2 = an+1bn+1 + an ≥ 2an+1 + an ≥ 2 + max{an, an+1}

• Nếu bn, bn+1 ≥ 2 thì theo phần chứng minh trên, ta có:

– Nếu bn−1 = 1 thì an < an+1 < an+2 < an+3, do đó thì:

max{an+2, an+3} ≥ 2 + an+1 = 2 + max{an, an+1}

– Nếu bn−1 ≥ 2 thì ta cũng có điều tương tự.

• Nếu bn ≥ 2 và bn+1 = 1, ta có các khả năng sau phải xem xét:

77

Page 78: I HÅC QUÈC GIA TP. HCM · 2020. 10. 18. · Ngo i b i vi¸t cõa anh Ph¤m Tu§n Huy tø n«m 2013, c¡c b¤n s³ th§y r§t nhi·u b i vi¸t m t¡c gi£ v¨n cán l håc sinh cõa

– Nếu bn−1 = 1 thì do bn ≥ 2 nên an < an+1 < an+2. Từ đó thì:

an+3 = an+2bn+2 + an+1 > an+2

Từ đó an < an+1 < an+2 < an+3 nên max{an+2, an+3} ≥ max{an, an+1}+2.

– Nếu bn−1 ≥ 2 thì ta cũng có an < an+1 < an+2 < an+3 nên cũng xong.

Tóm lại ta đã chứng minh được max{an+2, an+3} ≥ max{an, an+1}+ 2 ∀n ≥ 1.Áp dụng liên tiếp thì max{a2017, a2018} ≥ max{a1, a2}+ 2016 ≥ 2017. Vậy phải có ítnhất một trong hai số a2017, a2018 không nhỏ hơn 2017. Phép chứng minh kết thúc.

Những ví dụ được giới thiệu đều có điểm chung là ta cố gắng tìm một (hoặc mộtvài) yếu tố hữu hạn được ẩn giấu. Để kết thúc mục này, bạn đọc hãy tự mình xem xétbài tập 8, đó là một bài toán hay, đòi hỏi sự lao động tỉ mỉ và có nhiều thao tác "ở vôcùng".

2.3. Tham lam

Ý tưởng tham lam thường gợi các suy nghĩ về thuật toán. Thật ra không hẳn nhưthế, đơn giản chỉ là việc chọn thật nhiều các đối tượng thoả mãn bài toán, để việcchọn là tối đa theo tiêu chí nào đó. Một ví dụ dễ hiểu là nguyên lý cực hạn. Cụ thểhơn một chút, trong lĩnh vực đồ thị hữu hạn, để chứng minh các định lý về đường đivà chu trình Hamilton thì ta thường đề cập đến đường đi dài nhất. Nhưng ý tưởngtham lam không chỉ dừng lại ở cực hạn và các biến thể.

Để có thể hình dung rõ hơn, ta xét bài toán sau:

Bài toán 13. Trong các số có dạng k! (1 ≤ k ≤ 50), cần phải loại bỏ ít nhất bao nhiêu số đểcác số còn lại có tích là số chính phương?

Lời giải. Xét tích sau:

50

∏i=1

i! =25

∏i=1

(2i− 1)! · (2i)! = 225 · 25! ·25

∏i=1

((2i− 1)!)2

Bằng một vài tính toán, ta có 2 · 25! · k! phải là số chính phương với 1 ≤ k ≤ 50 nếuchỉ loại đi một số là k!, nhưng đây là điều vô lý, nên ta phải loại bỏ ít nhất hai số. Hơnnữa khi loại bỏ 2! và 25! thì tích còn lại là số chính phương. Vậy cần loại bỏ ít nhất haisố.

Bài toán 14. Cho A = {1; 2; ...; 2018}. Cần phải loại khỏi A ít nhất bao nhiêu phần tử đềtrong phần còn lại, không có phần tử nào bằng tích 2 phần tử khác?

Lời giải. Ta chứng minh rằng cần loại bỏ ít nhất 43 phần tử. Một ví dụ là loại bỏ{2; 3; ...; 44} để thu được tập hợp {1; 45; 46; ...; 2018}. Rõ ràng tích của hai phần tử bấtkỳ hoặc là bằng chính nó, hoặc là vượt quá 452 = 2025 > 2018.

78

Page 79: I HÅC QUÈC GIA TP. HCM · 2020. 10. 18. · Ngo i b i vi¸t cõa anh Ph¤m Tu§n Huy tø n«m 2013, c¡c b¤n s³ th§y r§t nhi·u b i vi¸t m t¡c gi£ v¨n cán l håc sinh cõa

Ta chứng minh rằng mọi cách loại bỏ 42 phần tử đều không thoả mãn yêu cầu. Xétcác bộ ba có dạng (k; 89− k; k(89− k)) với 2 ≤ k ≤ 44. Các phần tử được liệt kê đềulà phân biệt, và có 43 bộ ba, nên nếu loại bỏ 42 phần tử khỏi A thì vẫn còn ít nhất mộtbộ ba, dẫn đến điều mâu thuẫn.Vậy số phần tử cần loại bỏ ít nhất là 43 phần tử.

Trong lời giải trên, sự chọn lựa 43 phần tử là một việc khá liều lĩnh, bởi vì số nàyrất nhỏ so với kích thước của tập hợp A. Việc nhìn nhận theo hướng tối đại (hoặc tốithiểu) sẽ thường dẫn đến những kết luận không ngờ, và việc kiểm chứng tính đúngđắn phụ thuộc vào kinh nghiệm của người giải. Bài toán tiếp theo, rất giống với bài 1của Việt Nam TST 2015, sẽ cho thấy điều đó:

Bài toán 15 (Nga 2014). Kho bạc của nước "Cộng hoà toán học" chọn một số thực α > 2rồi phát hành các đồng xu có mệnh giá là 1 rub và αk rub ∀k ∈ N∗, hơn nữa αk là số vô tỉ∀k ∈ N∗. Liệu có xảy ra việc sau hay không: ∀n ∈ N∗, ta luôn chọn được một số đồng xu cótổng mệnh giá bằng n và mỗi loại được chọn không quá 6 đồng?

Lời giải. Rất ngạc nhiên, câu trả lời ở đây là khẳng định.

Xét α =−1 +

√29

2> 2 là nghiệm dương của phương trình x2 + x = 7. Dễ thấy α là

số vô tỉ. Bây giờ ta chứng minh rằng αk là số vô tỉ ∀k ∈ N∗.

Giả sử rằng tồn tại k để αk ∈ Q. Đặt β =1 +√

292

thì αβ = 7 hay αkβk = 7k. Từ việc

αk ∈ Q thì βk ∈ Q. Hơn nữa do βk > 0 nên βk ∈ Q+. Với m, n, p, q ∈ Q+, xét phântích sau:

(m + n√

29)(p + q√

29) = (mp + 29nq) + (np + mq)√

29 6∈ Q+

Áp dụng liên tục điều này thì βk 6∈ Q+, trái với điều giả sử. Do đó điều giả sử là sai,nghĩa là αk 6∈ Q ∀k ∈ N∗.Giờ ta chỉ ra thuật toán chọn bộ xu có tổng mệnh giá là n bằng nguyên lý quy nạp. Dễthấy với n ≤ 7 thì ta chọn được bộ xu. Giả sử rằng chọn được bộ xu với tổng mệnhgiá k ≥ 7, ta xây dựng bộ xu với tổng mệnh giá k + 7 như sau:

• Bổ sung 7 đồng 1 rub vào bộ xu có tổng mệnh giá bằng k.• Nếu có mệnh giá αl (l ∈ N) có ít nhất 7 đồng xu, do đẳng thức 7αl = (α2 + α)αl ,

ta thay thế 7 đồng xu αl trong bộ xu bởi 2 đồng xu là αl+2 + αl+1.

Do số đồng xu giảm ngặt nên thuật toán dừng sau hữu hạn bước. Sau cùng, ta sẽ thuđược bộ xu thoả mãn với tổng mệnh giá k + 7. Theo nguyên lý quy nạp, ∀n ∈ N∗, taluôn chọn được một số đồng xu có tổng mệnh giá bằng n, mỗi loại được chọn khôngquá 6 đồng. Bài toán kết thúc.

Bài toán 16 (Trung Quốc TST 2015). Cho X 6= ∅ là tập hữu hạn. Xét các tập hợpA1, A2, ..., Ak ⊂ X sao cho |Ai| ≤ 3 ∀i ∈ {1; 2; ...; k}. Giả sử rằng mỗi phần tử của X thuộc

79

Page 80: I HÅC QUÈC GIA TP. HCM · 2020. 10. 18. · Ngo i b i vi¸t cõa anh Ph¤m Tu§n Huy tø n«m 2013, c¡c b¤n s³ th§y r§t nhi·u b i vi¸t m t¡c gi£ v¨n cán l håc sinh cõa

ít nhất 4 tập hợp con. Chứng minh rằng có thể chọn ra được⌊

3k7

⌋tập con trong bộ trên mà

có hợp là X.

Lời giải. Ta sẽ chọn các tập hợp bởi thuật toán sau:

• Chọn họ tập hợp cực đại từ X gồm các tập hợp mà mỗi lượt ta thêm một tậpvào họ, số phần tử của hợp các tập hợp đó tăng thêm 3 đơn vị. Không mất tính

tổng quát, xem như đó là các tập hợp A1, ..., Am. Đặt X3 =m⋃

i=1

Ai.

Từ đó với i > m thì |Ai ∩ (X\X3)| ≤ 2 bởi tính cực đại.• Tiếp theo, chọn họ tập hợp cực đại từ X\X3 gồm các tập hợp mà mỗi lượt ta

thêm một tập vào họ, số phần tử của hợp các tập hợp đó tăng thêm 2 đơnvị. Không mất tính tổng quát, xem như đó là các tập hợp Am+1, ..., Am+n. Đặt

X2 =m+n⋃

i=m+1

Ai.

Từ đó với i > m + n thì |Ai ∩ (X\X3\X2)| ≤ 1 bởi tính cực đại.• Cuối cùng, chọn các tập hợp từ X\X3\X2 gồm các tập hợp mà mỗi lượt ta thêm

một tập vào họ, số phần tử của hợp các tập hợp đó tăng thêm 1 đơn vị. Chọnđến khi hợp tất cả các tập hợp bằng X. Giả sử ta đã chọn p tập hợp. Gọi hợp củahọ này là X1.

Ta cần chứng minh rằng m + n + p ≤ 3k7

, chú ý là 3m + 2n + p = |X|.

• Mỗi phần tử của X1 thuộc ít nhất 4 tập hợp, mà |Ai ∩ (X\X3\X2)| ≤ 1 (i >m + n) nên:

k ≥ m + n + 4p

• Mỗi phần tử của X1 ∪ X2 thuộc ít nhất 4 tập hợp, mà |Ai ∩ (X\X3)| ≤ 2 (i > m)

nên:

k ≥ m +4(2n + p)

2

• Mỗi phần tử của X thuộc ít nhất 4 tập hợp, mà |Ai| ≤ 3 với 1 ≤ i ≤ k nên:

k ≥ 4(3m + 2n + p)3

Từ đây ta có:

59k ≥ 20(m + n + 4p) + 12(

m +4(2n + p)

2

)+ 27 · 4(3m + 2n + p)

3

Hay là m + n + p ≤ 59k140

<3k7

. Vậy ta đã chọn được⌊

3k7

⌋tập con mà có hợp là X.

80

Page 81: I HÅC QUÈC GIA TP. HCM · 2020. 10. 18. · Ngo i b i vi¸t cõa anh Ph¤m Tu§n Huy tø n«m 2013, c¡c b¤n s³ th§y r§t nhi·u b i vi¸t m t¡c gi£ v¨n cán l håc sinh cõa

Ta kết thúc bằng một bài toán rất khó từ tạp chí KoMaL của Hungary:

Bài toán 17 (KoMaL, A.713). Dãy số vô hạn phần tử a1, a2, ... được gọi là tốt nếu như:

∀j ∈ N∗, ∀i < j : |ai − aj| ≥1j

Tìm tất cả các số thực c sao cho tồn tại một dãy số tốt trong đoạn [0, c].

Lời giải. Dễ thấy các số hạng là phân biệt. Với mỗi n ∈ N∗, ta đặt dãy số

0 ≤ ai1 < ai2 < ... < ain

là hoán vị được xếp theo thứ tự tăng dần của a1, a2, ..., an với các chỉ số là hoán vị của(1, 2, .., n).Ta có bất đẳng thức sau:

c ≥ ain − ai1 =n−1

∑k=1

(aik+1 − aik) ≥n−1

∑k=1

1min{ik, ik+1}

Mỗi số nguyên dương trong {1; 2; ...; n} chỉ xuất hiện tối đa 2 lần trong tất cả các mẫusố, nên để tổng thu được cuối cùng là nhỏ nhất thì mỗi số trong {1; 2; ...; n} phải xuấthiện 2 lần, theo thứ tự ưu tiên từ số lớn nhất đến số nhỏ nhất.Với n chẵn, đặt n = 2t (t ∈ N∗) thì:

c ≥ 22t

∑k=t+2

1k

Cho t→ +∞ thì ta có c ≥ 2 ln 2. Với n lẻ, tương tự ta cũng có c ≥ 2 ln 2.Bây giờ ta chỉ cần xây dựng dãy số với c = 2 ln 2. Ý tưởng xây dựng như sau:

• Đầu tiên, chọn a1 = 0, a2 = 2 ln 2. Ta có 1 khoảng với đầu mút là a1, a2.• Chọn a3 nằm giữa a1, a2. Ta có 2 khoảng rời nhau, nên khi xếp a4, a5 vào đó lần

lượt từ trái qua thì sẽ thu được 4 khoảng. Tương tự, khi có 2k + 1 số hạng đầutiên của dãy số xếp để tạo thành đầu mút của 2k khoảng, ta tiếp tục xếp 2k phầntử a2k+2, ..., a2k+1+1 vào các khoảng theo thứ tự từ trái qua.• Bây giờ chọn giá trị các số hạng để cách xây dựng thoả mãn. Chẳng hạn như:

a3 = 2 · limk→+∞

2k−1−1

∑l=1

12l + 2 + l

Giới hạn này dễ kiểm chứng rằng có tồn tại.

Việc xây dựng các số hạng khá phức tạp nên sẽ được để lại như bài tập.Tóm lại, tất cả các số thực c thoả mãn bài toán là c ≥ 2 ln 2. Bài toán kết thúc.

81

Page 82: I HÅC QUÈC GIA TP. HCM · 2020. 10. 18. · Ngo i b i vi¸t cõa anh Ph¤m Tu§n Huy tø n«m 2013, c¡c b¤n s³ th§y r§t nhi·u b i vi¸t m t¡c gi£ v¨n cán l håc sinh cõa

3. BÀI TẬP

Bài tập 1. Giải lại bài toán 1 khi các số được xếp trên một vòng tròn.Bài tập 2 (Bulgaria TST 2003). Liệu với mọi hoán vị (a1, a2, ..., a2002) của (1, 2, ..., 2002),có luôn tồn tại 1 ≤ m < n ≤ 2002 để m, n có cùng tính chẵn lẻ và 2a m+n

2= am + an ?

Bài tập 3. Một đa giác lồi được chia thành các tam giác cân bởi các đường chéo màđôi một không có điểm trong chung. Chứng minh rằng đa giác có ít nhất hai cạnhbằng nhau.Bài tập 4. Chứng minh rằng với a1, a2, ..., an là các số nguyên dương phân biệt thì:

n

∑i=1

(a7i + a5

i ) ≥ 2

(n

∑i=1

a3i

)2

Bài tập 5 (Nga 2017). Cho các số thực dương a1, a2, ..., a2017. Chứng minh rằng ta cóthể tìm được các số thực dương b1, b2, ..., b2017 thoả mãn đồng thời các điều kiện sau:

i. bi ≥ ai ∀i ∈ {1; 2; ...; 2017}ii. Với 1 ≤ i < j ≤ 2017 thì ít nhất một trong hai số

bi

bj,

bj

bilà số nguyên.

iii. b1b2...b2017 ≤ 21008a1a2...a2017

Bài tập 6 (RMM 2015). Cho đa giác đều n đỉnh với n ≥ 5. Ban đầu có 3 tấm thẻ đượcđặt trên 3 đỉnh kề nhau của đa giác. Có 2 người cùng trò chơi sau, luân phiên mỗingười 1 lượt: mỗi người đến lượt đi của mình dịch chuyển tấm thẻ dọc theo các đỉnhcủa đa giác, không được nhảy qua bất kỳ một tấm thẻ nào sao cho diện tích của tamgiác có 3 đỉnh ở vị trí 3 tấm thẻ luôn tăng thật sự so với lượt chơi trước. Ai không thựchiện được nước đi ấy nữa sẽ thua cuộc. Xác định tất cả các giá trị n để người chơi đầutiên luôn có chiến thuật thắng.Bài tập 7 (Định lý Cauchy - Davenport). Với p là số nguyên tố lẻ, đặt Zp = {0, 1, ..., p−1}. Gọi A, B là hai tập hợp con bất kỳ của Zp. Khi đó:

|A + B| ≥ min{p, |A|+ |B| − 1}

Với quy ước A + B = {a + b | a ∈ A, b ∈ B} và các phần tử của A + B được lấy từ Zp.Bài tập 8 (Bulgaria 2003). Tìm tất cả các đa thức hệ số nguyên P(x) thoả mãn điềukiện sau: với mỗi số nguyên dương n, tồn tại số nguyên x để P(x) = 2n.Bài tập 9. Cho số nguyên dương m > 1. Chứng minh rằng tồn tại đa thức hệ sốnguyên P(x), bất khả quy trên Z[x], có bậc 2018 và một nghiệm thực là r thoả mãnm | 1 + brnc ∀n ∈ N∗.Bài tập 10. Cho trước số nguyên dương n ∈ N∗. Tính số lượng các đa thức có bậckhông vượt quá n, có tất cả các hệ số thuộc {0; 1; ...; 8} và f (3) = n.Bài tập 11. Cho bảng ô vuông n× n. Tô màu đen một số ô vuông trong bảng.

1. Xét n là số chẵn và mỗi ô vuông trong bảng đều có cạnh chung với ít nhất 1 ôđen. Hãy tìm số lượng ô đen nhỏ nhất có thể trên bảng ô vuông.

82

Page 83: I HÅC QUÈC GIA TP. HCM · 2020. 10. 18. · Ngo i b i vi¸t cõa anh Ph¤m Tu§n Huy tø n«m 2013, c¡c b¤n s³ th§y r§t nhi·u b i vi¸t m t¡c gi£ v¨n cán l håc sinh cõa

2. Xét n ≥ 5 và mỗi ô vuông không được tô màu đều có cạnh chung với ít nhất 1 ôđen. Gọi f (n) là số ô đen nhỏ nhất. Đặt n + 2 ≡ r (mod 5) với 1 ≤ r ≤ 5. Chứngminh rằng:

f (n) ≤ 5⌊

n− 35

⌋2

+ r

(⌊n + 2

5

⌋2

−⌊

n− 35

⌋2)− 4

Liệu bất đẳng thức này có phải là đánh giá tốt nhất không ?

Bài tập 12. Ta biết rằng tồn tại đẳng thức sau:

∑n=1

1n2 =

π2

6

Cho q là số hữu tỉ và q ∈(

0,π2

6− 1)⋃ [

1,π2

6

). Chứng minh rằng tồn tại các số

nguyên dương phân biệt q1 < q2 < ... < qk để đẳng thức sau được thoả mãn:

q =1q2

1+

1q2

2+ ... +

1q2

k

Bài tập 13. Cho 100 điểm nằm trong hình vuông đơn vị. Chứng minh rằng ta có thểnối chúng bởi 1 đường gấp khúc có độ dài không vượt quá 20.Bài tập 14. Cho n điểm A1, A2, ..., An nằm trong hình vuông đơn vị. Gọi d1, d2, ..., dn

lần lượt là khoảng cách từ mỗi điểm A1, A2, ..., An đến điểm gần nhất. Chứng minhrằng:

d21 + d2

2 + ... + d2n ≤ 4

Bài tập 15 (USA TSTST 2018). Chứng minh rằng tồn tại một hằng số c < 1 sao cho vớimọi đa giác P có diện tích bằng 1, ta có thể tịnh tiến P theo một chiều xác định nàođó để thu được đa giác Qmà diện tích chung giữa P ,Q không vượt quá c.

Lời kết

Bài viết đã giới thiệu một số bài toán điển hình cho những hướng tiếp cận tổngquát nhất khi giải toán Olympic. Dĩ nhiên để giải quyết những bài toán phức tạp, takhông thể phủ nhận vai trò của các kiến thức bổ sung, nhưng việc rèn luyện với cácvấn đề tổng quát nhất sẽ giúp tư duy được trong sáng và sắc bén hơn. Rất mong nhậnđược sự góp ý của bạn đọc về nội dung.

Cuối cùng, tác giả gửi lời cảm ơn đến thầy Lê Phúc Lữ đã đọc lại bản thảo và đưara những góp ý xác đáng để bài viết được hoàn thiện hơn.

TÀI LIỆU THAM KHẢO1. Các bài toán ôn thi HSG Quốc gia năm 2018 của đội tuyển PTNK.2. Dongryul Kim, Problems of Week for SSHS Korea 2014, 2015.

83

Page 84: I HÅC QUÈC GIA TP. HCM · 2020. 10. 18. · Ngo i b i vi¸t cõa anh Ph¤m Tu§n Huy tø n«m 2013, c¡c b¤n s³ th§y r§t nhi·u b i vi¸t m t¡c gi£ v¨n cán l håc sinh cõa

3. Trần Nam Dũng, Các bài toán ôn tập đội tuyển IMO 2016.4. IMO Shortlist 2017.5. Tạp chí Kvant các năm 2014-2017.6. Tạp chí Pi số 1/2018, chuyên mục Thách thức toán học.7. Diễn đàn Mathscope, topic Hướng tới Việt Nam TST 2018: http://mathscope.org/showthread.php?t=51642

84

Page 85: I HÅC QUÈC GIA TP. HCM · 2020. 10. 18. · Ngo i b i vi¸t cõa anh Ph¤m Tu§n Huy tø n«m 2013, c¡c b¤n s³ th§y r§t nhi·u b i vi¸t m t¡c gi£ v¨n cán l håc sinh cõa

VỀ BÀI TOÁN TỔ HỢP TRONG KỲ THICHỌN ĐỘI DỰ TUYỂN PTNK NĂM 2017

Phan Quốc Vượng

Lớp chuyên Toán khoá 2016-2019

LỜI BAN BIÊN TẬP. Bài viết được hoàn thành vào tháng 4/2017 trước khi tác giảphải chuyển trường bởi lý do cá nhân. Được sự đồng ý của tác giả, ban biên tậpchỉnh sửa và đăng bài viết này dựa trên sự tổng hợp hai bản thảo: bản thảo đầutiên rất vắn tắt và có độ dài... một nửa mặt giấy A5, bản thảo còn lại giải thích rõhơn những ý tưởng chính với độ dài một mặt giấy A4. Có thể xem đây là một kỷniệm đẹp của tác giả với những người bạn: "Một lần là Năng Khiếu, mãi mãi là NăngKhiếu".

1. BÀI TOÁN GỐC

Trong kỳ thi chọn đội dự tuyển khối 10 trường PTNK năm 2017 xuất hiện bàitoán sau ở vị trí bài toán số 4, cũng là bài toán cuối cùng.

Bài toán 1. Cho S là tập hợp khác rỗng và A1, A2, ..., Am (m ≥ 2) là m tập con của S. GọiT là tập hợp gồm tất cả các tập Ai4 Aj (1 ≤ i, j ≤ m). Chứng minh rằng |T| ≥ m

Lời giải sau được đề nghị trong kỳ thi bởi Nguyễn Nguyễn.

Lời giải thứ nhất. Trước hết ta chứng minh rằng, nếu M4N = M4P thì N = P. Giảsử rằng N 6= P. Không mất tính tổng quát, gọi x là phần tử sao cho x ∈ N và x 6∈ P.Có hai khả năng sau xảy ra:

• Nếu x ∈ M, khi đó ta có x 6∈ M\N và x 6∈ N\M, do đó x 6∈ M4N, nhưng lạicó x ∈ M4P nên M4N 6= M4P, trái với giả thiết.• Nếu x 6∈ M, khi đó tương tự ta có x ∈ M4N và x 6∈ M4P, dẫn đến M4N 6=

M4P, trái với giả thiết.

Do đó N = P. Bổ đề được chứng minh. Quay trở lại bài toán ban đầu. Ta sẽ thực hiệnchứng minh bằng quy nạp theo m. Với m = 2, dễ thấy rằng T = {∅, A14A2} hay|T| = 2. Bây giờ giả sử rằng bài toán đã đúng đến m = k (k ∈ N∗, k ≥ 2), ta sẽ chứngminh rằng bài toán cũng đúng với m = k + 1. Nếu như với m = k mà ta có |T| = k + 1thì bài toán hiển nhiên đúng, nên ta chỉ xét trường hợp |T| = k.

85

Page 86: I HÅC QUÈC GIA TP. HCM · 2020. 10. 18. · Ngo i b i vi¸t cõa anh Ph¤m Tu§n Huy tø n«m 2013, c¡c b¤n s³ th§y r§t nhi·u b i vi¸t m t¡c gi£ v¨n cán l håc sinh cõa

Xét dãy tập hợp Bi = Ak+14Ai(1 ≤ i ≤ k + 1). Ta bổ sung vào T các tập hợp Bi. Rõràng |T| là đại lượng không giảm. Giả sử rằng vẫn xảy ra |T| = k, do trong dãy tậphợp vừa thêm có k + 1 tập nên phải tồn tại chỉ số p, q (1 ≤ p < q ≤ k + 1) sao choBp = Bq. Từ đó theo bổ đề dẫn đến Ap = Aq, trái với giả thiết các tập hợp Ai là đôimột phân biệt. Do đó điều đã giả sử là sai, hay là |T| ≥ k + 1. Tóm lại ta luôn chứngminh được |T| ≥ k + 1, quy nạp thành công.Vậy bài toán được giải quyết hoàn toàn.

Lời giải trên có thể làm gọn hơn hẳn lại như sau mà không cần quy nạp:

Lời giải thứ hai. Ta cũng cần bổ đề ở lời giải thứ nhất.Xét tập hợp A1 và các tập hợp A14 Ai với 1 ≤ i ≤ m. Theo bổ đề trên thì m tập hợpnày đôi một phân biệt nên |T| có ít nhất m phần tử hay |T| ≥ m.

Qua những lời giải trên, ta nhận thấy ý tưởng giải chung đều khá đẹp và trongsáng, do đó đây không thể xem là bài toán khó. Tuy nhiên với thời gian làm bài ngắncùng việc 3 bài toán đầu tiên không hề dễ dàng nên dẫn đến việc rất ít học sinh làmđược bài toán này. Với cách chứng minh trên, ta cũng đồng thời thấy được đánh giá|T| ≥ m là không chặt, ta sẽ tập trung khai thác phần này.

2. KHAI THÁC BÀI TOÁN GỐC

Một hướng khai thác rất tự nhiên cho bài toán 1 chính là tìm ra một chặn dưới tốthơn cho |T|. Điều này nghĩa là ta sẽ tìm biểu thức A = f (m) mà có được đánh giá:

|T| ≥ A = f (m) ≥ m ∀m ∈ N∗, m ≥ 2

Để dễ dàng, ta xét một trường hợp nhỏ và dễ kiểm soát như S = {1, 2, 3, 4, 5}.Khi đó, bằng việc thử các giá trị m một cách cụ thể, ta có được các kết quả sau:

• Với m = 2 thì |T| = 2• Với m ∈ {3, 4} thì |T| ≥ 4• Với m ∈ {5, 6, 7, 8} thì |T| ≥ 8• Với m ∈ {9, 10, 11, ..., 16} thì |T| ≥ 16

Từ đây ta mạnh dạn nghĩ đến đánh giá |T| ≥ 2r với r = dlog2(m)e. Rất thú vị,đây lại là một nhận xét đúng và chúng ta sẽ chứng minh bài toán sau:

Bài toán 2. Cho S là tập hợp khác rỗng và A1, A2, ..., Am (m ≥ 2) là m tập con của S. GọiT là họ gồm tất cả các tập Ai4 Aj (1 ≤ i, j ≤ m). Tìm giá trị lớn nhất và nhỏ nhất của |T|.

Lời giải. Nếu không gây bất kỳ nhầm lẫn, ta quy ước XY = X4 Y. Cách viết nàyhoàn toàn hợp lý trong phạm vi bài toán do dễ dàng kiểm chứng được rằng phép lấyhiệu đối xứng có tính chất giao hoán và kết hợp.

86

Page 87: I HÅC QUÈC GIA TP. HCM · 2020. 10. 18. · Ngo i b i vi¸t cõa anh Ph¤m Tu§n Huy tø n«m 2013, c¡c b¤n s³ th§y r§t nhi·u b i vi¸t m t¡c gi£ v¨n cán l håc sinh cõa

Rõ ràng Ai Ai = ∅ (1 ≤ i ≤ m). Ta có C2m cách chọn ra 2 tập hợp phân biệt từ m tập

hợp Ai (1 ≤ i ≤ m) cho trước, do đó để |T| lớn nhất thì C2m hiệu đối xứng tạo thành

từ các cặp phải đôi một phân biệt, từ đó ta có đánh giá:

|T| ≤ C2m + 1 =

m(m− 1)2

+ 1

Đẳng thức xảy ra chẳng hạn khi S = N∗ và Ai = {i} (1 ≤ i ≤ m).Bây giờ ta tìm giá trị nhỏ nhất của |T|. Với m tập hợp A1, A2, ..., Am, ta định nghĩa họtập hợp (C1, C2, ..., Cl) là một hệ cơ sở của m tập hợp trên nếu như chúng thoả mãnđồng thời các điều kiện sau đây:

• Không tồn tại x, y1, y2, ..., yj (1 ≤ x, yi ≤ l, 1 ≤ i ≤ j ≤ l − 1) thoả mãn:

Cx = Cy1Cy2 ...Cyt

• Với mỗi tập hợp Ai (1 ≤ i ≤ m), tồn tại xi1 , xi2 , ..., xij (1 ≤ j ≤ l) thoả mãn:

Ai = Cxi1Cxi2

...Cxij

Nói cách khác, biểu diễn của mỗi tập hợp Ai theo hệ cơ sở trên là duy nhất. Ta địnhnghĩa rằng số phần tử của một hệ cơ sở chính là số tập hợp của hệ đó.Ta khẳng định sự tồn tại của ít nhất một hệ cơ sở với mỗi bộ m tập hợp mà ta chọn.

Thật vậy, xét tập hợpm⋃

i=1

Ai có các phần tử là a1, a2, ..., an thì ta chọn n tập hợp:

Ci = {ai} (1 ≤ i ≤ n).

Dễ thấy rằng các tập hợp này thoả mãn những tính chất của hệ cơ sở. Do đó sự tồntại của hệ cơ sở được khẳng định. Hơn nữa số phần tử của hệ dĩ nhiên là số nguyêndương nên phải tồn tại một hệ cơ sở có số phần tử nhỏ nhất. Gọi hệ cơ sở có số phầntử nhỏ nhất đó là (C1, C2, ..., Ck).Ta sẽ xét hệ cơ sở nhỏ nhất trong toàn bộ phần sau của lời giải.Nhận xét rằng để biểu diễn được một tập hợp Ai (1 ≤ i ≤ m), mỗi tập hợp Cj

(1 ≤ j ≤ k) có đúng 2 trạng thái là xuất hiện hay không xuất hiện trong biểu diễn.Mặt khác hệ cơ sở nhỏ nhất có đúng k phần tử nên m ≤ 2k hay k ≥ dlog2(m)e.Có 2 khả năng sau có thể xảy ra:

• k = dlog2(m)e. Điều này nghĩa là 2m > 2k. Ta chứng minh rằng mỗi tập hợp Ctrong hệ cơ sở đều có thể biểu diễn được dưới dạng Ai Aj (1 ≤ i, j ≤ m). Đếnđây có 2 khả năng:

– Nếu như C = ∅ thì rõ ràng C = Ai Ai với 1 ≤ i ≤ m.– Nếu như C 6= ∅ thì ta xét các tập hợp Ai và AiC (1 ≤ i ≤ m). Số tập hợp

vừa chọn là 2n > 2k nên tồn tại 2 tập hợp vừa chọn có cùng biểu diễn trong

87

Page 88: I HÅC QUÈC GIA TP. HCM · 2020. 10. 18. · Ngo i b i vi¸t cõa anh Ph¤m Tu§n Huy tø n«m 2013, c¡c b¤n s³ th§y r§t nhi·u b i vi¸t m t¡c gi£ v¨n cán l håc sinh cõa

hệ cơ sở nhỏ nhất. Ta dễ dàng loại các khả năng Ai = Aj, AiC = AjC (dobổ đề ở bài toán 1) nên tồn tại 2 chỉ số i, j mà 1 ≤ i < j ≤ m mà Ai = AjC.Từ đó do tính giao hoán:

Ai Aj = AjCAj = (Aj Aj)C = C

Mặt khác Ai Aj ∈ T nên ta có |T| = 2k với k = dlog2(m)e.• k > dlog2(m)e = r. Điều này nghĩa là m < 2k. Giả sử rằng |T| < 2r.

Gọi X, Y lần lượt là họ các tập hợp Ai (1 ≤ i ≤ m) mà Ai chứa Ck và không chứaCk trong biểu diễn theo hệ cơ sở. Đặt Q = {Ai Aj|1 ≤ i, j ≤ m, Ai ∈ X, Aj ∈ Y}.Rõ ràng Q ⊂ T nên |Q| ≤ |T| < 2r. Gọi R là họ các tập hợp mà có thể biểu diễnđược dưới dạng Cx1Cx2 ...Cxi (1 ≤ x1 < x2 < ... < xi ≤ k− 1). Tương tự nhưtrên, ta có |R| = 2k−1. Mà lại có được:

|{Ai AjCk|1 ≤ i, j ≤ m, Ai ∈ X, Aj ∈ Y}| = |Q| < 2k−1

Hơn nữa Ai AjCk biểu diễn được trong hệ cơ sở k phần tử nên tồn tại tập hợpZ ∈ R mà không thuộc {Ai AjCk|1 ≤ i, j ≤ m, Ai ∈ X, Aj ∈ Y}.Ta sẽ lập các tập hợp mới bằng cách giữ nguyên các phần tử của Y, các phần tửcủa X sẽ được lấy phép hiệu đối xứng với ZCk. Ta dễ thấy rằng các tập hợp mớicó hệ cơ sở là (C1, C2, ..., Ck−1) và hơn nữa chúng đôi một phân biệt. Bây giờ tachứng minh rằng các tập hợp mới đôi một phân biệt với các tập hợp trong họ Y.

Giả sử rằng ta có AiZCk = Aj với Ai ∈ X và Aj ∈ Y. Khi đó:

Ai AjCk = Ai(AiZCk)Ck = Z

Do đó Z ∈ {Ai AjCk|1 ≤ i, j ≤ m, Ai ∈ X, Aj ∈ Y}, trái với điều đã khẳng định.

Điều này dẫn đến các tập hợp mới tạo ra từ X và các tập hợp thuộc Y đôi mộtphân biệt.

Gọi X′, Y′ lần lượt là họ các tập hợp Ai Aj (1 ≤ i, j ≤ m), Ai Aj chứa và khôngchứa Ck trong cách biểu diễn theo hệ cơ sở. Ta nhận thấy rằng, sau khi lập cáctập hợp mới thì |X′|, |Y′| không đổi. Theo nguyên lý bù trừ, ta lại có:

|T| = |X′ ∪Y′| = |X′|+ |Y′| − |X′ ∩Y′|

Hơn nữa ban đầu |X′ ∩Y′| = 0 nên qua phép biến đổi thì |T| không tăng.

Mặt khác sau phép biến đổi trên, họ các tập hợp lúc này có hệ cơ sở là(C1, C2, ..., Ck−1) có số phần tử nhỏ hơn hệ cơ sở ban đầu nên điều này là vô lý.

Vậy việc giả sử |T| < 2r là sai hay |T| ≥ 2r.

Tóm lại, ta luôn có |T| ≥ 2r với r = dlog2(m)e. Việc chỉ ra trường hợp xảy ra đẳngthức không hề khó, xin dành lại cho bạn đọc.

88

Page 89: I HÅC QUÈC GIA TP. HCM · 2020. 10. 18. · Ngo i b i vi¸t cõa anh Ph¤m Tu§n Huy tø n«m 2013, c¡c b¤n s³ th§y r§t nhi·u b i vi¸t m t¡c gi£ v¨n cán l håc sinh cõa

3. LỜI KẾT

Bài viết trên đã trình bày một hướng khai thác của tác giả về bài toán 4 trong kỳthi chọn đội dự tuyển trường PTNK năm 2017. Rất mong nhận được ý kiến đóng gópcủa bạn đọc để tác giả có thể chỉnh sửa lại bài viết cho hoàn thiện hơn.

Cuối cùng, tác giả cảm ơn các bạn học sinh cùng lớp 10 Toán PTNK niên khoá2016-2019 đã đưa ra những ý tưởng thú vị đưa đến bài toán mở rộng và giúp đỡ biêntập bản thảo bài viết này.

89

Page 90: I HÅC QUÈC GIA TP. HCM · 2020. 10. 18. · Ngo i b i vi¸t cõa anh Ph¤m Tu§n Huy tø n«m 2013, c¡c b¤n s³ th§y r§t nhi·u b i vi¸t m t¡c gi£ v¨n cán l håc sinh cõa
Page 91: I HÅC QUÈC GIA TP. HCM · 2020. 10. 18. · Ngo i b i vi¸t cõa anh Ph¤m Tu§n Huy tø n«m 2013, c¡c b¤n s³ th§y r§t nhi·u b i vi¸t m t¡c gi£ v¨n cán l håc sinh cõa

VỀ BÀI TOÁN XÁC ĐỊNH HẰNG SỐTRONG GIẢI PHƯƠNG TRÌNH HÀM

Nguyễn Mạc Nam Trung

Lớp chuyên Toán khoá 2017 - 2020

GIỚI THIỆU. Đa số các bài toán phương trình hàm trong các kỳ thi học sinh giỏiđều có nghiệm hàm dễ đoán định như hàm hằng hoặc hàm đa thức,... Tuy nhiên,nếu đưa tham số vào bài toán hoặc thay đổi các biến thì ta thu được những bài toánmới có thể có nhiều nghiệm lạ hơn hoặc vô nghiệm. Bài viết này sẽ đề cập một cáchtổng quan về vấn đề xác định tham số để phương trình hàm có nghiệm hoặc một lớpnghiệm mong muốn. Định hướng chung là ta sẽ dự đoán tham số và cố gắng chứngminh bởi các kĩ thuật giải phương trình hàm thông thường, tuy nhiên cũng có đôikhi chúng ta cần phải linh hoạt hơn, như hai bài toán trong phần 3 của bài viết.

1. MỘT SỐ BÀI TOÁN KHỞI ĐỘNG

Ví dụ 1. Tìm tất cả các số nguyên dương a để tồn tại hàm số f : N∗ → N∗ thỏa mãn

f (x + y) = a( f (x) + f (y)) ∀x, y ∈ N∗.

Phân tích. Nếu a = 1 thì dễ thấy f (x) = x f (1) là nghiệm của phương trình hàm. Ta sẽ thửáp dụng kĩ thuật trong trường hợp a = 1 để giải quyết bài toán tổng quát, đó chính là quynạp. Ta tính thử vài giá trị ban đầu để làm cơ sở như sau:

f (2) = a( f (1) + f (1)) = 2a f (1)f (3) = a( f (1) + f (2)) = a(2a + 1) f (1)f (4) = a( f (1) + f (3)) = a( f (1) + a(2a + 1) f (1)) = a(2a2 + a + 1) f (1)

Nếu tiếp tục tính các giá trị sau như f (5), f (6), ... thì sẽ khá phức tạp nhưng nếu quan sát kỹthì ta thấy có thể tính f (4) theo cách khác

f (4) = f (2 + 2) = 2a f (2) = 4a2 f (1)

Đến đây việc còn lại là giải phương trình để tìm a.

91

Page 92: I HÅC QUÈC GIA TP. HCM · 2020. 10. 18. · Ngo i b i vi¸t cõa anh Ph¤m Tu§n Huy tø n«m 2013, c¡c b¤n s³ th§y r§t nhi·u b i vi¸t m t¡c gi£ v¨n cán l håc sinh cõa

Lời giải. Ta sẽ tính f (4) bằng hai cách. Trước hết tính f (2) và f (3):

f (2) = a( f (1) + f (1)) = 2a f (1)f (3) = a( f (2) + f (1)) = a(2a + 1) f (1)f (4) = a( f (1) + f (3)) = 2a f (2)

Từ đây f (1) + f (3) = 2 f (2) hay (2a2 + a + 1) f (1) = 4a f (1), nghĩa là:

f (1)(2a2 − 3a + 1) = 0

Giải phương trình, ta được các nghiệm là a = 1 hoặc a =12

.

Nhưng a ∈ N∗ nên ta nhận a = 1. Đây cũng là giá trị cần tìm do tồn tại hàm fthỏa mãn bài toán khi a = 1, chẳng hạn như f (n) = n.

Nhận xét. Khi giải quyết bài toán ta phát hiện được việc tính bằng hai cách khá thú vị táchra khỏi ý tưởng quy nạp. Đây là một kỹ thuật hiệu quả với các bài toán có tham số. Nếu a là số

thực thì ta vẫn có thể nhận a =12

do lúc đó có thể chọn f là hàm hằng.

Ví dụ 2. Tìm tất cả các số nguyên dương p sao cho tồn tại hàm số f : N∗ → N∗ thỏa mãn

f ( f (n)) = n + p ∀n ∈ N∗

Phân tích. Dự đoán rằng hàm số thoả mãn bài toán nên là một đa thức, Khi đó deg f = 1 vàsau khi đồng nhất hệ số, ta được

f (n) = n +p2

Điều này chỉ có nghĩa khi p là số chẵn, nên ta chỉ còn tìm định hướng cho trường hợp p làsố lẻ, mà thường gặp nhất là chứng minh không tồn tại. Để khai thác được tốt, ta sẽ sử dụngphép thế và các tính chất của ánh xạ.

Lời giải. Từ giả thiết thì f là đơn ánh. Xét phép đặt các tập hợp sau:

A = {k ∈ N∗|@ n ∈ N∗ : f (n) = k}

B = {k ∈ N∗|@ n ∈ N∗ : f ( f (n)) = k}Dễ thấy A ⊂ B và |B| = p. Với phần tử x ∈ B bất kỳ, ta có các khả năng sau

• Nếu x ∈ A thì f (x) ∈ B. Thật vậy, nếu f (x) 6∈ B thì tồn tại n nguyên dương saocho f ( f (n)) = f (x). Bởi tính chất đơn ánh mà f (n) = x, vô lý.• Nếu x 6∈ A thì tồn tại duy nhất số nguyên dương k để f (k) = x. Khi đó k ∈ A

vì nếu không thì có n nguyên dương để f (n) = k hay x = f ( f (n)), vô lý.

92

Page 93: I HÅC QUÈC GIA TP. HCM · 2020. 10. 18. · Ngo i b i vi¸t cõa anh Ph¤m Tu§n Huy tø n«m 2013, c¡c b¤n s³ th§y r§t nhi·u b i vi¸t m t¡c gi£ v¨n cán l håc sinh cõa

Xét ánh xạ g : A → B\A với phép tương ứng x 7→ f (x). Từ những lập luận trên thìdễ thấy g là song ánh, vậy |A| = |B\A| hay |B| = 2|A|, dẫn đến p là số chẵn. Đặtp = 2c. Khi đó dễ thấy hàm số f (n) = n + c thỏa mãn bài toán. Vậy p là số chẵn.

Ví dụ 3. (Vietnam TST 2004) Tìm tất cả các số thực a sao cho tồn tại duy nhất một hàm sốf : R→ R thỏa mãn:

f (x2 + y + f (y)) = f 2(x) + ay ∀x, y ∈ R.

Phân tích. Dự đoán f (x) = x là nghiệm hàm sẽ dẫn đến a = 2 và mong muốn chứng minhđây là hằng số duy nhất. Giả thiết nhắc đến điều kiện duy nhất của nghiệm hàm để loại trườnghợp a = 0, vì lúc ấy có ít nhất 2 nghiệm hàm là f ≡ 0 và f ≡ 1. Vậy ta chỉ cần giải bài toánvới a 6= 0. Đây là phương trình hàm có biến tự do nên đến đây việc giải quyết dựa vào vậndụng khéo léo tính chất ánh xạ và các phép thế biến.

Lời giải. Nếu a = 0 thì có ít nhất hai hàm số thỏa mãn bài toán là f ≡ 0 và f ≡ 1. Dođó a 6= 0. Thay x bởi 0 vào giả thiết, ta được:

f (y + f (y)) = f 2(0) + ay ∀y ∈ R (1)

Do a 6= 0 nên từ đây f là toàn ánh. Thay x bởi −x vào giả thiết:

f 2(x) = f 2(−x) ∀x ∈ R

Do f là toàn ánh nên tồn tại b sao cho f (b) = 0. Từ đây ta cũng thấy được f (−b) = 0.Lần lượt thay y bởi b và −b vào phương trình (1), ta được

f 2(0) + ab = f 2(0)− ab = 0

Do a 6= 0 nên b = 0. Từ đây dẫn đến khẳng định sau:

f (x) = 0⇔ x = 0 (2)

Thay y bởi− f 2(x)

avào giả thiết, ta được:

f(

x2 − f 2(x)a

+ f(− f 2(x)

a

))= 0 ∀x ∈ R

Kết hợp với (2) thì x2 − f 2(x)a

+ f(− f 2(x)

a

)= 0 ∀x ∈ R. Thay y bởi

− f 2(y)a

vào

giả thiết và thu gọn, ta thu được:

f (x2 − y2) = f 2(x)− f 2(y) ∀x, y ∈ R (3).

93

Page 94: I HÅC QUÈC GIA TP. HCM · 2020. 10. 18. · Ngo i b i vi¸t cõa anh Ph¤m Tu§n Huy tø n«m 2013, c¡c b¤n s³ th§y r§t nhi·u b i vi¸t m t¡c gi£ v¨n cán l håc sinh cõa

Thay y bởi 0 thì f (x2) = f 2(x) ∀x ∈ R nên f (x) ≥ 0 ∀x ≥ 0. Hơn nữa, trong (3) khithay x bởi 0 thì ta thu được:

f (−y2) = − f (y2) ∀y ∈ R

Do đó f là hàm lẻ. Bây giờ ta chứng minh rằng f là hàm cộng tính.

• Với x ≥ 0, y ≤ 0, ta có:

f (x + y) = f 2(√

x)− f 2(√−y) = f (x)− f (−y) = f (x) + f (y)

• Với x ≤ 0, y ≥ 0, ta thực hiện tương tự như trên.• Với x ≥ 0, y ≥ 0, ta có:

f (x + y)− f (y) = f 2(√

x + y)− f 2(√

y) = f (x + y− y) = f (x)

Từ đó thì f (x + y) = f (x) + f (y).

• Với x ≤ 0, y ≤ 0, để ý rằng −x ≥ 0,−y ≥ 0 nên:

f (−x) + f (−y) = f (−x− y)

Bởi f là hàm lẻ, ta có − f (x)− f (y) = − f (x + y) hay f (x + y) = f (x) + f (y).

Tóm lại f là hàm cộng tính trên R. Ta chứng minh rằng f đơn điệu trên R.

• Nếu x ≥ 0 ≥ y thì f (x) ≥ 0 ≥ f (y)• Nếu x > y ≥ 0, thay

√x,√

y vào (3), ta được:

f (x)− f (y) = f (x− y) ≥ 0

Bất đẳng thức cuối do x− y ≥ 0. Do đó f (x) ≥ f (y).• Nếu 0 ≥ x > y, khi đó 0 6 −x < −y nên f (−y) ≥ f (−x) hay f (x) ≥ f (y).

Do f cộng tính và đơn điệu trên R nên f (x) = cx ∀x ∈ R. Thay vào giả thiết thử lại

c(x2 + y + cy) = c2x2 + ay ∀x, y ∈ R

Thay x bởi 1 và thay y bởi 0, ta được c = c2. Loại trường hợp c− 0, ta phải có c = 1.Thế ngược lại vào giả thiết, ta thu được 2y = ay ∀y ∈ R hay a = 2. Vậy a = 2 là hằngsố duy nhất thỏa mãn yêu cầu bài toán.

2. SỬ DỤNG DÃY SỐ

Ví dụ 4. Tìm tất cả các số nguyên dương k sao cho tồn tại hàm f : N∗ → N∗ thỏa mãn

f ( f (n)) + f (n) = kn ∀n ∈ N∗

94

Page 95: I HÅC QUÈC GIA TP. HCM · 2020. 10. 18. · Ngo i b i vi¸t cõa anh Ph¤m Tu§n Huy tø n«m 2013, c¡c b¤n s³ th§y r§t nhi·u b i vi¸t m t¡c gi£ v¨n cán l håc sinh cõa

Phân tích. Ý tưởng về dãy số có thể thấy rõ trong bài toán này. Khi có các đại lượngf ( f (n)), f (n) thì việc đặt dãy số bởi u1 = n, un+1 = f (un) là tự nhiên. Sau đó, khi thay un

vào giả thiết, ta sẽ được một quan hệ hồi quy tuyến tính của dãy số, đến đây có thể sử dụngphương trình đặc trưng để xử lí.

Lời giải. Với mỗi n ∈ N∗, xét dãy số bởi u0 = 1, u1 = n, ut+1 = f (ut). Khi đó dễ thấyrằng:

ut+2 + ut+1 = kut ∀t ∈ N∗

Phương trình đặc trưng của quan hệ trên là λ2 + λ− k = 0. Từ đó ta có:

ut = a

(√k +

14− 1

2

)t

+ b

(−√

k +14− 1

2

)t

∀t ∈ N∗

Tiếp theo, ta sẽ chứng minh b = 0 bằng việc tính giới hạn dãy chẵn, lẻ khi b < 0, b > 0

• Nếu b < 0, khi đó:

u2t =

(√k +

14+

12

)2t

b + a

√k +

14− 1

2√k +

14+

12

2t

Khi đó dễ thấy cho t→ ∞ thì lim u2t = −∞, vô lý.• Nếu b > 0, khi đó:

u2t+1 =

(√k +

14+

12

)2t+1

−b + a

√k +

14− 1

2√k +

14+

12

2t+1

Khi đó dễ thấy cho t→ ∞ thì lim u2t+1 = −∞, vô lý.

Vậy b = 0. Do u1 ∈ N∗ nên√

4k + 1− 12

là số nguyên dương, nghĩa là:

√4k + 1− 1 = 2y (y ∈ N∗)

Từ đây dễ dàng giải ra được k = y2 + y với y ∈ N∗. Ngược lại, nếu k = y2 + y vớiy ∈ N∗ thì hàm số f (n) = yn thoả mãn bài toán. Vậy tất cả các số nguyên dương cầntìm có dạng k = y2 + y (y ∈ N∗).

Ví dụ 5. Tìm tất cả các số nguyên dương p sao cho tồn tại hàm số f : N∗ → N∗ thỏa mãn

f(

f (n)n

)= pn2 ∀n ∈ N∗

95

Page 96: I HÅC QUÈC GIA TP. HCM · 2020. 10. 18. · Ngo i b i vi¸t cõa anh Ph¤m Tu§n Huy tø n«m 2013, c¡c b¤n s³ th§y r§t nhi·u b i vi¸t m t¡c gi£ v¨n cán l håc sinh cõa

Phân tích. Trước hết ta cần dự đoán p có thể nhận giá trị nào. Trước hết, ta thử chọn f là đathức và so sánh bậc thì deg f = 2. Nếu f (n) = cn2 thì:

f(

f (n)n

)= c3n2 = pn2

Vậy nếu p là lập phương đúng của một số nguyên dương thì yêu cầu được thoả mãn. Vấn đềnằm ở các trường hợp khác, và ta sẽ tiếp tục sử dụng dãy số. Đặt f (n) = ng(n) để đơn giảnvề hình thức thì g : N∗ → N∗ và:

g(n)g(g(n)) = pn2 ∀n ∈ N∗

Để chuyển thành phép cộng, ta sẽ logarith hoá cả hai vế của đẳng thức trên, hơn nữa do có sốp nên ta chọn cơ số p để việc tính toán trở nên gọn gàng.

Lời giải. Nếu p = 1 thì hàm số f (n) = n2 thoả mãn bài toán. Ta xét p > 1.

Đặt g(n) =f (n)

n. Từ điều kiện bài toán thì g : N∗ → N∗ và:

g(g(n))g(n) = pn2

Với mỗi n ∈ N∗, xét dãy số bởi u0 = 1, u1 = n, ut+1 = g(ut) với t ≥ 1. Khi đó:

ut+2ut+1 = pu2t ∀t ∈ N∗

Đặt logp ut = vt. Do ut ≥ 1 ∀t nên vt ≥ 0 và v0 = 0. Khi đó:

vt+2 + vt+1 = 2vt + 1 ∀t ∈ N∗

Lại đặt wt = vt −t3

thì wt+2 + wt+1 = 2wt. Chú ý rằng từ cách đặt thì w0 = 0. Bâygiờ sử dụng phương trình đặc trưng, ta thu được

wt =1− (−2)t

3w1

Từ đó vt =1− (−2)t

3

(v1 −

13

)+

13

t. Đặt c = v1 −13

. Có các trường hợp sau xảy ra

• Nếu c > 0, thay t bởi 2k, ta được

v2k =1− 4k

3c +

23

k ∀k ∈ N∗

Cho k→ ∞, ta thu được lim v2k = −∞, mà vn ≥ 0 nên vô lí.

96

Page 97: I HÅC QUÈC GIA TP. HCM · 2020. 10. 18. · Ngo i b i vi¸t cõa anh Ph¤m Tu§n Huy tø n«m 2013, c¡c b¤n s³ th§y r§t nhi·u b i vi¸t m t¡c gi£ v¨n cán l håc sinh cõa

• Nếu c < 0, thay t bởi 2k + 1, ta được

v2k+1 =1 + 2.4k

3c +

23

k

Cho k→ +∞, ta thu được lim v2k+1 = −∞, mà vn ≥ 0 nên vô lí.

Vậy c = 0 hay v1 =13

. Khi đó u1 = 3√

p nên p = u31, và ta phải có p là số lập phương

đúng. Ngược lại, dễ thấy p là số lập phương đúng thỏa mãn yêu cầu đề bài khi chọnđược hàm số f (n) = 3

√p.n2. Bài toán kết thúc.

Nhận xét. Nếu thay p là một số lập phương đúng lớn hơn 1 và yêu cầu tìm tất cả hàm sốthỏa mãn thì sử dụng phương pháp dãy số, ta có thể giải bài toán triệt để và thu được hàm sốduy nhất như trên. Tuy nhiên, ta lại không sử dụng được phương pháp ấy cho trường hợpp = 1 vì log1 x không có nghĩa. Việc tìm tất cả hàm số thoả mãn yêu cầu trong trường hợpp = 1 chính là bài toán trong đề thi chọn đội tuyển PTNK năm 2014.

3. SỬ DỤNG Ý TƯỞNG VỀ ĐIỂM BẤT ĐỘNG

Ví dụ 6. Tìm tất cả các số tự nhiên m sao cho tồn tại hàm số f : R→ R thỏa mãn

f ( f (x)) = x2 −m ∀x ∈ R

Phân tích. Nếu so sánh bài toán này với hai bài toán hàm hợp phía trên, ta sẽ thấy ngay sựkhác biệt về độ khó. Trong trường hợp m = 0, ta có hàm số f (x) = |x|

√2 với cách chọn cũng

khá tinh tế. Việc m ≥ 0 là một điều kiện ta cần quan tâm kỹ. Ta sẽ sử dụng ý tưởng về dãy sốđể tính thử f 4 vì đã có f 2 và f 0, với quy ước f k = f ◦ f ◦ . . . ◦ f (hợp k lần). Ta có:

f 4(x) = (x2 −m)2 −m

Điều đặc biệt là (x2 −m)2 − x−m chia hết cho x2 − x−m. Có thể tạm gọi là đại lượngx2 − x−m được bảo toàn qua phép lấy hàm hợp. Ta sẽ tập trung tìm hiểu các nghiệm của đathức này thay vì các nghiệm của hàm số, khác với kỹ thuật xử lý mọi khi.

Lời giải. Với m = 0, ta chọn f (x) = |x|√

2. Chỉ cần xét khả năng m 6= 0.Dễ thấy phương trình x2 − x−m = 0 luôn có 2 nghiệm x1, x2 phân biệt và trái dấu,do đó f ( f (x)) có 2 điểm bất động. Ta gọi các giá trị đó là x1, x2. Thay x bởi f ( f (x)):

f 4(x) = f ( f ( f ( f (x)))) = (x2 −m)2 −m = (x2 − x−m)(x2 + x + 1−m)

Do m ≥ 1 nên phương trình x2 + x + 1−m = 0 có 2 nghiệm phân biệt. Bây giờ ta sẽchứng minh rằng vế phải có 4 nghiệm phân biệt. Thật vậy, giả sử rằng hai phươngtrình x2 − x − m = 0 và x2 + x + 1− m = 0 có nghiệm chung là x0 thì dễ dàng có

x0 = −12

và m =34

. Điều này vô lý do m ≥ 1. Vậy vế phải luôn có 4 nghiệm phân biệt.

97

Page 98: I HÅC QUÈC GIA TP. HCM · 2020. 10. 18. · Ngo i b i vi¸t cõa anh Ph¤m Tu§n Huy tø n«m 2013, c¡c b¤n s³ th§y r§t nhi·u b i vi¸t m t¡c gi£ v¨n cán l håc sinh cõa

Nói cách khác, f 4(x) có 4 điểm bất động phân biệt là x1, x2, x3, x4. Do f ( f (x)) có 2điểm bất động là x1, x2 nên f (x1), f (x2) nhận giá trị trong tập hợp {x1, x2}. Do f 4(x)có 4 điểm bất động là x1, x2, x3, x4 nên kết hợp tính chất trên, ta có được f (x3), f (x4)

nhận 2 giá trị trong tập hợp {x1, x2, x3, x4}. Nếu f (xi) = xj (i ∈ {3; 4}, j ∈ {1; 2}) thìf 3(xi) = xj hay f 4(xi) = f (xj) = xi. Do đó f (x3), f (x4) nhận 2 giá trị trong tập hợp{x3, x4}. Xét các khả năng sau:

• Nếu f (x3) = x3 thì f ( f (x3)) = x3 hay x3 là điểm bất động của f 2(x), vô lý.• Nếu f (x4) = x4, tương tự như trên thì ta cũng có điều vô lý.• Nếu f (x3) = x4, f (x4) = x3, khi đó f ( f (x3)) = x3, vô lý.

Vậy m = 0 là số tự nhiên duy nhất thỏa mãn bài toán.

Nhận xét. Với m ∈ R+, không tồn tại hàm số thoả mãn bài toán khi m >34

. Tuy nhiên, việc

xây dựng hàm số thoả mãn khi m ≤ 34

vẫn chưa được biết.

Ví dụ 7. (Baltic Way 1992) Tìm tất cả L sao cho tồn tại song ánh f : N∗ → N∗ thỏa mãn:

limf (n)

n= L.

Phân tích. Hàm số đầu tiên chúng ta nghĩ đến là f (n) = n, như vậy thì L = 1. Ta biết rằngnếu f là song ánh thì f−1 xác định và cũng là một song ánh. Sử dụng tính chất đó, ta thấy

nếu L là số thực thỏa mãn thì1L

cũng thỏa mãn. Trường hợp L = 0, xem10= +∞). Thật vậy

L = limf (n)

n= lim

f (n)f−1 f ((n))

= limn

f−1(n)(do f (n)→ +∞ khi n→ +∞).

Từ đây limf−1(n)

n=

1L

. Ta có thể nhận thấy 1 là nghiệm dương của phương trình L =1L

nên sẽ định hướng việc chứng minh đây là hằng số duy nhất. Để có thêm thông tin, ta sẽ lập

luận trên các epsilon để xử lí trường hợp L > 1 và xét luôn trường hợp limf (n)

n= +∞.

Lời giải. Ta có hai nhận xét sau:

(i) Nếu f là một song ánh từ N∗ → N∗ thì lim f (n) = +∞. Điều này hiển nhiên dof là song ánh nên sẽ không bị chặn trên, do đó f có thể nhận giá trị lớn tuỳ ý.

(ii) Nếu L 6= 0 thỏa mãn bài toán thì1L

cũng thế. Thật vậy, ta có:

L = limf (n)

n= lim

f (n)f−1(n)

= limn

f−1(n)(do f (n)→ +∞ khi n→ +∞)

Từ đó thì limf−1(n)

n=

1L

.

98

Page 99: I HÅC QUÈC GIA TP. HCM · 2020. 10. 18. · Ngo i b i vi¸t cõa anh Ph¤m Tu§n Huy tø n«m 2013, c¡c b¤n s³ th§y r§t nhi·u b i vi¸t m t¡c gi£ v¨n cán l håc sinh cõa

Trở lại bài toán. Nếu L = 1 thì chọn f (n) = n. Xét L 6= 1. Dễ thấy rằng L ≥ 0.

• Với L > 1, do limf (n)

n= L nên với mọi ε > 0, ∃N sao cho với mọi n ≥ N thì

∣∣∣∣f (n)

n− L

∣∣∣∣ < ε⇒ f (n)n

> L− ε

Chọn 0 < ε ≤ L− 1 thì có N sao chof (n)

n> 1 ∀n ≥ N hay:

f (n) ≥ n + 1 ∀n ≥ N

Khi đó f (x) chỉ nhận giá trị trong tập hợp {1; 2; . . . ; N} với 1 ≤ x ≤ N − 1. Nhưvậy tồn tại một số nguyên dương k sao cho f (n) 6= k với mọi n nguyên dương.Điều này vô lí vì f là song ánh.

• Với 0 < L < 1, nếu L thỏa mãn thì1L> 1 cũng thế, nhưng điều này vô lí

• Với L = 0, ta nhận thấy 0 = limn

f−1(n), nghĩa là lim

f−1(n)n

= +∞. Do đó tồn

tại N sao cho với mọi n ≥ N thì f−1(n) > n. Đến đây giải tương tự như trườnghợp ban đầu cũng dẫn đến điều vô lý.

Vậy L = 1 là hằng số duy nhất thoả mãn. Bài toán kết thúc.

Nhận xét. Ta cũng có thể chứng minh trực tiếp sự vô lý trong trường hợp 0 ≤ L < 1 bằngcác lập luận trên epsilon. Hơn nữa f (n) = n không phải hàm số duy nhất thoả mãn, chẳnghạn ta có thể chọn hàm số bởi f (2n− 1) = 2n và f (2n) = 2n− 1.

4. BÀI TOÁN ĐỀ NGHỊ

Bài toán 1. Tìm tất cả số nguyên dương p sao cho tồn tại hàm số f : N∗ → N∗ thỏa mãn

f ( f (n)) = pn ∀n ∈ N∗

Bài toán 2. Tìm tất cả các số nguyên dương a sao cho tồn tại hàm số f : N∗ → N∗ thỏa mãn

f ( f (m) + f (n)) = a(m + n)∀m, n ∈ N∗

Bài toán 3. (Vietnam 2016) Tìm tất cả các số thực α để tồn tại hàm số f : R→ R thỏa mãn

i. f (1) = 2016ii. f (x + y + f (y)) = f (x) + α ∀x, y ∈ R.

Bài toán 4. Tìm tất cả các số thực k sao cho tồn tại hàm số khác hằng f : R→ R thỏa mãn

f (a)2 + f (b)2 + f (c)2 = k( f (a) f (b) + f (b) f (c) + f (c) f (a))

99

Page 100: I HÅC QUÈC GIA TP. HCM · 2020. 10. 18. · Ngo i b i vi¸t cõa anh Ph¤m Tu§n Huy tø n«m 2013, c¡c b¤n s³ th§y r§t nhi·u b i vi¸t m t¡c gi£ v¨n cán l håc sinh cõa

với mọi số thực a, b, c ∈ R thoả mãn a + b + c = 0.

Bài toán 5. Tìm tất cả các số nguyên dương k sao cho tồn tại hàm số f : Z→ Z thỏa mãn

f (x + y + f (y)) = f (x) + ky ∀x, y ∈ Z

Bài toán 6. Tìm tất cả các số nguyên c sao cho tồn tại hàm số f : N∗ → N∗ thỏa mãn

f ( f (n− 1)) = f (n + 1) + c f (n) ∀n ∈ N∗, n ≥ 2

Bài toán 7. Tìm tất cả các số tự nhiên (a, b) sao cho tồn tại hàm số f : N∗ → N∗ thỏa mãn

f ( f (n)) = an + b ∀n ∈ N∗

Bài toán 8. Tìm tất cả a ∈ N∗ sao cho tồn tại hàm số f : N∗ → N∗ thỏa mãn

i. f (2) = a.ii. f (mn) = f (m). f (n) với mọi m, n nguyên dương.

iii. f là hàm số đơn điệu thật sự.

Bài toán 9. Tìm tất cả các số nguyên dương p sao cho tồn tại hàm số f : Z→ Z thỏa mãn

f ( f (m) + n) + f (m) = f (n) + f (3m) + p ∀m, n ∈ Z

Bài toán 10. Tìm tất cả các số thực a sao cho tồn tại k > 0 và hàm số f : R→ R thỏa mãn:

f (x) + f (y)2

≥ f(

x + y2

)+ k|x− y|a ∀x, y ∈ R

TÀI LIỆU THAM KHẢO1. Titu Andreescu, Iurie Boreico, Functional Equations.2. Trần Nam Dũng, Dương Bửu Lộc, Phương trình hàm trên N.3. Chuyên đề Toán học số 10 của trường Phổ thông Năng khiếu.4. Tạp chí Epsilon, Lời giải và bình luận đề thi VMO 2016.5. Diễn đàn Art of Problem Solving.

100

Page 101: I HÅC QUÈC GIA TP. HCM · 2020. 10. 18. · Ngo i b i vi¸t cõa anh Ph¤m Tu§n Huy tø n«m 2013, c¡c b¤n s³ th§y r§t nhi·u b i vi¸t m t¡c gi£ v¨n cán l håc sinh cõa

VỀ PHƯƠNG PHÁP QUY NẠP TRONG GIẢI TOÁNHÌNH HỌC TỔ HỢP

Võ Kế Huy-Bùi Duy Khang

Lớp chuyên Toán khoá 2017 - 2020

1. DẪN NHẬP

Hình học tổ hợp là một lĩnh vực chuyên xét tính tổ hợp từ các phần tử hình học,thường tập trung giải quyết các bài toán về sự sắp xếp các phần tử hình học và cáctính chất đặc thù của nó, điển hình như tính chất bao, tô màu, phủ và chia cắt hình,hơn nữa tính "hình học" chỉ làm nền. Để giải quyết các bài toán có số số hạng cao, cácmệnh đề có tính tổng quát, ta thường sử dụng phương pháp quy nạp. Để áp dụnghiệu quả phương pháp quy nạp trong giải toán, ta cần phân tích kĩ lưỡng vấn đề, vàchứng minh chặc chẽ các khẳng định được dự đoán nhờ quy nạp, hay nói cách kháclà "nghệ thuật" đề xuất những mệnh đề quy nạp.

2. CÁC BÀI TOÁN ÁP DỤNG

Bài toán 1. Ta sẽ chứng minh phát biểu sau của Euler dựa trên phép quy nạp đầy đủ (tứcxét tất cả các trường hợp có thể) như sau: Đối với đa diện đều tuỳ ý ta có D− C + M = 2,trong đó D là số đỉnh, C là số cạnh, M là số mặt.

Lời giải. Vì số hình đa diện đều là hữu hạn, ta sẽ xét tất cả các trường hợp có thể nhưsau:

Đa diện đều D (số đỉnh) C (số cạnh) M (số mặt)Tứ diện 4 6 4

Lập phương 8 12 6Bát diện 6 12 8

Thập nhị diện 20 30 12Nhị thập diện 12 30 20

101

Page 102: I HÅC QUÈC GIA TP. HCM · 2020. 10. 18. · Ngo i b i vi¸t cõa anh Ph¤m Tu§n Huy tø n«m 2013, c¡c b¤n s³ th§y r§t nhi·u b i vi¸t m t¡c gi£ v¨n cán l håc sinh cõa

Với cả 5 trường hợp trên, ta đều có D− C + M = 2. Hoàn tất phép quy nạp.

Phương pháp quy nạp đầy đủ gần với phương pháp phân tích hơn, khi ta cầnphải chia trường hợp tổng quát thành hữu hạn trường hợp riêng và xét chúng mộtcách riêng lẽ, vì thế có phạm vi áp dụng rất hạn chế trong toán học. Ta sẽ đến với bàitập thứ 2, dùng phương pháp quy nạp tiêu chuẩn, để chứng minh một định lý trungtâm của hình học tổ hợp.

Bài toán 2 (Định lí Helly). Cho n hình lồi (n ≥ 4) thoả mỗi 3 hình có ít nhất 1 điểm chung.Chứng minh rằng tất cả n hình lồi đó đều có ít nhất 1 điểm chung.

Lời giải. Để chứng minh bằng quy nạp, ta thử xét n = 4. Khi đó, đặt 4 tập lồi làM1, M2, M3, M4. Gọi Ai là giao của tất cả các cặp trừ Mi. Ta xét hai trường hợp:

TH1: 1 trong 4 điểm Ai nằm trong tam giác tạo bởi 3 điểm còn lại. Không mất tìnhtổng quát giả sử điểm đó là A4. Nhận xét: vì A1, A2, A3 đều thuộc M4 nên cácđiếm nằm thuộc A1A2A3 cũng thuộc M4.

TH2: A1A2A3A4 tạo thành 1 tứ giác lồi. Gọi O là giao điểm A1A2 và A3A4. Ta sẽchứng minh O thuộc tất cả các điểm trên. Ta có: A1 và A3 thuộc M2 và M4 nênA1A3 cũng thuộc M2 và M4. Tương tự, ta có A2A4 thuộc M1 và M3 nên giaocủa chúng, O, thuộc cả 4 tập lồi đang xét.

Vậy bài toán đúng với n = 4. Ta giả sử đúng n = k. Gọi M′n là giao diểm của Mn

và Mn+1 . Hiển nhiên M′n cũng là 1 tập lồi. Với mọi i khác j ta có Mi, Mj, Mn vàMn+1 có ít nhất 1 điểm chung (vì ta đã xét trong trường hợp n = 4).

Từ đó, theo giả thiết quy nạp ta có điều phải chứng minh.

Ta sẽ đến với bài toán tiếp theo. Một lần nữa, ý tưởng quy nạp ở bài này khá đơn giản,nhưng cần phải xét thật kĩ bước quy nạp.

Bài toán 3. Cho n điểm phân biệt trên mặt phẳng sao cho có ít nhất 3 điểm không thẳnghàng. Chứng minh qua các điểm có thể vẽ được ít nhất n đoạn thẳng. (n >= 3)

Phân tích. Đây là một bài dễ, có thể chứng minh bằng nhiều cách nhưng ở đây ta tập trunggiải bằng phương pháp quy nạp, với ý tưởng quy nạp khá rõ.

102

Page 103: I HÅC QUÈC GIA TP. HCM · 2020. 10. 18. · Ngo i b i vi¸t cõa anh Ph¤m Tu§n Huy tø n«m 2013, c¡c b¤n s³ th§y r§t nhi·u b i vi¸t m t¡c gi£ v¨n cán l håc sinh cõa

Lời giải. Với n = 3, ta có 3 điểm không thẳng hàng tạo thành 1 tam giác, mệnh đềhiển nhiên đúng. Giả sử mệnh đề đúng tới n = k, ta sẽ chứng minh mệnh đề cũngđúng với n = k + 1.Trong k + 1 điểm, giả sử tồn tại k điểm thẳng hàng thì ta xét k đường thẳng nối kđiểm đó với điểm còn lại, cộng thêm đường thẳng qua k điểm, ta có thể tạo ra k + 1đường thẳng.Giả sử ngược lại, nếu không tồn tại k điểm thẳng hàng thì tồn tại ít nhất 2 điểm A, Bthoả AB không đi qua bất kì điểm nào. Xét tập k điểm không chứa A ta có ít nhất kđường thẳng, cộng với AB thì ta có k + 1 đường thẳng.Vậy theo giả thiết quy nạp ta có điều phải chứng minh.

Những bài tập tiếp theo khá thử thách, thử thách ở cả việc đặt ra mệnh đề quynạp và chứng minh bước quy nạp, đòi hỏi bạn đọc nắm vững tư duy quy nạp cùng ýtưởng rõ ràng về phương pháp quy nạp dự định sử dụng.

Bài toán 4 (Liên Xô 1989). Một con ruồi và một con nhện cùng nằm trên một trần nhà hìnhvuông có cạnh 1 mét. Trong 1 giây con nhện có thể di chuyển từ vị trí ban đầu đến trung điểmcủa đoạn thẳng từ nhện đến 1 trong 4 góc trần nhà. Trong thời gian trên, con ruồi không dichuyển. Chứng minh rằng trong vòng 8 giây, con nhện có thể tiếp cận con ruồi trong khoảngcách 1 cm.

Phân tích. Ở bài này ta có thể phát triển ý tưởng theo 2 hướng đi.

• Hướng đi thứ nhất: từ hình ảnh trần nhà hình vuông, cùng với sự độc lập của con nhệnkhi di chuyển theo chiều ngang và chiều dọc của trần nhà gợi cho ta phương pháp sửdụng tọa độ.• Hướng đi thứ hai: ta có nhận xét sau: con nhện di chuyển đến vị trí một nửa quãng

đường từ nó đến góc trần nhà, sau 8 bước cần nhảy vào trong một hình hình tròn bánkính 1cm, có tâm là con ruồi. Mục đích của ta trong 8 bước nhảy của nhện là tìm cáchđưa nhện nhảy vào (C) ((C) là đường tròn bán kính 1cm, tâm là ruồi). Thế nhưng, đểnhện nhảy vào C, 1 giây trước đó nhện phải nằm trong một hình tròn (C1) là phép vịtự với đỉnh là 1 trong 4 đỉnh hình vuông, hệ số vị tự là 2 của hình tròn (C). Nhận xéttrên gợi cho ta ý tưởng giải bài toán, đó là sử dụng phép vị tự.

Lời giải. Dưới đây là hai cách giải cho bài này.Cách 1. Ta sẽ chứng minh một mệnh đề tổng quát hơn như sau. Đặt trần nhà hìnhvuông vào hệ tọa độ, với gốc tọa độ là một góc trần nhà, trục Ox và Oy nằm trên cạnhtrần nhà. Ta đặt chiều dài đơn vị là 1m.

103

Page 104: I HÅC QUÈC GIA TP. HCM · 2020. 10. 18. · Ngo i b i vi¸t cõa anh Ph¤m Tu§n Huy tø n«m 2013, c¡c b¤n s³ th§y r§t nhi·u b i vi¸t m t¡c gi£ v¨n cán l håc sinh cõa

(0, 0) x2

x + 12

(1, 0)

y2

y + 12

(1, 1)(0, 1)

(x, y)

Gọi (x; y) là tọa độ vị trí ban đầu của con nhện. Xét tập hợp các điểm:

Ak =

{x + i

2k ;y + i

2k | i, j ∈ Z, 0 ≤ i, j < 2k}

Tập hợp trên cho ta một mạng lưới các điểm hình vuông, mỗi cạnh có chiều dài 2−k.A0 chỉ gồm 1 điểm (x; y); A1 gồm 4 điểm:

(x2

;y2

),(

x + 12

;y2

),(

x2

;y + 1

2

),(

x + 12

;y + 1

2

)

Khi nhảy từ (x; y), nhện có thể chọn vị trí theo tung độ và hoành độ riêng biệt. Ta sẽchứng minh bằng quy nạp theo k, trong lần nhảy thứ k, nhện có thể đến được bất kìvị trí nào trong Ak.Với k = 0, mệnh đề đúng. Giả sử nhện có thể nhảy đến bất kì vị trí nào trong Ak, sau

k lượt. Lấy vị trí M bất kì M =

(x + i2k+1 ;

y + j2k+1

)thuộc Ak+1. Ta sẽ chỉ ra vị trí N(x1; y1)

thuộc Ak trước đó, để từ đó nhện có thể nhảy đến M.

Lấy x1 =x + i

2k nếu i < 2k và x1 =x + i

2k − 1 nếu i ≥ 2k. Tương tự lấy y1 =y + j

2k nếu

j < 2k và y1 =y + j

2k − 1 nếu j ≥ 2k. Điểm N(x1; y1) với tọa độ trên sẽ thuộc tập hợp

Ak và nhện có thể nhảy từ N đến M.Ta lại có một nhận xét như sau: với mọi điểm P trên trần nhà, tồn tại một điểm trongAk sao cho khoảng cách từ điểm đó đến P không lớn hơn

√2.2−k. Với k = 8, ta có

√2.2−8 <

1100

, hoàn tất việc chứng minh.

104

Page 105: I HÅC QUÈC GIA TP. HCM · 2020. 10. 18. · Ngo i b i vi¸t cõa anh Ph¤m Tu§n Huy tø n«m 2013, c¡c b¤n s³ th§y r§t nhi·u b i vi¸t m t¡c gi£ v¨n cán l håc sinh cõa

A B

D C

WO N

P

M

Qi−1

Qi

Cách 2. Đặt trần nhà thành hình vuông ABCD có tâm O, cạnh có chiều dài 100 đơnvị. Gọi điểm xác định vị trí con ruồi là R, vị trí con nhện lúc đầu là N. Gọi dãyQ8, Q7, Q6, Q5, Q4, Q3, Q2, Q1, Q0 là các điểm được xác định bởi phép vị tự của đỉnhtrước đó, với đỉnh là 1 trong 4 đỉnh ABCD, hệ số vị tự là 2, và Q8 = R. Gọi dãyN0, N1, N2, N3, N4, N5, N6, N7, N8 là vị trí con nhện ở giây thứ 0, 1, 2. . . , 8.Để N8 thuộc (Q8; 1), ta cần có N7 thuộc (Q7; 2) và tương tự. Một phép quy nạp đơngiản ta suy ra điều cần chứng minh tương đương với N0 thuộc (Q0; 256). Ở đây, tahoàn toàn có thể chỉ ra thuật toán để (Q0; 256) bao tất cả các điểm nằm trong hìnhvuông ABCD.Gọi M, N, P, W lần lượt là trung điểm AB, BC, CD, DA. Gọi SA là hình vuông AMOW,SB là hình vuông MONB, SC là hình vuông ONCP, SD là hình vuông WOPD. Ta sẽtìm đỉnh ABCD thích hợp để các điểm Qi luôn nằm trong hình vuông. Thật vậy, giả sửQi thuộc ABCD, ta sẽ chỉ ra tồn tại phép vị tự để Qi−1 cũng thuộc ABCD. Không mấttính tổng quát, giả sử Qi thuộc SA,HA

2 (Qi; 28−i) 7→ (Qi−1; 28−i+1),HA2 (Qi) 7→ Qi−1,

HA2 (SA) 7→ ABCD. Vì Qi ∈ SA =⇒ Qi−1 ∈ ABCD. Hoàn tất mệnh đề quy nạp, ta có

Q0 ∈ ABCD, mà 256 > 10√

2 là khoảng cách lớn nhất của hai điểm bất kì nằm tronghình vuông, suy ra ABCD ∈ (Q0; 256).Ta có N thuộc hình tròn (Q0; 256). Đặt Ki = A nếu Qi ∈ SA. Làm phép đặt tương tựvới SB, SC, SD. Ta xét:

HKi12(N0) 7→ N1, HKi

12(Q0; 256) 7→ (Q1; 128)

Vì N ∈ (Q0; 256) suy ra N1 ∈ (Q1; 128). Làm tương tự, ta có N8 ∈ (Q0; 1). Vị trí ởgiây thứ 8 của nhện cách ruồi ít hơn 1 đơn vị. Vậy tồn tại cách di chuyển sao cho sau8 bước nhảy khoảng cách giữa nhện và ruồi nhỏ hơn 1 cm. Ta có điều phải chứngminh.

105

Page 106: I HÅC QUÈC GIA TP. HCM · 2020. 10. 18. · Ngo i b i vi¸t cõa anh Ph¤m Tu§n Huy tø n«m 2013, c¡c b¤n s³ th§y r§t nhi·u b i vi¸t m t¡c gi£ v¨n cán l håc sinh cõa

Bài toán 5 (IMO Shortlist 2006). Một tam giác đều có cạnh n được gọi là lỗ nếu trong tamgiác này có n tam giác đơn vị bị cắt ra. Một hình kim cương là một hình kim cương với cácgóc 60-120. Chứng minh ta chỉ có thể phủ tam giác lỗ bằng các hình kim cương khi và chỉ khimỗi tam giác đều hướng lên có cạnh là k thì không có quá k lỗ, với 1 ≤ k ≤ n.

Phân tích. Đây là một bài toán khá hay sử dụng quy nạp. Tuy vậy, theo em chướng ngại vậtlớn nhất trong bài toán là việc hiểu được mô hình bài toán cùng hướng tiếp cận. Với nhữngtam giác đơn vị, ta chỉ ra những trường hợp không thoả khi một tam giác đều có cạnh là k cónhiều hơn k lỗ. Việc đục nhiều lỗ của một tam giác sẽ tạo ra những tam giác không thể tiếpcận, hay nói cách khác, không thể phủ được, từ đó suy ra mệnh đề quy nạp đúng.

Lời giải. Gọi các tam giác hướng lên là tam giác loại 1, các tam giác hướng xuống làtam giác loại 2, hình kim cương được tạo bởi 2 tam giác đều ở 2 hàng liền nhau làhình kim cương loại 1, hình kim cương được tạo bởi 2 tam giác kề nhau là hình kimcương loại 2.

Tam giác loại 1 Tam giác loại 2 Kim cương loại 1 Kim cương loại 2

Với tam giác có n cạnh, ta có:

n2 + n2

tam giác loại 1 vàn2 + n

2tam giác loại 2.

Hiển nhiên, 1 hình kim cương 60− 120 được tạo thành bằng việc ghép 1 tam giác mỗiloại. Vì ta chỉ có thể bỏ đi các tam giác loại 1 nên ta phải bỏ đúng:

n2 + n2− n2 + n

2= n tam giác loại 1.

Đây là 1 kết quả khá thú vị, và là phần dễ của bài.Ta có thể dễ dàng chỉ ra 1 trường hợp thoả yêu cầu để bài, ví dụ bỏ n tam giác loại 1 ởđáy tầng cuối của tam giác ban đầu. Khi đó ta có thể phủ tam giác bằng những hìnhkim cương loại 1 với mô hình khá rõ.Bây giờ, ta sẽ chứng minh mệnh đề sau đúng:

Với mỗi tam giác đều có cạnh là k thuộc tam giác đều có cạnh là n, nếu không có quák lỗ thì các tam giác có k− 1 cạnh bên trong nó cũng không có quá k− 1 lỗ.

(Lưu ý: Lỗ được tạo ra khi ta cắt 1 tam giác đơn vị ra khỏi hình.)Với k = n, xét 1 tam giác có cạnh là n− 1 và 1 hình thang có 2 đáy lần lượt là n− 1, nvà có số đo các góc là 120− 60. Không mất tính tổng quát, cho hình thang nghiêng 1góc 120. Khi đó, nếu ta ghép 2 hình lại với nhau ta sẽ được tam giác ban đầu.

106

Page 107: I HÅC QUÈC GIA TP. HCM · 2020. 10. 18. · Ngo i b i vi¸t cõa anh Ph¤m Tu§n Huy tø n«m 2013, c¡c b¤n s³ th§y r§t nhi·u b i vi¸t m t¡c gi£ v¨n cán l håc sinh cõa

Vì tam giác ban đầu, có n cạnh và tam giác có cạnh n− 1 cùng có n lỗ nên mọi điểmtrong hình thang đều được phủ bởi các hình kim cương.Ta xét vị trí của tam giác đợn vị trên cùng và dưới cùng của hình thang. Khi đó, chỉ cóthể lấp hình thang bằng các hình kim cương loại 1. Tuy vậy, cách lấp này sẽ làm xuấthiện 1 tam giác loại 1 có 2 cạnh tiếp xúc với 2 hình kim cương, mặt còn lại tiếp xúcvới đáy của hình thang, hay cũng chính là cạnh của tam giác ban đầu. Tam giác nàykhông thể ghép được với bất kì tam giác loại 2 nào. Suy ra mệnh đề đúng với k = n.Giả sử mệnh đề đúng tới k = u. Ta cần chứng minh mệnh đề đúng với u− 1. Tới đây,ta xét 3 trường hợp:

TH1: Tam giác có cạnh là u− 1 có 2 cạnh thuộc 2 cạnh của tam giác có độ dài bằng n.

Xét tam giác có cạnh là u nằm ở vị trí tương tự. Ta cũng chia tam giác cạnh là unay thành 1 tam giác đều và 1 hình thang như khi xét k = n. Trong hìn thangđó, hiển nhiên tồn tại 1 tam giác loại 1 không thể nối với bất kì tam giác loại 2nào trong hình tam giác đều có cạnh là u. Coi tam giác giác đó là “xấu”.Khi tamở rộng mô hình và xét những hình thang tiếp theo với độ dài các đáy tăng 1thì tam giác xấu này không thể bị loại bỏ mà chỉ đổi chỗ, dũ xét với hình thangloại 1 hay loại 2. Ta lặp lại bước lấy thêm tam giác với cạnh lớn hơn cho tới khita xét xong tam giác có cạnh là n.

Vậy, trường hợp 1 không cho phép phủ tam giác bằng những hình thang.TH2: Tam giác có cạnh là u− 1 có 1 cạnh thuộc cạnh của tam giác có độ dài bằng n.

107

Page 108: I HÅC QUÈC GIA TP. HCM · 2020. 10. 18. · Ngo i b i vi¸t cõa anh Ph¤m Tu§n Huy tø n«m 2013, c¡c b¤n s³ th§y r§t nhi·u b i vi¸t m t¡c gi£ v¨n cán l håc sinh cõa

Từ đáy tam giác có độ dại là u kẻ d1 song song với đáy hình thang. Khi đó d1chia tam giác có độ dài n thành 1 tam giác đều và 1 hình thang. Mô hình bàitoán trở về trường hợp 1.

TH3: Tam giác có cạnh là u− 1 có 0 cạnh thuộc cạnh của tam giác có độ dài bằng n.

Kẻ tương tự trường hợp 2, một đường song song với đáy và một đường songsong với 1 trong 2 cạnh của tam giác. Mô hình bài toán tới đây cũng quay vềtrường hợp 1.

Vậy ta có nhận xét rằng mệnh đề mà ta đưa ra là đúng với mọi 1 ≤ k ≤ n, từ đó suyra được điều phải chứng minh.

Bài toán 6 (IMO 2013). Một tập hợp gồm đúng 4027 điểm trên mặt phẳng được gọi là tậpColombia nếu không có ba điểm nào trong các điểm đó thẳng hàng, đồng thời có 2013 điểmđược tô màu đỏ và 2014 điểm còn lại được tô màu xanh. Mặt phẳng được phân chia thành cácmiền khi ta kẻ một số đường thẳng. Một cách kẻ một số đường thẳng được gọi là cách kẻ tốtđối với tập Colombia cho trước nếu hai điều kiện sau được thoả mãn:

1. Không đường thẳng nào đi qua dù chỉ một điểm thuộc tập hợp đó;2. Không miền nào chứa cả điểm màu đỏ và điểm màu xanh.

Tìm số k nhỏ nhất sao cho với tập Colombia tuỳ ý gồm đúng 4027 điểm, tồn tại một cách kẻ kđường thẳng là cách kẻ tốt.

Phân tích. Dự đoán bước quy nạp: Từ điều kiện không có 3 điểm nào thẳng hàng cho ta ýtưởng về việc xây dựng và xét cạnh đa giác bao. Gọi S là một tập không có 3 điểm nào thẳnghàng được tô hai màu xanh và đỏ, k là số đường thẳng nhỏ nhất chia S thoả đề bài, K là tậphợp các đường thẳng trên. Xét A và B là hai điểm trên cạnh đa giác bao, nếu A và B cùng

108

Page 109: I HÅC QUÈC GIA TP. HCM · 2020. 10. 18. · Ngo i b i vi¸t cõa anh Ph¤m Tu§n Huy tø n«m 2013, c¡c b¤n s³ th§y r§t nhi·u b i vi¸t m t¡c gi£ v¨n cán l håc sinh cõa

màu, nếu cả hai không cùng nằm trên phần diện tích cùng màu được chia bởi các đường thẳngthuộc K, cần thêm 1 đường thẳng song song với AB và chia mặt phẳng thành 2 bờ, 1 bờ chứaA và B, một bờ chứa các điểm thuộc S, nâng tổng số đường thẳng lên k + 1 là tối thiểu đểchia tập S ∪ {A; B} thoả mãn đề bài. Nếu A và B khác màu và chia bởi 1 đường thẳng thuộcK, cần nhiều nhất 1 đường thẳng song song với AB và chia mặt phẳng thành 2 bờ, 1 bờ chứaA và B, một bờ chứa các điểm thuộc S, nâng tổng số đường thẳng lên k + 1 là tối thiểu đểchia tập S ∪ {A; B}. Nếu A và B khác màu và cùng trong 1 vùng, thì A hoặc B khác màuso với các điểm còn lại, và vì A và B là đỉnh của đa giác bao nên tồn tại đường thẳng chia Ahoặc B ra khỏi các điểm cùng nằm trong phần diện tích trên. Ở bước quy nạp trên không cósự phân biệt về số lượng của các điểm màu xanh và màu đỏ. Gọi n là tổng số điểm tạo thànhtập theo đề. Ta dự đoán |K| tăng 1 đơn vị mỗi khi n tăng 2 đơn vị. Điều ta cần hiện nay làmột mệnh đề quy nạp đủ rộng để có thể thực hiện bước quy nạp.

A

B

Đ

X

X

Dự đoán mệnh đề quy nạp: Số 2013 và 2014 hoàn toàn không đóng một vai trò cụ thể nào,ta nghĩ ngay đến việc quy nạp theo n điểm màu đỏ và n + 1 điểm màu xanh. Với n = 0, dễthấy k = 0. Với n = 1, vì 3 điểm không thẳng hàng tạo thành hình tam giác, ta chọn đườngtrung trực ứng với đỉnh có màu đỏ suy ra k = 1. Với n = 2, phân tích một số trường hợp tadự đoán k = 2. Với n bất kì, kiểm tra trường hợp lí tưởng đa giác đều 2n + 1 đỉnh, hai đỉnhkề nhau không đồng thời màu đỏ, tồn tại một đa giác n đỉnh đồng dạng với đa giác n đỉnh đỏ,các cặp cạnh tương ứng song song với nhau và bao n điểm đỏ, không chứa điểm xanh nào.Trong trường hợp trên có thể thấ k = n. Từ đây ta dự đoán k = n với mọi n và tìm cách quynạp theo n. Ta sẽ cố gắng chứng minh bằng quy nạp mệnh đề sau: Cần ít nhất k = n đườngthẳng để chia n điểm đỏ và n + 1 điểm xanh thành các vùng sao cho trong mỗi vùng khôngcùng chứa 2 điểm khác màu. Thế nhưng, áp dụng ý tưởng quy nạp đã phân tích với 2 điểmcùng màu đỏ không có kết quả, vì mệnh đề quy nạp của ta chưa bao gồm trường hợp trên. Đểý các trường hợp cơ bản tổng số điểm s = 2 và s = 3. Với s = 2, 2 điểm cùng màu k = 0, haiđiểm khác màu k = 1, suy ra k = 1. Với s = 3, trường hợp 1 điểm khác màu k = 1, trườnghợp 3 điểm cùng màu k = 0, ta kết luận k = 1 với s = 2, 3, trong mọi cách tô màu khác nhau.Mệnh đề quy nạp không cần phát biểu số lượng hai màu xanh đỏ mà tập trung vào tổng sốđiểm thoả đề bài.

Lời giải. Ta phát biểu mệnh đề quy nạp như sau:Cho n điểm điểm được tô màu xanh hoặc đỏ, trong đó không có 3 điểm nào thẳnghàng. Khi đó, k =

[n2

]là số đường thẳng ít nhất có thể để chia mặt phẳng thành các

vùng sao cho các điểm khác màu không cùng nằm trong 1 vùng. Với n ≤ 2 khẳng

109

Page 110: I HÅC QUÈC GIA TP. HCM · 2020. 10. 18. · Ngo i b i vi¸t cõa anh Ph¤m Tu§n Huy tø n«m 2013, c¡c b¤n s³ th§y r§t nhi·u b i vi¸t m t¡c gi£ v¨n cán l håc sinh cõa

định trên là hiển nhiên. Giả sử n ≥ 3, xét một đường thẳng ` chứa 2 điểm A và B saocho tất cả những điểm màu còn lại cùng nằm về một phía của `. Ta có ` là đườngthẳng chứa một cạnh của đa giác bao của các điểm được đánh màu, và A, B là đỉnhcủa đa giác bao. Loại A, B khỏi các điểm đang xét, theo giả thiết quy nạp, cần

[n2

]− 1

đường thẳng chia các điểm còn lại thoả mãn đề bài. Sau đó, ta xét hai điểm A và B,với các trường hợp như sau:

TH1: Nếu A và B có cùng màu, vẽ đường thẳng song song với ` và chia A, B khỏi cácđiểm còn lại. Khi đó mệnh đề quy nạp được thoả mãn.

TH2: Nếu A và B có màu khác nhau, nhưng được chia bởi một đường thẳng, ta lại vẽmột đường thẳng tương tự như trên, mệnh đề quy nạp được thoả mãn.

TH3: Nếu A và B có màu khác nhau và nằm vào cùng một phần diện tích, theo giảthiết quy nạp, các điểm trong phần diện tích trên cùng màu với A hoặc B. Khôngmất tính tổng quát, giả sử A có màu khác với tất cả các điểm còn lại. Khi đó, tồntại một đường thẳng tách A khỏi các điểm còn lại trong phần diện tích trên, vìA là đỉnh của đa giác bao. Mệnh đề quy nạp được thoả mãn.

Áp dụng với tập Colombia có tổng số điểm là 4027 ta có k = 2013 là số đường thẳngtối thiểu cần có. k = 2013 là đáp án của bài toán trên.

3. BÀI TẬP

Bài tập 1 (Austrian-Polish MO 2000). Cho 27 điểm phân biệt trong mặt phẳng, không có3 điểm nào thẳng hàng với bốn điểm tạo thành một hình vuông, 23 điểm còn lại thuộc hình

vuông đó. Chứng minh tồn tại 3 điểm X, Y, Z phân biệt thoả [XYZ] ≤ 148

.

Bài tập 2 (Moscow 1999). Cho đa giác lồi trên mặt phẳng thoả mỗi cạnh được tô màu phíangoài đa giác (tức chỉ tô một vài đoạn của cạnh, những đoạn còn lại thì không tô). Trong đagiác ta vẽ một số đường chéo được tô màu như trên. Chứng minh một trong những đa giácđược tạo từ một phần của đa giác ban đầu cũng được tô màu các cạnh bên ngoài.

Bài tập 3. Cho 1 đa giác n cạnh với 200n đường chéo sao cho không có 3 đường chéo nàođồng quy. Hỏi các đường chéo chia hình thành bao nhiêu phần?

Bài tập 4. Trong một đa giác lồi n đỉnh (n ≥ 403), có 200n đường chéo được vẽ. Chứngminh một đường trong số đó giao với ít nhất 10000 đường còn lại.

Bài tập 5. Cho đa giác lồi có n đỉnh, n ≥ 4. Ta tách đa giác thành các tam giác có đỉnh làđỉnh đa giác một cách bất kì sao cho không tồn tại 2 tam giác có cùng điểm chung bên trong.Ta tô đen các tam giác có 2 cạnh là cạnh đa giác ban đầu, tô đỏ nếu tam giác chỉ có 1 cạnh làcạnh đa giác và tô trắng nếu không có cạnh nào của tam giác là cạnh đa giác. Chứng minh sốtam giác đen nhiều hơn số tam giác trắng 2 tam giác.

110

Page 111: I HÅC QUÈC GIA TP. HCM · 2020. 10. 18. · Ngo i b i vi¸t cõa anh Ph¤m Tu§n Huy tø n«m 2013, c¡c b¤n s³ th§y r§t nhi·u b i vi¸t m t¡c gi£ v¨n cán l håc sinh cõa

Bài tập 6. Xét một đa giác lồi n đỉnh sao cho không có 3 đường chéo nào của nó đồng quy.Hỏi các đường chéo trên chia đa giác thành bao nhiêu phần?

Bài tập 7 (Vietnam 2007). Cho đa giác 2007 đỉnh. Tìm k min thoả: nếu xét k đỉnh bất kì củađa giác, luôn tồn tại 4 đỉnh tạo thành một tứ giác lồi có 3 cạnh là cạnh đa giác ban đầu.

TÀI LIỆU THAM KHẢO1. Mathematical Induction: A Powerful and Elegant Method of Proof (Titu Andreescu, Vlad Crisan).2. Tổ hợp và Quy nạp ( N.IA.VILENKIN Dịch giả: GS.TSKH. Hà Huy Khoái)3. Hình học tổ hợp (Vũ Hữu Bình).4. IMO shortlist các năm.

111

Page 112: I HÅC QUÈC GIA TP. HCM · 2020. 10. 18. · Ngo i b i vi¸t cõa anh Ph¤m Tu§n Huy tø n«m 2013, c¡c b¤n s³ th§y r§t nhi·u b i vi¸t m t¡c gi£ v¨n cán l håc sinh cõa
Page 113: I HÅC QUÈC GIA TP. HCM · 2020. 10. 18. · Ngo i b i vi¸t cõa anh Ph¤m Tu§n Huy tø n«m 2013, c¡c b¤n s³ th§y r§t nhi·u b i vi¸t m t¡c gi£ v¨n cán l håc sinh cõa

BÀI TOÁN CHIA KẸO EULER

Trần Nguyễn Nam Hưng

Lớp chuyên Toán khoá 2018-2021

GIỚI THIỆU. Trong nhiều bài toán tổ hợp, thường là các bài toán tổ hợp tính toán,đếm, ngoài các phương pháp thông thường như áp dụng quy tắc cộng, quy tắc nhân,nguyên lý bao hàm loại trừ,. . . , việc áp dụng “ Bài toán chia kẹo của Euler” cũng hếtsức cần thiết. Bài toán được đề xuất bởi nhà toán học người Thụy Sĩ Leonhard Euler(1707 – 1783) – một trong những nhà toán học lừng lẫy nhất mọi thời đại và đã trởthành một trong những công cụ không thể thiếu trong việc giải quyết các bài toán tổhợp ngày nay. Vậy trên thực tế bài toán này có nội dung như thế nào, và nó lợi hạinhư thế nào trong các bài toán như vậy?

1. SƠ LƯỢC VỀ NỘI DUNG CƠ BẢN VÀ MỘT VÀI MỞ RỘNGTIÊU BIỂU CỦA BÀI TOÁN CHIA KẸO EULER

1.1. Khái quát về bài toán chia kẹo của Euler

Bài toán chia kẹo của Euler về cơ bản có nội dung như sau

Bài toán 1. Có bao nhiêu cách chia n viên kẹo cho k đứa trẻ (n ≥ k) sao cho đứa trẻ nào cũngcó kẹo ?

Chứng minh. Đáp số cho bài toán này là Ck−1n−1.

Thật vậy, gọi k đứa trẻ là A1, A2, . . . , Ak. Ta xếp n viên kẹo trên một hàng ngang. Xétmột cách đặt k− 1 vách ngăn giữa các viên kẹo bất kì thỏa

• Những viên kẹo nằm bên trái vách ngăn thứ nhất là của A1, nhưng viên kẹonằm bên phải vách ngăn cuối cùng là của Ak.• Những viên kẹo nằm giữa vách ngăn thứ i và vách ngăn thứ i + 1 là của Ai+1

Một cách đặt như vậy tương ứng với duy nhất một cách chia kẹo cho k đứa trẻ cũngnhư một cách chọn ra k− 1 số nguyên dương từ n− 1 số nguyên dương đầu tiên. Dođó một cách chia kẹo cho k đứa trẻ cũng tương ứng với đúng một cách chọn ra k− 1số nguyên dương từ n− 1 số nguyên dương đầu tiên, và như vậy số cách chia kẹocho k đứa trẻ này bằng số cách chọn nói trên và bằng Ck−1

n−1.

113

Page 114: I HÅC QUÈC GIA TP. HCM · 2020. 10. 18. · Ngo i b i vi¸t cõa anh Ph¤m Tu§n Huy tø n«m 2013, c¡c b¤n s³ th§y r§t nhi·u b i vi¸t m t¡c gi£ v¨n cán l håc sinh cõa

1.2. Một vài mở rộng và ứng dụng tiêu biểu của bài toán chia kẹo của Euler

Đầu tiên, ta hãy xét ví dụ sau.

Bài toán 2. Có bao nhiêu bộ các số tự nhiên có thứ tự (x1, x2, . . . , xk) sao cho

x1 + x2 + . . . + xk = n,

với k là số nguyên dương và n là số tự nhiên cho trước ?

Nếu để ý, có thể thấy bài toán chia kẹo của Euler nói trên chính là bài toán tính sốnghiệm nguyên dương của phương trình

x1 + x2 + . . . + xk = n, (với n ≥ k).

Với bài toán 1, ta không những có thể tính số nghiệm nguyên dương của phươngtrình trên mà còn có thể tính số nghiệm tự nhiên của phương trình

x1 + x2 + . . . + xk = n, (với n tự nhiên bất kỳ).

Chứng minh. Thật vậy, nếu xi là số tự nhiên thì xi + 1 là số nguyên dương và ngượclại. Do đó nếu xi là số tự nhiên với mọi 1 ≤ i ≤ k và

x1 + x2 + . . . + xk = n, (1)

thì xi + 1 nguyên dương với mọi 1 ≤ i ≤ k và

(x1 + 1) + (x2 + 1) + . . . + (xk + 1) = n + k, (2)

và ngược lại. Do đó một bộ nghiệm tự nhiên của (1) tương ứng với một và chỉ một bộnghiệm nguyên dương của (2), và như vậy (1) và (2) có số nghiệm như nhau. Theo bàitoán 1, số nghiệm đó là Ck−1

n+k−1. Bài toán 2 được giải quyết hoàn toàn.

Như vậy, từ bài toán 2, chúng ta có thể dễ dàng tính được số nghiệm tự nhiên củaphương trình

x1 + x2 + . . . + xk = n, (với k nguyên dương và n tự nhiên bất kỳ).

Điều này khá thường gặp trong các bài toán tổ hợp đếm. Từ đây, có thể dễ dàng giảiquyết bài toán sau.

Bài toán 3. Có bao nhiêu đa thức đơn khởi, bậc 2018 và có các hệ số đều là số tự nhiên saocho P(1) = 1285 ? (Đa thức đơn khởi, hay còn được gọi là đa thức monic, là đa thức có hệ sốbậc cao nhất bằng 1, trong trường hợp này đa thức P(x) có hệ số bậc 2018 bằng 1)

114

Page 115: I HÅC QUÈC GIA TP. HCM · 2020. 10. 18. · Ngo i b i vi¸t cõa anh Ph¤m Tu§n Huy tø n«m 2013, c¡c b¤n s³ th§y r§t nhi·u b i vi¸t m t¡c gi£ v¨n cán l håc sinh cõa

Chứng minh. Giả sử P(x) = x2018 + a2017x2017 + . . . + a1x + a0, với a2017, a2016, . . . , a0

là các số tự nhiên. Thay x = 1 vào đa thức, ta có

P(1) = 1 + a2017 + a2016 + . . . + a0 = 1285,

suy raa0 + a1 + . . . + a2017 = 1284.

Như vậy, số đa thức P(x) thỏa yêu cầu đề bài bằng số nghiệm tự nhiên của phươngtrình trên, và bằng C2018−1

1284+2018−1, hay C20173301 . Bài toán được giải quyết hoàn toàn.

Tiếp theo, ta hãy xét bài toán sau

Bài toán 4. Để bảo vệ máy tính của mình khỏi sự tấn công của hacker, một lập trình viênmuốn thiết lập một mật khẩu cho máy tính của mình. Mật khẩu này bao gồm tất cả các chữcái trong bảng chữ cái tiếng Anh, mỗi chữ cái xuất hiện đúng 1 lần, hơn nữa 2 nguyên âmkhông đứng cạnh nhau. Hỏi lập trình viên đó có bao nhiêu cách cài đặt mật khẩu ?

Việc cài đặt mật khẩu bao gồm 2 công việc : Chọn các vị trí đặt nguyên âm và sắpxếp các nguyên âm đó, tương tự đối với các phụ âm. Ta đều biết trong bảng chữ cáitiếng Anh có 5 nguyên âm (A, E, I, O, U) và 21 phụ âm. Một mật khẩu thỏa yêu cầuđề bài khi và chỉ khi giữa 2 nguyên âm bất kì có ít nhất một phụ âm. Như vậy, có thểthấy, bài toán này, tính riêng công việc chọn vị trí các nguyên âm (phụ âm) đòi hỏiviệc giữa 2 nguyên âm liên tiếp phải có ít nhất một phụ âm. Để giải quyết vấn đề này,ta sẽ áp dụng “Bài toán chia kẹo của Euler” cho số phụ âm đứng giữa 2 nguyên âmliên tiếp, rồi từ đó tiếp tục tính toán bằng các công cụ thông thường.

Chứng minh. Giả sử các ký tự trong mật khẩu của máy tính là A1, A2, . . . , A26 (theothứ tự từ trái sang phải). Để cài đặt mật khẩu thỏa yêu cầu đề bài, ta phải thực hiện 2công việc:

• Công việc 1: Chọn các vị trí đặt nguyên âm (phụ âm).• Công việc 2 : Sắp xếp các nguyên âm (phụ âm) ứng vào các vị trí đó.

Để thực hiện công việc 1, giả sử 5 nguyên âm được đặt tại Ai, Aj, Ak, Al, Am (i < j <k < l < m). Ta chèn thêm 2 nguyên âm A0 và A27 vào 2 đầu mật khẩu. Ta xét 4 trườnghợp sau.

• i = 1, m = 26.

Khi này giữa A0 và Ai, giữa Am và A27 không có phụ âm nào. Như vậy, 21 phụâm tập trung vào 4 vị trí :giữa Ai và Aj, giữa Aj và Ak, giữa Ak và Al, giữa Alvà Am. Theo đề, mỗi vị trí như vậy phải có ít nhất 1 phụ âm và có tổng cộng 21phụ âm trong 4 vị trí đó.

115

Page 116: I HÅC QUÈC GIA TP. HCM · 2020. 10. 18. · Ngo i b i vi¸t cõa anh Ph¤m Tu§n Huy tø n«m 2013, c¡c b¤n s³ th§y r§t nhi·u b i vi¸t m t¡c gi£ v¨n cán l håc sinh cõa

Gọi x1, x2, x3, x4 là số phụ âm trong mỗi vị trí đó thì xi là số nguyên dương với1 ≤ i ≤ 4 và

x1 + x2 + x3 + x4 = 21.

Theo bài toán chia kẹo của Euler, số bộ (x1, x2, x3, x4) thỏa mãn là C320.

• i = 1, m 6= 26 (tương tự với trường hợp i 6= 1, m = 26).

Khi này giữa A0 và Ai không có phụ âm nhưng giữa Am và A27 có ít nhất 1 phụâm. Tương tự, 21 phụ âm sẽ tập trung vào 5 vị trí, mỗi vị trí ứng với 2 nguyênâm "kẹp” nó, và mỗi vị trí có ít nhất 1 phụ âm. Gọi x1, x2, x3, x4, x5 là số phụ âmtrong mỗi vị trí đó, tương tự thì ta có số bộ (x1, x2, x3, x4, x5) thỏa mãn là C4

20.• i 6= 1, m 6= 26

Tương tự, do 21 phụ âm tập trung tại 6 vị trí, mỗi vị trí ứng với 2 nguyên âm“kẹp” nó, và mỗi vị trí có ít nhất 1 phụ âm nên số cách chọn là C5

20.

Như vậy, tổng số cách chọn vị trí các phụ âm là C320 + 2 C4

20 + C520.

Tiếp theo, với công việc 2, khi đã thực hiện công việc 1, ta chỉ cần có được một hoánvị của các phụ âm và một hoán vị của các nguyên âm tương ứng. Có 21! hoán vị cóthể cho các phụ âm và 5! hoán vị cho các nguyên âm, nên số cách thực hiện công việcnày là 21!× 5!.Do hai công việc trên được thực hiện liên tục nên số cách để cài mật khẩu là

21!× 5!× (C320 + 2 C4

20 + C520).

Bài toán được giải quyết.

Có thể thấy, với “Bài toán chia kẹo của Euler”, chúng ta có thể dễ dàng tính đượcsố các chỉnh hợp, tổ hợp thỏa mãn các điều kiện cho sẵn. Trong các ví dụ trên, cácphần tử trong các chỉnh hợp và tổ hợp nói trên đôi một khác nhau. Vậy khi có 2 trongsố các phần tử đó giống nhau thì liệu rằng “ Bài toán chia kẹo của Euler” có còn làmột công cụ hữu dụng ? Hãy cùng đến với ví dụ đặc biệt sau.

Bài toán 5. Trong một cái hộp có 7 loại kẹo, mỗi loại kẹo có nhiều hơn 40 viên kẹo. Hỏi có baonhiêu cách lấy ra 40 viên kẹo từ hộp ? (hai cách lấy được gọi là khác nhau nếu tồn tại một loạikẹo nào đó mà số kẹo thuộc loại đó được lấy ra trong hai cách là khác nhau).

Chứng minh. Gọi x1, x2, . . . , x7 là số kẹo mỗi loại được lấy ra từ hộp trong mỗi cáchlấy. Theo đề bài, mỗi bộ 7 số tự nhiên (x1, x2, . . . , x7) thỏa mãn điều kiện

x1 + x2 + . . . + x7 = 40

tương ứng với một cách lấy phù hợp và ngược lại. Hơn nữa, do xi ≤ 40 mà trong cónhiều hơn 40 viên kẹo mỗi loại nên không có cách lấy nào trong số các cách lấy nóitrên “phạm quy” ( tức là không đủ kẹo để lấy). Do đó, các cách lấy này đều thỏa mãn

116

Page 117: I HÅC QUÈC GIA TP. HCM · 2020. 10. 18. · Ngo i b i vi¸t cõa anh Ph¤m Tu§n Huy tø n«m 2013, c¡c b¤n s³ th§y r§t nhi·u b i vi¸t m t¡c gi£ v¨n cán l håc sinh cõa

và số cách lấy, theo bài toán 1, bằng C7−140+7−1, hay C6

46. Bài toán được giải quyết hoàntoàn.

Có thể thấy, bài toán chia kẹo của Euler những tưởng chỉ đơn thuần là việc chiakẹo cho trẻ con hóa ra lại có nhiều ứng dụng trong các bài toán đến như vậy. Từ việcthiết lập mật mã, thiết lập đa thức cho đến việc tính số nghiệm tự nhiên của mộtphương trình dạng tổng, thậm chí là việc thành lập các tổ hợp lặp. Trong phần tiếptheo, chúng ta sẽ cùng đến với những “biến thể” của “Bài toán chia kẹo của Euler”, từđó giúp các bạn hiểu rõ các vận dụng phương pháp này vào việc giải toán.

2. MỘT SỐ BIẾN THỂ CỦA BÀI TOÁN CHIA KẸO EULER

2.1. Thêm, bớt để tạo ra các đại lượng mới

Trong phần này, chúng ta sẽ cùng đến với một ví dụ khá thú vị sau.

Bài toán 6. Một trường học có cách phát thưởng cho học sinh khá, giỏi vào cuối năm học kháđặc biệt. Theo đó, trường sẽ phát cho mỗi lớp 200 quyển vở và yêu cầu lớp phải phát thưởngcho 10 học sinh có thứ hạng cao nhất lớp theo quy tắc : Bạn học sinh đứng hạng i trong lớpphải nhận được ít nhất 15˘i quyển vở. Lớp 9A của trường này có đúng 1 bạn đứng hạng i, với1 ≤ i ≤ 10. Hỏi cô giáo chủ nhiệm của lớp có bao nhiêu cách chia 200 quyển vở nói trên cho10 bạn này ?

Chứng minh. Từ đề bài, ta suy ra bạn đứng hạng 1 phải được ít nhất 14 quyển vở,bạn đứng hạng 2 phải được ít nhất 13 quyển vở,...,bạn đứng hạng 10 phải được ít nhất5 quyển vở. Gọi xi là số quyển vở mà bạn đứng hạng i nhận được, với q ≤ i ≤ 10. Khiđó ta có

x1 + x2 + . . . + x10 = 200,

suy ra

(x1− (15− 1))+ (x2− (15− 2))+ . . .+(x10− (15− 10)) = 200− (5+ 6+ . . .+ 14) = 105.

Hơn nữa, các số xi − (15− i) trong mỗi cách chia của cô giáo đều phải là số tự nhiênnên số cách chia chính là số nghiệm tự nhiên của phương trình trên. Do đó, số cáchchia mà cô giáo có thể thực hiện chính là C10−1

105+10−1, hay C9114.

Với bài toán này, nếu chỉ giải theo phương pháp thông thường, rõ ràng việc vậndụng “Bài toán chia kẹo của Euler” vào bài toán là điều gần như không thể. Với việcthêm, bớt tinh tế như bài toán trên (hay nói một cách khác là “cho trước” các em họcsinh lượng vở mà nhà trường yêu cầu cô giáo phải thưởng cho các em rồi sau đó chiavở cho các em một cách tùy ý), có thể thấy bài toán trên đã được đưa về “Bài toán chiakẹo của Euler” dạng bình thường. Phương pháp này cũng được sử dụng rất nhiềutrong các bài toán, mà điển hình là bài 5 trong đề thi VMO 2012 (xem trong phần bàitập tự luyện).

117

Page 118: I HÅC QUÈC GIA TP. HCM · 2020. 10. 18. · Ngo i b i vi¸t cõa anh Ph¤m Tu§n Huy tø n«m 2013, c¡c b¤n s³ th§y r§t nhi·u b i vi¸t m t¡c gi£ v¨n cán l håc sinh cõa

2.2. Sử dụng bài toán chia kẹo của Euler nhiều lần trong cùng một bài toán

Trong các ví dụ trên, “Bài toán chia kẹo của Euler” chỉ được sử dụng một lần.Trong phần này, chúng ta sẽ được “chiêm ngưỡng” một hướng áp dụng mới của “Bàitoán chia kẹo của Euler” vào giải toán, đó là dùng chúng nhiều lần trong cùng mộtbài.

Bài toán 7. Nhằm thỏa mãn niềm mong ước được gặp thần tượng của nhiều fan K-pop (âmnhạc pop Hàn Quốc) tại Việt Nam, một công ty tổ chức sự kiện đã quyết định tổ chức đêmnhạc K-Pop Connection. Đêm nhạc có sự tham gia của 3 nhóm nhạc đình đám hiện nay làMomoland, Blackpink và BTS. Trong đêm nhạc này, Momoland sẽ trình diễn tổng cộng 6 tiếtmục, Blackpink sẽ trình diễn tổng cộng 8 tiết mục và BTS sẽ trình diễn tổng cộng 12 tiết mục.Một lượt diễn của một nhóm nhạc được coi là một chuỗi các tiết mục liên tiếp (có thể có đúng1 tiết mục) của 1 nhóm nhạc nào đó mà tiết mục ngay trước tiết mục đầu tiên của chuỗi đó vàtiết mục ngay sau tiết mục cuối cùng của chuỗi đó không phải do nhóm nhạc đó biểu diễn.Theo đó, trong đêm nhạc này, mỗi nhóm nhạc sẽ có 2 lượt diễn. Theo thứ tự, Blackpink sẽ thựchiện lượt diễn của mình trước, sau đó đến Momoland và cuối cùng đến BTS trong cả 2 lượt.Hỏi có bao nhiêu cách sắp xếp các buổi biểu diễn của các nhóm nhạc nói trên sao cho trong cảhai lượt diễn, số tiết mục Blackpink biểu diễn không ít hơn số tiết mục Momoland biểu diễnvà số tiết mục mà BTS biểu diễn không ít hơn số tiết mục Blackpink biểu diễn?(Hai cách sắp xếp được coi là khác nhau nếu như tồn tại một tiết mục thứ j của buổi diễn màtrong cách sắp xếp này với cách sắp xếp kia được biểu diễn bởi 2 nhóm nhạc khác nhau)

Chứng minh. Gọi A1, A2 là số tiết mục Momoland được biểu diễn trong 2 lượt diễn;B1, B2 là số tiết mục Blackpink được biểu diễn trong 2 lượt diễn; C1, C2 là số tiết mụcBTS được biểu diễn trong 2 lượt diễn. Vì thứ tự thực hiện lượt diễn của mình của cácnhóm nhạc đều đã được định sẵn nên 2 cách sắp xếp được coi là khác nhau khi và chỉkhi tồn tại 1 trong 6 số A1, A2, B1, B2, C1, C2 khác nhau trong cả 2 cách.Theo đề bài, ta có

A1 + A2 = 6, B1 + B2 = 8, C1 + C2 = 12,

vàC1 ≥ B1 ≥ A1, C2 ≥ B2 ≥ A2.

Số cách sắp xếp chính là số bộ có thứ tự (A1, A2, B1, B2, C1, C2) thỏa các yêu cầu trên.Để thu được bộ sắp này, ta sẽ lần lượt chọn A1, A2, sau đó chọn B1, B2 và cuối cùngchọn C1, C2.

• Chọn A1, A2. Theo đề bài thì A1, A2 nguyên dương và

A1 + A2 = 6

nên theo số cách chọn là C2−16−1 hay C1

5 .

118

Page 119: I HÅC QUÈC GIA TP. HCM · 2020. 10. 18. · Ngo i b i vi¸t cõa anh Ph¤m Tu§n Huy tø n«m 2013, c¡c b¤n s³ th§y r§t nhi·u b i vi¸t m t¡c gi£ v¨n cán l håc sinh cõa

• Chọn B1, B2. Theo đề bài ta có B1 ≥ A1 và B2 ≥ A2 nên B1 − A1, B2 − A2 là cácsố tự nhiên và

(B1 − A1) + (B2 − A2) = (B1 + B2)− (A1 + A2) = 8− 6 = 2.

Vì đã chọn A1, A2 nên số cách chọn B1, B2 cũng chính là số cách chọn B1 −A1, B2 − A2 thỏa yêu cầu trên, do đó số cách chọn là C2−1

2+2−1 hay C13 .

• Chọn C1, C2. Tương tư thì C1 − B1, C2 − B2 là các số tự nhiên và

(C1 − B1) + (C2 − B2) = (C1 + C2)− (B1 + B2) = 12− 8 = 4,

nên số cách chọn là C2−14+2−1 hay C1

5 .

Vì 3 công việc này được thực hiện một cách liên tiếp nên số cách thực hiện việc sắpxếp là C1

5 × C13 × C1

5 = 75.

2.3. Áp dụng “Bài toán chia kẹo của Euler” để tính số nghiệm nguyên không âm củabất phương trình dạng x1 + x2 + . . . + xn ≤ k (với n là số nguyên dương và k là sốtự nhiên)

Trong các bài toán trước đó, chúng ta đã được biết đến cách tính số nghiệm nguyêndương, nghiệm nguyên không âm của phương trình có dạng x1 + x2 + . . . + xn = kthông qua “Bài toán chia kẹo của Euler”. Vậy còn với bất phương trình dạng

x1 + x2 + . . . + xn ≤ k,

để tính được số nghiệm tự nhiên (hoặc là số nghiệm nguyên dương) của chúng, tacần phải làm như thế nào ? Hãy cùng đến với ví dụ sau.

Bài toán 8. Bốn anh em An, Bình, Cường, Danh rất thích ăn kẹo. Nhân dịp Tết, mẹ của cáccậu được thưởng rất nhiều quà, trong đó có một phần quà là một bịch kẹo bao gồm 125 viênkẹo. Vì quá thích ăn kẹo nên bốn anh em đã xin mẹ thêm một vài bịch kẹo khác giống như vậynữa, nhưng mẹ nhất quyết không cho vì sợ ăn nhiều kẹo sẽ bị sâu răng, do đó mẹ chỉ cho cáccậu ăn số kẹo trong bịch kia thôi. Thế là họ ngay lập tức chia nhau số kẹo nói trên. Hỏi họ cótổng cộng bao nhiêu cách chia kẹo ? (cho rằng 125 viên kẹo là giống hệt nhau, và anh em họ cóquyền không ăn một số viên kẹo trong số 125 viên kẹo ban đầu, đồng thời cũng có thể xảy ratrường hợp có một người không được chia viên kẹo nào).

Chứng minh. Gọi x1, x2, x3, x4 là số viên kẹo mà mỗi người nhận được. Theo đề bài,ta có x1, x2, x3, x4 là các số tự nhiên và

x1 + x2 + x3 + x4 ≤ 125.

119

Page 120: I HÅC QUÈC GIA TP. HCM · 2020. 10. 18. · Ngo i b i vi¸t cõa anh Ph¤m Tu§n Huy tø n«m 2013, c¡c b¤n s³ th§y r§t nhi·u b i vi¸t m t¡c gi£ v¨n cán l håc sinh cõa

Đặt x5 là số kẹo còn lại trong bịch sau khi họ đã chia kẹo xong, ta có x5 là số tự nhiênvà

x1 + x2 + x3 + x4 + x5 = 125.

Do đó, số cách chia kẹo chính là số nghiệm tự nhiên của phương trình trên (vì mỗibộ nghiệm ứng với một cách chia và ngược lại). Theo bài toán 1, số nghiệm đó làC5−1

125+5−1, hay C4129. Bài toán được giải quyết hoàn toàn.

Có thể thấy, bài toán này đưa về việc tìm số nghiệm tự nhiên của bất phươngtrình

x1 + x2 + . . . + xn ≤ k,

trong trường hợp cụ thể là n = 4 và k = 125. Trong bài toán này, với việc thêm biếnxn+1 có giá trị bằng k− (x1 + x2 + . . . + xn), ta đã đưa được bất phương trình đã chovề phương trình nghiệm tự nhiên

x1 + x2 + . . . + xn + xn+1 = k,

và biến bài toán tìm số nghiệm tự nhiên của bất phương trình nói trên thành “Bàitoán chia kẹo của Euler” dạng bình thường. Việc đặt x5 là số kẹo còn lại trong bịchsau khi họ đã chia kẹo xong trong bài toán trên chính là một cách giải thích “dân dã”cho kết quả của bài toán tìm số nghiệm tự nhiên của bất phương trình dạng này.

Qua các ví dụ trên đây, ta có thể nhận ra vô vàn những ứng dụng thú vị của “Bàitoán chia kẹo của Euler” trong việc giải toán. Để củng cố thêm kiến thức về công cụhữu dụng này, xin mời các bạn hãy cùng đến với phần bài tập tự luyện dưới đây.

3. MỘT SỐ BÀI TẬP TỰ LUYỆN

Bài tập 1. Lớp 10 chuyên Toán của một trường THPT được nhà trường thưởng 100 quyểnvở do có thành tích tốt trong các kỳ thi học sinh giỏi. Nhằm khích lệ tinh thần học tập củalớp, cô giáo đã quyết định thưởng các quyển vở này cho các bạn có thành tích trong các kỳ thihọc sinh giỏi. Theo đó, lớp có 13 bạn có thành tích trong các kỳ thi học sinh giỏi, và bạn đượcthưởng ít tập nhất được thưởng n quyển tập (với n là số nguyên dương). Hỏi cô giáo có baonhiêu cách thưởng tập cho các bạn học sinh sao cho

a) n có giá trị nhỏ nhất.b) n có giá trị lớn nhất.c) n = k với k là số nguyên dương cho trước bất kỳ.d) Giải lại câu a) và b), khi thay 100 bởi x, thay 13 bởi y (với x ≥ y).

Bài tập 2 (P140, Tạp chí Pi, số 1 năm 2018). Có 6 đội bóng đá tham gia một giải thi đấubóng đá học sinh của một trường Trung học phổ thông chuyên. Biết rằng giải đấu kéo dài 45ngày và được tổ chức theo thể thức thi đấu vòng tròn (nghĩa là, hai đội bất kì đều thi đấu vớinhau đúng một trận). Hỏi Ban tổ chức giải có tất cả bao nhiêu phương án sắp xếp lịch thi đấusao cho các điều kiện sau được đồng thời thỏa mãn

120

Page 121: I HÅC QUÈC GIA TP. HCM · 2020. 10. 18. · Ngo i b i vi¸t cõa anh Ph¤m Tu§n Huy tø n«m 2013, c¡c b¤n s³ th§y r§t nhi·u b i vi¸t m t¡c gi£ v¨n cán l håc sinh cõa

i. Ba trận đấu đầu tiên có sự góp mặt của cả 6 đội và diễn ra trong ba ngày liên tiếp, mỗingày một trận,

ii. Kể từ trận đấu thứ ba, giữa hai trận đấu liên tiếp có ít nhất 2 ngày không có trận đấunào.

(Hai phương án sắp xếp được coi là khác nhau nếu tồn tại trận đấu thứ k, với k ≥ 1 sao cho ởtrận đấu đó, hai đội gặp nhau trong phương án này khác hai đội gặp nhau trong phương ánkia hoặc tồn tại k, với k ≥ 3 mà thời gian giữa trận đó và trận thứ k + 1 ở phương án nàykhác phương án kia).

Bài tập 3 (Chọn Đội tuyển Quảng Ngãi 2017 – 2018). Trên một đường thẳng có 20 điểmP1, P2, . . . , P20 được sắp xếp theo thứ tự đó, mỗi điểm sẽ được tô bởi một trong hai màu xanhhoặc đỏ. Hỏi có bao nhiêu cách tô màu để sao cho nếu số các điểm liền kề được tô màu giốngnhau thì luôn là một số lẻ ?

Bài tập 4. Hỏi trong tam giác ABC có tổng cộng bao nhiêu điểm P sao cho từ điểm P ta cóthể kẻ 27 tia gốc P, đôi một phân biệt chia tam giác ABC thành 27 tam giác nhỏ có diện tíchbằng nhau ?

Bài tập 5 (Vietnam 2012). Cho một nhóm gồm 5 cô gái, kí hiệu là G1, G2, G3, G4, G5 và12 chàng trai. Có 17 chiếc ghế được xếp thành một hàng ngang. Người ta xếp nhóm người đãcho ngồi vào các chiếc ghế đó sao cho các điều kiện sau được đồng thời thỏa mãn

i. Mỗi người ngồi đúng 1 ghế.ii. Các cô gái xếp theo đúng thứ tự G1, G2, G3, G4, G5 từ trái qua phải.

iii. Giữa G1, G2 có ít nhất 3 chàng trai.iv. Giữa G4, G5 có ít nhất 1 chàng trai và nhiều nhất là 4 chàng trai.

Hỏi có tất cả bao nhiêu cách xếp như vậy ?(Hai cách xếp được coi là khác nhau nếu tồn tại một chiếc ghế mà người ngồi ở chiếc ghế đótrong hai cách xếp là khác nhau).

Bài tập 6 (Trường Đông Toán học 2013). Cho n ≥ 2 là một số nguyên dương. Xét tập hợpcác đường đi ngắn nhất trên lưới nguyên từ điểm A(0; 0) đến điểm B(n; n). Một đường đinhư thế sẽ tương ứng với dãy gồm n lệnh T (lên trên) và n lệnh P (sang phải). Trong dãy đó,một cặp lệnh (T, P) kề nhau được gọi là một bước chuyển (lưu ý, cặp (P, T) không được gọilà bước chuyển). Ví dụ dãy PTTPTPPT có 2 bước chuyển. Tìm số các đường đi ngắn nhất từA đến B có đúng

a) 1 bước chuyển.b) 2 bước chuyển.

Bài tập 7 (Chọn đội tuyển Sóc Trăng 2017 – 2018). Cho tập hợp A = {1, 2, . . . , 2049}gồm 2049 số nguyên dương đầu tiên. Hỏi có thể chọn được tất cả bao nhiêu tập con B =

{a1, a2, . . . , a9} là tập con gồm 9 phần tử của A thỏa điều kiện |ai − aj| ≥ 5, với mọi1 ≤ i < j ≤ 9 ?

121

Page 122: I HÅC QUÈC GIA TP. HCM · 2020. 10. 18. · Ngo i b i vi¸t cõa anh Ph¤m Tu§n Huy tø n«m 2013, c¡c b¤n s³ th§y r§t nhi·u b i vi¸t m t¡c gi£ v¨n cán l håc sinh cõa

Bài tập 8. Một tập hợp các số nguyên được gọi là tập nghỉ nếu như giữa phần tử nhỏ thứ ivà phần tử nhỏ thứ i + 1 của tập đó có ít nhất i2 số nguyên khác. Tập nghỉ bao gồm các sốnguyên dương thì được gọi là tập nghiêm. Hỏi có bao nhiêu tập nghiêm bao gồm 26 số nguyêndương và các phần tử trong tập hợp này đều không lớn hơn 1984 ?

Bài tập 9. Một số nguyên dương biểu diễn được dưới dạng a0a1 . . . an với a0 6= 0 trong hệthập phân được gọi là lên dốc nếu như a0 ≤ a1 ≤ . . . ≤ an, và được gọi là xuống dốc nếua0 ≥ a1 ≥ . . . ≥ an.

a) Hỏi có bao nhiêu số tự nhiên có 5 chữ số thỏa mãn điều kiện số đó hoặc là số lên dốchoặc là số xuống dốc ?

b) Với mỗi số tự nhiên n, đặt f (n) là số số tự nhiên lên dốc có n chữ số và g(n) là số số tựnhiên xuống dốc có n chữ số. Hãy so sánh f(n) và g(n).

Bài tập 10. Một dãy hữu hạn các số nguyên dương u1, u2, . . . , un được gọi là dãy đòn bẩynếu như dãy đó thỏa mãn đồng thời các điều kiện sau

i. n chia hết cho 3.ii. u1 = 1, un = 100.

iii. u3k−2 < u3k−1 < u3k, với mọi 1 ≤ k ≤ n3 .

Tìm số nguyên dương n lớn nhất sao cho tồn tại một dãy đòn bẩy bao gồm n số nguyên dươngmà

n3−1

∑i=0

u3k+1 = 266,

n3−1

∑i=0

u3k+2 = 355,

n3−1

∑i=0

u3k+3 = 414.

Tính xem với n lớn nhất đó, có bao nhiêu dãy đòn bẩy bao gồm n số hạng.

Bài tập 11 (Vietnam 2014). Cho đa giác đều có 103 cạnh. Tô màu đỏ 79 đỉnh của đa giác vàtô màu xanh các đỉnh còn lại. Gọi A là số cặp đỉnh đỏ kề nhau và B là số cặp đỉnh xanh kềnhau.

a) Tìm tất cả các giá trị có thể nhận được của cặp (A, B).b) Xác định số cách tô màu các đỉnh của đa giác để B = 14. Biết rằng hai cách tô màu

được xem là như nhau nếu chúng có thể nhận được nhau từ một phép quay quanh tâmcủa đường tròn ngoại tiếp đa giác.

Bài tập 12. Tại một văn phòng phẩm nọ, người ta bán rất nhiều loại mặt hàng, trong đó có 7loại mặt hàng quan trọng là bút bi, bút chì, tẩy (gôm), thước, vở 100 trang, vở 200 trang vàcompass. Bạn An đã đến văn phòng phẩm này 3 lần để mua 7 loại mặt hàng nói trên, mỗi loại1 cái. Khá là thú vị là kể từ lần thứ hai An đến trở đi, cậu ta đều phát hiện ra rằng giá của cácmặt hàng đều tăng so với lần trước cậu ta đến, hơn nữa cậu cũng phát hiện ra rằng giá của 7loại mặt hàng nói trên trong cả 3 lần đến đều được tính chẵn theo đơn vị ngàn đồng. Bình biếtcác thông tin trên và cũng biết rằng trong cả 3 lần nói trên, An lần lượt phải trả 31, 37 và 46ngàn đồng cho các loại mặt hàng (do tiền An dùng để mua hàng thực tế là tiền An vay mượn

122

Page 123: I HÅC QUÈC GIA TP. HCM · 2020. 10. 18. · Ngo i b i vi¸t cõa anh Ph¤m Tu§n Huy tø n«m 2013, c¡c b¤n s³ th§y r§t nhi·u b i vi¸t m t¡c gi£ v¨n cán l håc sinh cõa

từ Bình). Hỏi xác suất để Bình đoán đúng được giá của cả 7 mặt hàng trong cả 3 lần An đếnvăn phòng phẩm là bao nhiêu?

Bài tập 13. Trong mặt phẳng tọa độ, một con nhện đang ở gốc tọa độ và cho điểmA(2015; 2017). Trong mỗi bước di chuyển, con nhện chỉ có thể di chuyển sang phải hoặc dichuyển lên trên, hơn nữa nếu trong bước này nó đã di chuyển sang phải thì trong bước sau nóphải di chuyển lên trên và ngược lại. Ngoài ra, con nhện cũng chỉ có thể di chuyển đến cácđiểm nguyên trên mặt phẳng tọa độ. Hỏi con nhện có bao nhiêu cách di chuyển để sau đúng45 bước di chuyển lên trên và 44 bước di chuyển sang phải thì nó đến được điểm C thỏa mãnđiều kiện

−→CA có tọa độ (m, n), với m, n là các số nguyên dương ?

TÀI LIỆU THAM KHẢO1. Art of Problem Solving: http://artofproblemsolving.com

123

Page 124: I HÅC QUÈC GIA TP. HCM · 2020. 10. 18. · Ngo i b i vi¸t cõa anh Ph¤m Tu§n Huy tø n«m 2013, c¡c b¤n s³ th§y r§t nhi·u b i vi¸t m t¡c gi£ v¨n cán l håc sinh cõa
Page 125: I HÅC QUÈC GIA TP. HCM · 2020. 10. 18. · Ngo i b i vi¸t cõa anh Ph¤m Tu§n Huy tø n«m 2013, c¡c b¤n s³ th§y r§t nhi·u b i vi¸t m t¡c gi£ v¨n cán l håc sinh cõa

LỜI GIẢI ĐỀ THI CHỌN ĐỘI DỰ TUYỂN 2017

Ban biên tập

1. ĐỀ BÀI

Bài 1. Cho x, y, z là các số thực dương thoả mãn x + y + z = 1. Chứng minh rằng:

x4

x3 + y2 + z2 +y4

y3 + z2 + x2 +z4

z3 + x2 + y2 ≥17

.

Bài 2. Tìm tất cả các hàm số f : N∗ → N∗ thoả mãn đồng thời các điều kiện:

i. f (mn) = f (m) f (n) ∀m, n ∈ N∗.ii. f (m) + f (n) chia hết cho m + n ∀m, n ∈ N∗.

iii. f (2017) = 20173.

Bài 3. Cho đường tròn (O) và dây cung AB cố định. C là một điểm thay đổi trên cunglớn AB sao cho tam giác ABC nhọn. Gọi I, Ia, Ib lần lượt là tâm đường tròn nội tiếp,tâm đường tròn bàng tiếp góc ∠BAC và ∠ABC của tam giác ABC.

1. Gọi M đối xứng với I qua O. Chứng minh rằng tam giác MIa Ib cân.2. Gọi H, K lần lượt là hình chiếu của Ia, Ib trên OI. Đường thẳng qua H vuông góc

với BIa và đường thẳng qua K vuông góc với AIb cắt nhau tại P. Chứng minhrằng P thuộc một đường cố định khi C thay đổi.

Bài 4. Cho S là tập hợp khác rỗng và A1, A2, ..., Am (m ≥ 2) là m tập con của S. Gọi Tlà tập hợp gồm tất cả các tập hợp Ai4 Aj (1 ≤ i, j ≤ m). Chứng minh rằng |T| ≥ m.

2. LỜI GIẢI

Bài toán 1. Cho x, y, z là các số thực dương thoả mãn x + y + z = 1. Chứng minh rằng:

x4

x3 + y2 + z2 +y4

y3 + z2 + x2 +z4

z3 + x2 + y2 ≥17

Lời giải. Trong các lời giải, ta quy ước ∑ f (a, b, c) = f (a, b, c) + f (b, c, a) + f (c, a, b).

125

Page 126: I HÅC QUÈC GIA TP. HCM · 2020. 10. 18. · Ngo i b i vi¸t cõa anh Ph¤m Tu§n Huy tø n«m 2013, c¡c b¤n s³ th§y r§t nhi·u b i vi¸t m t¡c gi£ v¨n cán l håc sinh cõa

Cách 1. Theo bất đẳng thức Cauchy-Schwarz, ta có:

∑x4

x3 + y2 + z2 ≥(x2 + y2 + z2)2

x3 + y3 + z3 + 2(x2 + y2 + z2)

Do đó ta chỉ cần chứng minh rằng(x2 + y2 + z2)2

x3 + y3 + z3 + 2(x2 + y2 + z2)≥ 1

7hay:

7(x2 + y2 + z2)2 ≥ (x3 + y3 + z3)(x + y + z) + 2(x2 + y2 + z2)(x + y + z)2

Khai triển và thu gọn, điều trên tương đương với:

∑(2(x2 + y2 + z2)− xy)(x− y)2 ≥ 0

Nhưng bất đẳng thức này luôn đúng hay bài toán ban đầu được chứng minh hoàntoàn.Cách 2. Tương tự lời giải thứ nhất, ta sẽ chứng minh rằng:

7(x2 + y2 + z2)2 ≥ (x3 + y3 + z3) + 2(x2 + y2 + z2)

Từ giả thiết, ta dễ dàng suy ra được các đẳng thức sau:

x2 + y2 + z2 = 1− 2(xy + yz + zx)

x3 + y3 + z3 = 1− 3(xy + yz + zx) + 3xyz

Do đó bất đẳng thức trên sau khi thu gọn có thể viết lại được thành

7(1− 2(xy + yz + zx))2 ≥ 3xyz + 3− 7(xy + yz + zx)

Do (xy + yz + zx)2 ≥ 3xyz(x + y + z) = 3xyz nên ta chỉ cần chứng minh rằng

7(1− 2(xy + yz + zx))2 ≥ (xy + yz + zx)2 + 3− 7(xy + yz + zx)

Nhưng điều này tương đương với (3(xy + yz + zx)− 1)(9(xy + yz + zx)− 4) ≥ 0.

Điều này đúng do xy + yz + zx ≤ (x + y + z)2

3=

13

. Bài toán ban đầu được chứngminh.Cách 3. Nhận thấy rằng vai trò của x, y, z là bình đẳng nên không mất tính tổng quát,giả sử rằng x ≥ y ≥ z. Khi đó do x, y, z ≤ 1, ta dễ dàng có các bất đẳng thức sau:

x4 ≥ y4 ≥ z4 và1

x3 + y2 + z2 ≥1

y3 + z2 + x2 ≥1

z3 + x2 + y2

126

Page 127: I HÅC QUÈC GIA TP. HCM · 2020. 10. 18. · Ngo i b i vi¸t cõa anh Ph¤m Tu§n Huy tø n«m 2013, c¡c b¤n s³ th§y r§t nhi·u b i vi¸t m t¡c gi£ v¨n cán l håc sinh cõa

Từ đó theo bất đẳng thức Chebyshev và bất đẳng thức Cauchy-Schwarz, ta có:

∑x4

x3 + y2 + z2 ≥13(x4 + y4 + z4)∑

1x3 + y2 + z2 ≥

3(x4 + y4 + z4)

∑ x3 + 2 ∑ x2

Do đó bây giờ ta chỉ cần chứng minh3(x4 + y4 + z4)

∑ x3 + 2 ∑ x2 ≥17

hay là:

21(x4 + y4 + z4) ≥ x3 + y3 + z3 + 2(x2 + y2 + z2)

Theo bất đẳng thức Cauchy-Schwarz và AM-GM, ta dễ dàng có được những điều sau:

18(x4 + y4 + z4) ≥ 6(x2 + y2 + z2)2 ≥ 2(x2 + y2 + z2)

3(x4 + y4 + z4) ≥ 43(x3 + y3 + z3)− 1

27≥ x3 + y3 + z3

Cộng hai bất đẳng thức trên lại theo vế, bài toán được chứng minh hoàn toàn.

Nhận xét. Lời giải thứ nhất ở trên là của Nguyễn Nguyễn, học sinh lớp 10 Toán trườngPTNK đề xuất, với ý tưởng chính là sử dụng bất đẳng thức Cauchy-Schwarz dạngphân thức, sau đó thuần nhất hoá và đưa về dạng tổng các bình phương. Cũng ýtưởng sử dụng bất đẳng thức Cauchy-Schwarz trong lời giải thứ hai, thầy NguyễnViệt Hùng, GV trường THPT chuyên KHTN, đã có một cách xử lý khá tự nhiên là cốgắng đưa về bất đẳng thức 1 biến theo ab + bc + ca. Vũ Thế Việt, học sinh lớp 10 Toántrường PTNK, tác giả của lời giải thứ ba đánh giá vế trái của bài toán bằng bất đẳngthức Chebyshev, và phần sau của bài toán khá đơn giản như đã trình bày.

Bài toán 2. Tìm tất cả các hàm số f : N∗ → N∗ thoả mãn đồng thời các điều kiện:

i. f (mn) = f (m) f (n) ∀m, n ∈ N∗.ii. f (m) + f (n) chia hết cho m + n ∀m, n ∈ N∗.

iii. f (2017) = 20173.

Lời giải. Chúng ta có hai lời giải cho bài toán này.Cách 1 (Lâm Hữu Phúc). Ta sẽ giải bài toán khi thay 2017 bởi p ∈ N∗ bất kỳ.Từ i., bằng phép quy nạp đơn giản, ta có f (pn) = p3n (n ∈ N∗).

Từ ii., thay n bởi m, ta có f (m)... m (m ∈ N∗).

Đặt f (m) = m · g(m) (m ∈ N∗). Với phép đặt đó thì ta dễ dàng có được các điều sau:

• g(mn) = g(m).g(n) ∀m, n ∈ N∗

• m · g(m) + n · g(n) chia hết cho m + n ∀m, n ∈ N∗

• g(pn) = p2n (n ∈ N∗)

Lại đặt h(m) = g(m)−m2 và thay n bởi pn tại b), ta có m · h(m)... m + pn.

Cố định số nguyên dương m và cho n đủ lớn. ta phải có h(m) = 0 ∀m ∈ N∗.

127

Page 128: I HÅC QUÈC GIA TP. HCM · 2020. 10. 18. · Ngo i b i vi¸t cõa anh Ph¤m Tu§n Huy tø n«m 2013, c¡c b¤n s³ th§y r§t nhi·u b i vi¸t m t¡c gi£ v¨n cán l håc sinh cõa

Từ đó ta có được g(m) = m2 hay f (m) = m3 ∀m ∈ N∗.Thử lại, tất cả các hàm số thoả mãn là f (m) = m3 ∀m ∈ N∗.Cách 2 (Nguyễn Nguyễn). Ta chứng minh được f (pn) = p3n (n ∈ N∗).Từ i., thay n bởi 1, ta tính được f (1) = 1.

Từ ii., ta suy ra f (n) + 20173m ... n + 2017m.

Lại để ý rằng n3 + 20173m ... n + 2017m nên f (n)− n3 ... n + 2017m.Cố định số nguyên dương n và cho m đủ lớn thì f (n)− n3 = 0 hay f (n) = n3.Thử lại, tất cả các hàm số thoả mãn là f (m) = m3 ∀m ∈ N∗

Nhận xét. Lời giải thứ hai ngắn gọn hơn và phát triển bài toán được xa hơn khi tahoàn toàn thay được điều kiện iii bởi số mũ k lẻ bất kỳ.

Bài toán 3. Cho đường tròn (O) và dây cung AB cố định. C là một điểm thay đổi trên cunglớn AB sao cho tam giác ABC nhọn. Gọi I, Ia, Ib lần lượt là tâm đường tròn nội tiếp, tâmđường tròn bàng tiếp góc ∠BAC và ∠ABC của tam giác ABC.

1. Gọi M đối xứng với I qua O. Chứng minh rằng tam giác MIa Ib cân.2. Gọi H, K lần lượt là hình chiếu của Ia, Ib trên OI. Đường thẳng qua H vuông góc với

BIa và đường thẳng qua K vuông góc với AIb cắt nhau tại P. Chứng minh rằng P thuộcmột đường cố định khi C thay đổi.

Lời giải. Lời giải sau đây là của bạn Vũ Thế Việt.

1. Gọi Ic là tâm đường tròn bàng tiếp góc ∠BCA thì (O) chính là đường tròn Eulercủa4Ia Ib Ic. Mặt khác4Ia Ib Ic có O là tâm đường tròn Euler và I là trực tâm nênđối xứng M của I qua O chính là tâm ngoại tiếp4Ia Ib Ic hay4MIb Ic cân tại M.

2. Không mất tính tổng quát, giả sử vị trí tương đối của các điểm như hình vẽ trên.Gọi D, E, F lần lượt là trung điểm BC, CA, AB thì các tứ giác ODBP, ODCQnội tiếp nên ∠DPQ = ∠OBC = ∠OCB = ∠DQP hay tam giác DPQ cân tạiD, dẫn đến ∠PDQ = 180◦ − 2∠DPQ = 180◦ − 2∠OBC = ∠BOC = 2∠BAC.Mặt khác ∠PRQ = 180◦ −∠BAC và D, R khác phía với PQ nên D là tâm ngoạitiếp tam giác PQR. Do DF || AC nên PR ⊥ AC hay P, R đối xứng qua DF,tương tự thì Q, R đối xứng qua DE. Từ đó ∠PFR = 2∠PFD = 2∠POD =

360◦ − 2∠DOQ = 360◦ − 2∠DEQ = ∠REQ hay 4PFR ∼ 4REQ (c.g.c). Vậy∠FRE = ∠PRQ = ∠180◦ −∠BAC = 180◦ −∠FDE nghĩa là R thuộc đườngtròn Euler của tam giác ABC.

Bài toán kết thúc.

Nhận xét. Qua lời giải trên, ta nhận thấy vai trò của điểm I không quan trọng nênđường thẳng OI có thể thay thế bằng đường thẳng bất kỳ đi qua tâm ngoại tiếp. Bàitoán trên không mới mà chỉ là một kết quả kinh điển về cực trực giao: cực trực giao

128

Page 129: I HÅC QUÈC GIA TP. HCM · 2020. 10. 18. · Ngo i b i vi¸t cõa anh Ph¤m Tu§n Huy tø n«m 2013, c¡c b¤n s³ th§y r§t nhi·u b i vi¸t m t¡c gi£ v¨n cán l håc sinh cõa

của đường thẳng đi qua tâm ngoại tiếp thì nằm trên đường tròn Euler của tam giácđó. Tuy nhiên phép chứng minh trên lại rất mới khi chỉ sử dụng các kiến thức đơngiản về tam giác đồng dạng và biến đổi góc. Về khái niệm cực trực giao, có thể thamkhảo thêm trong [2].Về kết quả được nêu ra ở phần sau bài toán, xin được giới thiệu một chứng minhkhác tham khảo từ thầy Trần Quang Hùng, trường THPT chuyên KHTN tại [3]: Gọi llà đường thẳng qua trực tâm H của tam giác ABC và vuông góc với d. Gọi S là điểmanti-Steiner của l. K đối xứng H qua BC thì K thuộc (O). Do S là điểm anti-Steinercủa l nên T đối xứng S qua BC nằm trên l. Từ đó gọi ST cắt (O) tại X khác S thì theotính đối xứng ta có ∠AXS = ∠AKS = ∠KHT do đó AX ‖ HT ⊥ d nên AX đi qua Dhay D là trung điểm AX. Từ đó đường thẳng qua D vuông góc BC đi qua trung điểmG của SH. Tương tự các đường thẳng còn lại qua G và dễ thấy G nằm trên đườngtròn Euler của tam giác ABC �

Bài toán 4. Cho S là tập hợp khác rỗng và A1, A2, ..., Am (m ≥ 2) là m tập con của S. GọiT là tập hợp gồm tất cả các tập hợp Ai4 Aj (1 ≤ i, j ≤ m). Chứng minh rằng |T| ≥ m.

Lời giải. Lời giải sau được đề nghị bởi Nguyễn Nguyễn và đã được chỉnh sửa lại chorõ ràng và gọn gàng hơn.Trước hết ta chứng minh bổ đề sau: "Nếu M4N = M4P thì N = P".Thật vậy, giả sử rằng N 6= P. Không mất tính tổng quát, gọi x là phần tử sao chox ∈ N và x 6∈ P. Có hai khả năng sau xảy ra:

• Nếu x ∈ M, khi đó ta có x 6∈ M\N và x 6∈ N\M, do đó x 6∈ M4N, nhưng lạicó x ∈ M4P nên M4N 6= M4P, trái với giả thiết.• Nếu x 6∈ M, khi đó tương tự ta có x ∈ M4N và x 6∈ M4P, dẫn đến M4N 6=

M4P, trái với giả thiết.

Do đó N = P. Bổ đề được chứng minh. Quay trở lại bài toán ban đầu.Ta sẽ thực hiện chứng minh bằng quy nạp theo m. Với m = 2, dễ thấy rằng T =

{∅, A14A2} hay |T| = 2. Bây giờ giả sử rằng bài toán đã đúng đến m = k (k ∈N∗, k ≥ 2), ta sẽ chứng minh rằng bài toán cũng đúng với m = k + 1. Nếu như vớim = k mà ta có |T| = k + 1 thì bài toán hiển nhiên đúng, nên ta chỉ xét trường hợp|T| = k.Xét dãy tập hợp Bi = Ak+14Ai(1 ≤ i ≤ k + 1). Ta bổ sung vào T các tập hợp Bi. Rõràng |T| là đại lượng không giảm. Giả sử rằng vẫn xảy ra |T| = k, do trong dãy tậphợp vừa thêm có k + 1 tập nên phải tồn tại chỉ số p, q (1 ≤ p < q ≤ k + 1) sao choBp = Bq. Từ đó theo bổ đề dẫn đến Ap = Aq, trái với giả thiết các tập hợp Ai là đôimột phân biệt. Do đó điều đã giả sử là sai, hay là |T| ≥ k + 1. Tóm lại ta luôn chứngminh được |T| ≥ k + 1, quy nạp thành công.Vậy bài toán được giải quyết hoàn toàn.

129

Page 130: I HÅC QUÈC GIA TP. HCM · 2020. 10. 18. · Ngo i b i vi¸t cõa anh Ph¤m Tu§n Huy tø n«m 2013, c¡c b¤n s³ th§y r§t nhi·u b i vi¸t m t¡c gi£ v¨n cán l håc sinh cõa

3. LỜI KẾT

Sau kỳ thi, đội dự tuyển khối 10 gồm 17 học sinh đã được thành lập, và 3 học sinhsau đây đã được chọn để tham dự kỳ thi Olympic truyền thống 30/4 lần thứ XXIIImôn Toán 10:

1. Nguyễn Nguyễn, lớp 10 Toán.2. Vũ Thế Việt, lớp 10 Toán.3. Nguyễn Tiến Hoàng, lớp 10 Toán.

Xin cảm ơn thầy Nguyễn Tăng Vũ đã đọc lại toàn bộ và chỉnh sửa bài viết.

TÀI LIỆU THAM KHẢO1. Đề thi chọn đội dự tuyển PTNK năm 2017: http://diendantoanhoc.net/topic/169274-\T5\djề-thi-chọn-

\T5\djội-dự-tuyển-lớp-10-ptnk-\T5\djhqgtphcm/2. Orthopole - Wolfram MathWorld: http://mathworld.wolfram.com/Orthopole.html.3. VMF Marathon Hình học Olympic - Trang 12: http://diendantoanhoc.net/topic/159439-vmfs-marathon-

hình-học-olympic/page-12

130

Page 131: I HÅC QUÈC GIA TP. HCM · 2020. 10. 18. · Ngo i b i vi¸t cõa anh Ph¤m Tu§n Huy tø n«m 2013, c¡c b¤n s³ th§y r§t nhi·u b i vi¸t m t¡c gi£ v¨n cán l håc sinh cõa

LỜI GIẢI ĐỀ THI CHỌN ĐỘI TUYỂN TOÁNTRƯỜNG PHỔ THÔNG NĂNG KHIẾUNĂM 2016, 2017, 2018

Nguyễn Ngọc Duy, Nguyễn Tăng Vũ,Vương Trung Dũng, Lê Phúc Lữ

1. ĐỀ BÀI

1.1. Đề thi năm 2016

Bài 1. Tìm a để dãy số (un) hội tụ biết u1 = a và

un+1 =

2un − 1 khi un > 0−1 khi − 1 ≤ un ≤ 0u2

n + 4un + 2 khi un < −1∀n ∈ N

Bài 2. Tìm số nguyên dương k nhỏ nhất sao cho bất đẳng thức

xkykzk(x3 + y3 + z3) ≤ 3

với x, y, z là các số dương thoả mãn x + y + z = 3.

Bài 3. Cho hàm số f : N∗ → N∗ thoả mãn điều kiện: f tăng thực sự và f (2n) = 2 f (n) vớimọi số nguyên dương n.

a) Giả sử f (1) = 3 và p là số nguyên tố lớn hơn 3. Chứng minh rằng tồn tại n sao chof (n) chia hết cho p.

b) Cho q là số nguyên tố lẻ. Hãy xây dựng một hàm f thoả mãn các điều kiện của bài toánmà f (n) không chia hết cho q với mọi n.

Bài 4. Cho tam giác ABC có góc ∠BAC tù, AH⊥BC (H thuộc BC). Điểm M thay đổi trêncạnh AB. Dựng điểm N sao cho ∆BMN ∼ ∆HCA (H và N khác phía đối với AB).

a) Gọi CM cắt đường tròn ngoại tiếp tam giác BMN tại điểm K (K 6= M). Chứng minhrằng đường thẳng NK luôn đi qua một điểm cố định.

131

Page 132: I HÅC QUÈC GIA TP. HCM · 2020. 10. 18. · Ngo i b i vi¸t cõa anh Ph¤m Tu§n Huy tø n«m 2013, c¡c b¤n s³ th§y r§t nhi·u b i vi¸t m t¡c gi£ v¨n cán l håc sinh cõa

b) Gọi NH cắt AC tại điểm P. Dựng Q sao cho ∆HPQ ∼ ∆HNM (Q và M khác phíađối với PN). Chứng minh rằng Q thuộc một đường thẳng cố định.

Bài 5. Với mỗi số nguyên dương n, ta biết rằng tồn tại duy nhất số tự nhiên a thoả mãna2 ≤ n < (a + 1)2. Đặt ∆n = n− a2.

a) Tìm giá trị nhỏ nhất của ∆n khi n thay đổi và thoả mãn n = 15m2 với m ∈ N∗.b) Cho p, q là các số nguyên dương và d = 5(4p + 3)q2. Chứng minh rằng ∆d ≥ 5.

Bài 6. Với các số nguyên a, b, c, d thoả mãn 1 ≤ a < b < c < d; kí hiệu

T(a, b, c, d) = {{x, y, z, t}|1 ≤ x < y < z < t; x ≤ a, y ≤ b, z ≤ c, t ≤ d}

a) Tính số phần tử của T(1, 4, 6, 7).b) Cho a = 1, b ≥ 4. Gọi d1 là số phần tử của T(a, b, c, d) chứa 1 và không chứa 2; d2 là

số phần tử chứa 1, 2 và không chứa 3; d3 là số phần tử chứa 1, 2, 3 và không chứa 4.Chứng minh rằng d1 ≥ 2d2 − d3. Đẳng thức xảy ra khi nào?

Bài 7. Trong một hệ thống máy tính, một máy tính bất kì có kết nối trực tiếp với ít nhất 30%máy tính khác của hệ thống. Hệ thống này có một chương trình cảnh báo và ngăn chặn khátốt, do đó khi một máy tính bị virus, nó chỉ có đủ thời gian lây cho các máy tính được kết nốitrực tiếp với nó. Chứng minh rằng dù vậy, kẻ tấn công vẫn có thể chọn hay máy tính của hệthống mà nếu thả virus vào hai máy đó, ít nhất 50% máy tính của hệ thống bị nhiễm virus.

Bài 8. Cho tam giác ABC nhọn. Đường tròn (I) có tâm I thuộc cạnh BC và tiếp xúc với cáccạnh AB, AC lần lượt tại E, F. Lấy M, N bên trong tứ giác BCEF sao cho EFNM nội tiếp(I) và các đường thẳng MN, EF, BC đồng quy. MF cắt NE tại P, AP cắt BC tại D.

a) Chứng minh rằng A, D, E, F cùng thuộc một đường tròn.b) Lấy trên các đường thẳng BN, CM cắt điểm H, K sao cho ∠ACH = ∠ABK = 90o.

Gọi T là trung điểm HK. Chứng minh rằng TB = TC.

1.2. Đề thi năm 2017

Bài 1. Cho dãy số (un) thoả mãn un > 0, un > un+1 ∀n ∈ N∗ và dãy (sn) hội tụ với

sn =n

∑i=1

ui .

a) Chứng minh rằng lim(nun) = 0.

b) Đặt bn =1

un+1− 1

un, ∀n ∈ N∗. Chứng minh rằng dãy số (bn) không bị chặn.

Bài 2. Gọi S là tập con của {1, 2, . . . , 2017} sao cho S không chứa hai phần tử mà phần tửnày chia hết cho phần tử kia và cũng không chứa hai phần tử nguyên tố cùng nhau. Hỏi Schứa nhiều nhất bao nhiêu phần tử?

132

Page 133: I HÅC QUÈC GIA TP. HCM · 2020. 10. 18. · Ngo i b i vi¸t cõa anh Ph¤m Tu§n Huy tø n«m 2013, c¡c b¤n s³ th§y r§t nhi·u b i vi¸t m t¡c gi£ v¨n cán l håc sinh cõa

Bài 3. Cho n là số tự nhiên lớn hơn 2 và tập hợp X = {1, 2, 3, . . . , n}. Với mỗi song ánhf : X → X, gọi A f là tập hợp tất cả các bộ số (i; j) sao cho i < j và f (i) > f (j).

a) Có bao nhiêu song ánh f thoả mãn |A f | = 1?b) Giả sử f là một song ánh mà |A f | = k > 0. Chứng minh rằng tồn tại một song ánh

g : X → X sao cho |Ag| = k− 1 vàn∑

i=1| f (i)− i| >

n∑

i=1|g (i)− i|.

Ở đây |X| chỉ số phần tử của tập hợp X.

Bài 4. Cho tam giác ABC nhọn nội tiếp (O) và điểm D di động trên cung BC chứa điểm A(D 6= A). Trên AB, AC lần lượt lấy các điểm M, N sao cho MD = MB, NC = ND.

a) Chứng minh đường cao DH trong tam giác DMN luôn đi qua một điểm cố định.b) DM, DN theo thứ tự cắt (O) tại E, F ( E, F khác D). Chứng minh các đường tròn

ngoại tiếp các tam giác EMB, FNC cắt nhau tại điểm K thuộc đường thẳng BC vàđường cao KI của tam giác KMN luôn qua một điểm cố định.

Bài 5. Với mỗi số nguyên n /∈ {−1, 0, 1}, ký hiệu p(n) là ước nguyên tố lớn nhất của n. GọiF là tập hợp tất cả các đa thức f (x) có hệ số nguyên và thoả mãn

f (n + p(n)) = n + p( f (n))

với mọi số nguyên n > 2017 và f (n) /∈ {−1, 0, 1}.

a) Tìm tất cả các đa thức bậc nhất thuộc F.b) F có bao nhiêu phần tử ?

Bài 6. Với mỗi số tự nhiên n, ký hiệu T(1 + n, 3 + n, 4 + n) là tập hợp tất cả các bộ (a, b, c)với a, b, c là các số tự nhiên thoả mãn:

1 ≤ a ≤ 1 + n, a + 1 ≤ b ≤ 3 + n, b + 1 ≤ c ≤ 4 + n

Gọi an là số phần tử của T(1 + n, 3 + n, 4 + n).

a) Tính a4.b) Tìm tất cả n sao cho an chia hết cho 3.

Bài 7. An và Bình luân phiên đánh dấu các ô vuông của hình vuông 101× 101 ô. An làngười bắt đầu. Một ô sẽ không thể được đánh dấu nếu trên cùng hàng với nó hoặc cùng cộtvới nó đã có ít nhất 2 ô được đánh dấu. Ai không đi được nữa sẽ thua. Hãy xác định ai làngười có chiến thuật thắng.

Bài 8. Cho đường tròn (O) nội tiếp tứ giác ABCD và lần lượt tiếp xúc với các cạnhAB, BC, CD, DA tại E, F, G, H. Gọi I, J là trung điểm của AC, BD và gọi IB, ID, JA, JCtheo thứ tự cắt EF, GH, HE, FG tại M, N, P, Q.

133

Page 134: I HÅC QUÈC GIA TP. HCM · 2020. 10. 18. · Ngo i b i vi¸t cõa anh Ph¤m Tu§n Huy tø n«m 2013, c¡c b¤n s³ th§y r§t nhi·u b i vi¸t m t¡c gi£ v¨n cán l håc sinh cõa

a) Chứng minh rằng I J, MN, PQ đồng quy. Gọi điểm đồng quy là S.b) Các tia đối của các tia JA, IB, JC, ID lần lượt cắt (O) tại các điểm A′, B′, C′, D′. Giả

sử A′C′, B′D′ lần lượt cắt PQ, MN tại U, V. Gọi K là hình chiếu của S trên UV.Chứng minh rằng ∠AKB = ∠CKD.

1.3. Đề thi năm 2018

Bài 1. Cho số nguyên dương a > 1. Tìm giá trị lớn nhất của số thực d sao cho tồn tại mộtcấp số cộng có công sai d và số hạng đầu là a và có đúng hai số trong các số a2, a3, a4, a5 làcác số hạng của cấp số cộng đó.

Bài 2. Cho n số thực x1, x2, ..., xn. Với mỗi i ∈ {1, 2, 3, . . . , n}, gọi ai là số các chỉ số j saocho

∣∣xi − xj∣∣ ≤ 1 và bi là số các chỉ số j sao cho

∣∣xi − xj∣∣ ≤ 2 (i, j có thể bằng nhau).

a) Chứng minh rằng tồn tại i mà bi ≤ 3ai.b) Gọi A là số cặp số (i, j) có thứ tự mà

∣∣xi − xj∣∣ ≤ 1 và B là số cặp số (i, j) có thứ tự mà∣∣xi − xj

∣∣ ≤ 2 (các số i, j có thể bằng nhau). Chứng minh rằng B ≤ 3A.

Bài 3. Cho p là số tự nhiên. Xét phương trình x3 + x + p = y2 trên tập hợp các số nguyêndương.

a) Tìm số nguyên tố p nhỏ nhất dạng 4k + 1 sao cho phương trình có nghiệm.b) Chứng minh rằng nếu p là số chính phương thì phương trình luôn có nghiệm.

Bài 4. Cho tam giác ABC nhọn nội tiếp trong đường tròn (O) có B, C cố định và A di độngtrên (O). Gọi D là trung điểm BC. Trên AB lấy điểm M, P và trên AC lấy các điểm N, Qsao cho DA = DP = DQ, DM⊥AC, DN⊥AB.

a) Chứng minh rằng các điểm M, N, P, Q cùng thuộc một đường tròn, đặt là (C) vàđường tròn này luôn đi qua một điểm cố định.

b) Chứng minh rằng tâm của (C) luôn di chuyển trên một đường tròn cố định.

Bài 5. Cho số thực a 6= 0 và dãy số (un) thoả mãn:

{u1 = 0,

un+1(un + a) = a + 1 ∀n ∈ Z+

Tìm giới hạn của dãy số (un).

Bài 6. Tìm tất cả các hàm số f : R+ → R+ thoả mãn điều kiện

f(

x f (y2)− y f (x2))= (y− x) f (xy) ∀ y > x > 0.

134

Page 135: I HÅC QUÈC GIA TP. HCM · 2020. 10. 18. · Ngo i b i vi¸t cõa anh Ph¤m Tu§n Huy tø n«m 2013, c¡c b¤n s³ th§y r§t nhi·u b i vi¸t m t¡c gi£ v¨n cán l håc sinh cõa

Bài 7. Cho n = 2018.2019. Gọi A là tập hợp tất cả các bộ số (a1; a2; . . . ; an−1; an) có tính

thứ tự sao cho ai ∈ {0; 1} ∀i = 1, 2, ..., n vàn

∑i=1

ai = 20182. Có bao nhiêu bộ số như thế để

thoả mãnk

∑i=1

ai ≥k2

vàk

∑i=1

an−k+1 ≥k2

với k = 1, 2, ..., n?

Bài 8. Cho đường tròn (C) có tâm I nội tiếp tam giác ABC và tiếp xúc với các cạnh AB, ACtại E, F. Gọi AM, AN là phân giác trong và ngoài của tam giác với M, N ∈ BC. Gọi dM, dN

lần lượt là tiếp tuyến khác BC của (C) và đi qua M, N.

a) Chứng minh rằng dM, dN và EF đồng quy. Gọi điểm đồng quy đó là D.b) Trên AB, AC lần lượt lấy P, Q thoả mãn DP ‖ AC, DQ ‖ AB. Gọi R, S lần lượt là

trung điểm DE, DF. Chứng minh rằng I thuộc đường thẳng qua trực tâm của các tamgiác DPS, DQR.

2. LỜI GIẢI

2.1. Đề thi năm 2016

Bài 1. Tìm a để dãy số (un) hội tụ biết u1 = a và

un+1 =

2un − 1 khi un > 0−1 khi − 1 ≤ un ≤ 0u2

n + 4un + 2 khi un < −1∀n ∈ N

Lời giải. Ta xét các trường hợp sau:

a) Nếu a > 1 thì u2 = 2u1− 1 > 1, từ đó un > 1 với mọi n. Hơn nữa tìm được côngthức tổng quát un = 2n−1(a− 1) + 1. Dãy số (un) không hội tụ.

b) Nếu a = 1 thì u2 = 1. Dễ dàng có được un = 1, ∀n. Vậy (un) hội tụ.c) Nếu 0 < a < 1, ta chứng minh rằng trong dãy có ít nhất một số hạng âm.

Thật vậy, giả sử rằng un > 0 ∀n ∈ N∗. Dễ dàng có được 0 < un < 1 và:

un = 2n−1(a− 1) + 1

Cho n → ∞ thì ta có un → −∞, vô lý. Do đó tồn tại n sao cho un < 0. Gọik là chỉ số nhỏ nhất để thoả mãn uk < 0 thì k ≥ 2 và 0 < uk−1 < 1, khi đó−1 < uk = 2uk−1 − 1 < 0 và uk+1 = −1 hay un = −1 ∀n ≥ k. Vậy (un) hội tụ.

d) Nếu −1 ≤ a ≤ 0 thì u2 = −1, từ đó un = −1 ∀n > 1. Dãy số (un) hội tụ.e) Nếu a < −1, ta có u2 = a2 + 4a + 2. Do các trường hợp đã xét, nếu u2 > 1 hay

nói cách khác là a < −2−√

3 thì dãy số không hội tụ. Ta chứng minh rằng (un)

hội tụ khi −2−√

3 ≤ a < −1.

135

Page 136: I HÅC QUÈC GIA TP. HCM · 2020. 10. 18. · Ngo i b i vi¸t cõa anh Ph¤m Tu§n Huy tø n«m 2013, c¡c b¤n s³ th§y r§t nhi·u b i vi¸t m t¡c gi£ v¨n cán l håc sinh cõa

• Nếu−2 < a < −1, ta có u2− u1 = (a+ 2)(a+ 1) < 0, suy ra u2 < u1 < −1và u2 + 2 = (a + 2)2 > 0 nên u2 > −2. Do đó −2 < u2 < −1. Bằng quynạp, ta chứng minh được (un) giảm và −2 < un < −1 nên (un) hội tụ.

• Nếu −2−√

3 ≤ a ≤ −2 thì u2 + 2 = (a + 2)2, suy ra −1 ≤ u2 ≤ 1. Theocác trường hợp dã xét thì (un) hội tụ.

Vậy dãy số (un) hội tụ khi và chỉ khi −2−√

3 ≤ a ≤ 1.

Bài 2. Tìm số nguyên dương k nhỏ nhất sao cho bất đẳng thức

xkykzk(x3 + y3 + z3) ≤ 3

với x, y, z là các số dương thoả mãn x + y + z = 3.

Lời giải. Với k = 1, ta chọn bộ số(

12

,12

, 2)

thì bất đẳng thức sai.

Với k = 2 thì bộ số(

45

,45

,75

)cũng làm bất đẳng thức sai.

Ta chứng minh rằng với k = 3 thì bất đẳng thức đúng hay

x3y3z3(x3 + y3 + z3) ≤ 3 (1)

Không mất tính tổng quát, giả sử z nhỏ nhất. Khi đó z ≤ 1.Ta có x3 + y3 = (x + y)3 − 3xy(x + y) = (3− z)3 − 3xy(x + y) và

(1)⇔ (3− z)3 + z3 ≤ 3x3y3z3 + 3xy(x + y)⇔ 3z2 − 9z + 9 ≤ 1

x3y3z3 + x2y + xy2

Lại có1

x3y3z3 + x2y + xy2 ≥ 3 3

√x3y3

x3y3z3 =3z

Do đó ta chỉ cần chứng minh rằng:

3z2 − 9z + 9 ≤ 3z⇔ 3(z− 1)3 ≤ 0 (đúng)

Vậy (1) đúng hay k = 3 là số nguyên dương nhỏ nhất thoả mãn đề bài.

Bài 3. Cho hàm số f : N∗ → N∗ thoả mãn điều kiện: f tăng thực sự và f (2n) = 2 f (n) vớimọi số nguyên dương n.

a) Giả sử f (1) = 3 và p là số nguyên tố lớn hơn 3. Chứng minh rằng tồn tại n sao chof (n) chia hết cho p.

b) Cho q là số nguyên tố lẻ. Hãy xây dựng một hàm f thoả mãn các điều kiện của bài toánmà f (n) không chia hết cho q với mọi n.

136

Page 137: I HÅC QUÈC GIA TP. HCM · 2020. 10. 18. · Ngo i b i vi¸t cõa anh Ph¤m Tu§n Huy tø n«m 2013, c¡c b¤n s³ th§y r§t nhi·u b i vi¸t m t¡c gi£ v¨n cán l håc sinh cõa

Lời giải.

a) Đặt A = { f (n + 1)− f (n)|n ∈ N∗}. Vì f là tăng thực sự nên A là tập con củatập hợp các số nguyên dương. Khi đó tồn tại phần tử nhỏ nhất của A, ta đặt làk. Khi đó tồn tại số nguyên dương n thoả mãn k = f (n + 1)− f (n). Khi đó:

f (2n + 2)− f (2n) = 2 f (n + 1)− 2 f (n) = 2k

Mặt khác ta có:

f (2n) < f (2n + 1) < f (2n + 2)

f (2n + 1)− f (2n) ≥ k

f (2n + 2)− f (2n + 1) ≥ k

Do đó ta có f (2n + 1)− f (2n) = k hay f (2n + 1) = f (2n) + k.

Bằng quy nạp, ta chứng minh được:

f (2in) = 2i f (n), f (2in + 1) = 2i f (n) + k, ..., f (2in + i) = 2i f (n) + ik.

Do f (1) = 3, f (2) = 6 nên k ≤ 3 hay (k, p) = 1. Khi đó các số

f (2pn), f (2pn + 1), f (2pn + 2), ... f (2pn + p− 1)

sẽ tạo thành một hệ thặng dư đầy đủ modulo p nên sẽ có một số chia hết cho p.b) Ta xây dựng hàm số f như sau:

f (1) = 2a > q

f (2n) = 2 f (n) ∀n ∈ N∗

f (2n + 1) = f (2n) + q ∀n ∈ N∗

Ta chứng minh f là hàm số thoả mãn đề bài.

i. f là hàm tăng thực sự. Ta chứng minh f (n + 1)− f (n) ≥ q bằng quy nạp.Với n = 1, ta có f (2)− f (1) = 2.2a − 2a = 2a > q. Giả sử điều này đúngđến k. Ta xét f (k + 1).

• Nếu k chẵn, ta có f (k + 1) = f (k) + q, thoả mãn yêu cầu.• Nếu k lẻ, ta có:

f (k + 1) = 2 f(

k + 12

)≥ 2

(f(

k− 12

)+ q)= f (k− 1) + 2q.

Vậy ta có điều cần chứng minh.ii. Ta chứng minh không tồn tại n thoả mãn f (n) chia hết cho q bằng quy nạp.

• Ta có f (1) = 2a không chia hết cho q.

137

Page 138: I HÅC QUÈC GIA TP. HCM · 2020. 10. 18. · Ngo i b i vi¸t cõa anh Ph¤m Tu§n Huy tø n«m 2013, c¡c b¤n s³ th§y r§t nhi·u b i vi¸t m t¡c gi£ v¨n cán l håc sinh cõa

• Giả sử điều này đúng đến k.

– Nếu k chẵn thì f (k + 1) = f (k) + q không chia hết cho q.

– Nếu k lẻ thì f (k + 1) = 2 f(

k + 12

)không chia hết cho q.

Vậy f (n) không chia hết cho q với mọi số nguyên dương n.

Tóm lại hàm số f được xây dựng như trên thoả mãn đề bài. Bài toán kết thúc.

Bài toán kết thúc.

Bài 4. Cho tam giác ABC có góc ∠BAC tù, AH⊥BC (H thuộc BC). Điểm M thay đổi trêncạnh AB. Dựng điểm N sao cho ∆BMN ∼ ∆HCA (H và N khác phía đối với AB).

a) Gọi CM cắt đường tròn ngoại tiếp tam giác BMN tại điểm K (K 6= M). Chứng minhrằng đường thẳng NK luôn đi qua một điểm cố định.

b) Gọi NH cắt AC tại điểm P. Dựng Q sao cho ∆HPQ ∼ ∆HNM (Q và M khác phíađối với PN). Chứng minh rằng Q thuộc một đường thẳng cố định.

A

B

C

H

M

X

N

K

P

Q

F

Lời giải.

138

Page 139: I HÅC QUÈC GIA TP. HCM · 2020. 10. 18. · Ngo i b i vi¸t cõa anh Ph¤m Tu§n Huy tø n«m 2013, c¡c b¤n s³ th§y r§t nhi·u b i vi¸t m t¡c gi£ v¨n cán l håc sinh cõa

a) Gọi X là giao điểm của đường thẳng vuông góc với BC tại B và đường thẳngAC. K′ là giao điểm của NX và CM. Ta có ∆BMN ∼ ∆BCX (cùng hướng). Dođó có một phép vị tự quay tâm B, biến M 7→ N, C 7→ X. Khi đó K′ là giaođiểm của CM và BX thì K′ thuộc đường tròn ngoại tiếp tam giác BMN. Từ đâyK′ ≡ K. Vậy NK luôn đi qua điểm X cố định.

b) Xét phép vị tự tâm H : N 7→ P, M 7→ Q, B 7→ F. Ta có được ∆BMN ∼ ∆FQP.Từ đó ∠FQP = ∠BMN = ∠ACB = ∠FCP hay tứ giác CFPQ nội tiếp. Do đó∠QCP = ∠QFP = ∠MBN = 90◦. Vậy Q thuộc đường thẳng qua C vuông gócvới AC cố định.

Bài toán kết thúc.

Bài 5. Với mỗi số nguyên dương n, ta biết rằng tồn tại duy nhất số tự nhiên a thoả mãna2 ≤ n < (a + 1)2. Đặt ∆n = n− a2.

a) Tìm giá trị nhỏ nhất của ∆n khi n thay đổi và thoả mãn n = 15m2 với m ∈ N∗.b) Cho p, q là các số nguyên dương và d = 5(4p + 3)q2. Chứng minh rằng ∆d ≥ 5.

Lời giải.

a) Ta cần tìm số ∆n nhỏ nhất để phương trình 15m2 − a2 = ∆n có nghiệm nguyêndương. Ta thấy 15− 32 = 6. Do đó ta sẽ chứng minh phương trình không có

nghiệm với ∆n < 6. Ta có a2 + ∆n... 3 nên

∆n... 3 hoặc ∆n + 1

... 3 (2)

Mặt khác a2 + ∆n... 5 nên

∆n ≡ 0, 1, 4 (mod 15) (3)

Từ (2) và (3), ta có nếu ∆n < 6 thoả mãn đề bài thì ∆n = 5.Ta có 15m2 − a2 = 5 nên a là bội của 5. Đặt a = 5s, ta có 3m2 − 5s2 = 1.Mà 3m2 − 1 ≡ −1, 2, 1 (mod 5) nên phương trình vô nghiệm. Vậy ∆n nhỏ nhấtlà 6.

b) Xét phương trình 5(4p + 3)q2 − a2 = ∆d. Ta có a2 ≡ 0, 1, 4 (mod 5) nên k ≡0, 1, 4 (mod 5). Ta có 5(4p + 3) ≡ 3 (mod 4) nên không phải số chính phương.

Nếu ∆d = 1 ta có a2 + 1 = 5(4p + 3)q2. Ta có 5(4p + 3) là số có dạng 4k + 3nên tồn tại ước nguyên tố r có dạng 4k + 3, suy ra a2 + 1 ≡ 0 (mod r) và 1 ≡ 0(mod r), vô lý.Tương tự cho trường hợp ∆d = 4 thì phương trình 5(4p + 3)q2 − a2 = 4 cũngvô nghiệm. Vậy ∆d ≥ 5.

Bài toán kết thúc.

139

Page 140: I HÅC QUÈC GIA TP. HCM · 2020. 10. 18. · Ngo i b i vi¸t cõa anh Ph¤m Tu§n Huy tø n«m 2013, c¡c b¤n s³ th§y r§t nhi·u b i vi¸t m t¡c gi£ v¨n cán l håc sinh cõa

Bài 6. Với các số nguyên a, b, c, d thoả mãn 1 ≤ a < b < c < d; kí hiệu

T(a, b, c, d) = {{x, y, z, t}|1 ≤ x < y < z < t; x ≤ a, y ≤ b, z ≤ c, t ≤ d}

a) Tính số phần tử của T(1, 4, 6, 7).b) Cho a = 1, b ≥ 4. Gọi d1 là số phần tử của T(a, b, c, d) chứa 1 và không chứa 2; d2 là

số phần tử chứa 1, 2 và không chứa 3; d3 là số phần tử chứa 1, 2, 3 và không chứa 4.Chứng minh rằng d1 ≥ 2d2 − d3. Đẳng thức xảy ra khi nào?

Lời giải.

a) Do x ≤ 1 nên x = 1. Do đó 2 ≤ y ≤ 4 và y ∈ {2, 3, 4}. Có các khả năng:

i. Nếu y = 2 thì 3 ≤ z ≤ 6. Mỗi giá trị của z có 7− z giá trị của t nên có 10 bộsố.

ii. Nếu y = 3 thì 4 ≤ z ≤ 6. Tương tự, ta có 6 bộ số.iii. Nếu y = 4 thì 5 ≤ z ≤ 6. Tương tự, ta có 3 bộ số.

Vậy có 19 bộ số thoả mãn yêu cầu bài toán.b) Đặt các tập hợp sau:

T1 = T(1, b, c, d) = {(1, y, z, t)|3 ≤ y ≤ b, y ≤ z ≤ c, z ≤ t ≤ d}T2 = T(1, 2, c, d) = {(1, 2, z, t)|4 ≤ z ≤ c, z ≤ t ≤ d}T3 = T(1, 2, 3, d) = {(1, 2, 3, t)|5 ≤ t ≤ d}

Ta có d3 = |T3| = d− 4 và

d2 =c

∑z=4

(d− z) = (c− 3) d +(c + 4) (c− 3)

2

Tiếp theo ta tính d1 = |T1|. Vì b ≥ 4 nên y ≥ 3. Có các khả năng sau:

• Nếu y = 3 thì T(1, 3, z, t) = d2.

• Nếu y = 4 thì T(1, 4, z, t) =c

∑z=5

(d− z) = (c− 4)d− (c + 5)(c− 4)2

.

Suy ra d1 ≥ d2 + (c− 4)d− (c + 5)(c− 4)2

. Do đó:

d1 + d3− 2d2 ≥ (c− 4)d− (c + 5)(c− 4)2

+ d− 4− (c− 3)d+(c + 4)(c− 3)

2= 0

Vậy d1 + d3 ≥ 2d2. Đẳng thức xảy ra khi b = 4. Bài toán kết thúc.

Bài toán kết thúc.

140

Page 141: I HÅC QUÈC GIA TP. HCM · 2020. 10. 18. · Ngo i b i vi¸t cõa anh Ph¤m Tu§n Huy tø n«m 2013, c¡c b¤n s³ th§y r§t nhi·u b i vi¸t m t¡c gi£ v¨n cán l håc sinh cõa

Bài 7. Trong một hệ thống máy tính, một máy tính bất kì có kết nối trực tiếp với ít nhất 30%máy tính khác của hệ thống. Hệ thống này có một chương trình cảnh báo và ngăn chặn khátốt, do đó khi một máy tính bị virus, nó chỉ có đủ thời gian lây cho các máy tính được kết nốitrực tiếp với nó. Chứng minh rằng dù vậy, kẻ tấn công vẫn có thể chọn hay máy tính của hệthống mà nếu thả virus vào hai máy đó, ít nhất 50% máy tính của hệ thống bị nhiễm virus.

Lời giải. (Trần Nam Dũng) Trước hết ta chứng minh bổ đề sau:

Bổ đề. Xét một tập con S bất kỳ của tập các máy tính X. Khi đó tồn tại 1 máy tính của hệthống kết nối trực tiếp với ít nhất 30% máy tính của S.

Chứng minh. Xét các cặp (s, x) với s thuộc S và x thuộc X và s, x kết nối trực tiếp

nhau. Khi đó, tính theo s thì số cặp như vậy sẽ không ít hơn3

10|S|.|X|. Do đó nếu tính

theo x thì sẽ phải tồn tại x kết nối trực tiếp với ít nhất3

10|S|.

Quay trở lại bài toán. Xét hệ thống có n máy tính và máy tính A bất kỳ. Gọi S là tậphợp các máy tính không kết nối trực tiếp với A. Nếu S rỗng thì bài toán là hiển nhiên.Nếu S không rỗng thì theo bổ đề, tồn tại máy tính B kết nối trực tiếp với ít nhất 30%máy tính trong S. Ta chứng minh hai máy tính A và B thoả mãn yêu cầu bài toán.

Thật vậy, giả sử A kết nối trực tiếp với k máy khác. Khi đó, theo cách chọn A hợp vớiB sẽ kết nối trực tiếp với ít nhất:

k + 0, 3(n− k) = 0, 7k + 0, 3n ≥ 0, 7.0, 3n + 0, 3n = 0, 51n

Do đó ta có điều phải chứng minh.

Bài 8. Cho tam giác ABC nhọn; đường tròn (I) có tâm I thuộc cạnh BC và tiếp xúc với cáccạnh AB, AC lần lượt tại E, F. Lấy M, N bên trong tứ giác BCEF sao cho EFNM nội tiếp(I) và các đường thẳng MN, EF, BC đồng quy. MF cắt NE tại P, AP cắt BC tại D.

a) Chứng minh A, D, E, F cùng thuộc một đường tròn.b) Lấy trên các đường thẳng BN, CM cắt điểm H, K sao cho ∠ACH = ∠ABK = 90o.

Gọi T là trung điểm HK. Chứng minh TB = TC.

Lời giải.

141

Page 142: I HÅC QUÈC GIA TP. HCM · 2020. 10. 18. · Ngo i b i vi¸t cõa anh Ph¤m Tu§n Huy tø n«m 2013, c¡c b¤n s³ th§y r§t nhi·u b i vi¸t m t¡c gi£ v¨n cán l håc sinh cõa

A

B CI

E

F

Q

NM

D

K

HT

Bổ đề 1 Cho đường tròn (O; R) và điểm Q nằm ngoài đường tròn. Cát tuyến qua Q cắt(O) tại M, N. Tiếp tuyến tại M, N cắt nhau tại A, vẽ AD⊥OQ. Khi đó OD.OQ = R2.

Bổ đề 2 (Định lý Brocard) Cho tứ giác ABCD nội tiếp đường tròn. Gọi P là giao điểmcủa AD và BC, Q là giao điểm của AB, CD. I là giao điểm của AC và BD. Khi đóPI⊥OQ và nếu D là giao điểm của PI và OQ thì OD.OQ = R2.

A

B

C

D

O

Q

P

I

H

K

Bổ đề 3 (Định lý Desargue) Cho hai tam giác ABC và A′B′C′. Gọi M là giao điểmcủa ABvA′B′, N là giao điểm của AC và A′C′; P là giao điểm của BC và B′C. Khi đóM, N, P thẳng hàng khi và chỉ khi AA′, BB′ và CC′ đồng quy.

142

Page 143: I HÅC QUÈC GIA TP. HCM · 2020. 10. 18. · Ngo i b i vi¸t cõa anh Ph¤m Tu§n Huy tø n«m 2013, c¡c b¤n s³ th§y r§t nhi·u b i vi¸t m t¡c gi£ v¨n cán l håc sinh cõa

Bổ đề 4. Cho tam giác ABC,hai tia Ax, Ay đối xứng nhau qua phân giác góc A. GọiH, K là hình chiếu của B trên Ax, Ay; P, Q là hình chiếu của C trên Ax, Ay. Khi đó 4điểm H, K, P, Q cùng thuộc một đường tròn và tâm là trung điểm BC.

A

BC

H

Q

P

K

M

Trở lại bài toán.

a) Theo Bổ đề 1, gọi D′ là hình chiếu của A trên BC. Ta có ID′.IQ = R2.Gọi X là giao điểm của ME và NF. XP cắt BC tại D′′. Suy ra XP⊥BC vàID′′.IQ = R2. Suy ra D′′ ≡ D′. Từ đó ta có X, A, P, D thẳng hàng và EFID nộitiếp.

b) Gọi S là giao điểm của CM và CN. Áp dụng định lý Desargue cho hai tam giácPEF và SBC ta có A, P, S thẳng hàng. Suy ra S ∈ AD.Ta chứng minh được ∠BAK = ∠CAH. Suy ra AK, AH đối xứng nhau qua phângiác góc ∠BAC.Áp dụng Bổ đề 4, ta có trung điểm HK cách đều BC.

Bài toán kết thúc.

2.2. Đề thi năm 2017

Bài 1. Cho dãy số (un) thoả un > 0, un > un+1, ∀n ∈ N∗ và dãy (sn) hội tụ với sn =n∑

i=1ui

.

a) Chứng minh lim(nun) = 0.

b) Đặt bn =1

un+1− 1

un, ∀n ∈ N∗. Chứng minh dãy (bn) không bị chặn.

143

Page 144: I HÅC QUÈC GIA TP. HCM · 2020. 10. 18. · Ngo i b i vi¸t cõa anh Ph¤m Tu§n Huy tø n«m 2013, c¡c b¤n s³ th§y r§t nhi·u b i vi¸t m t¡c gi£ v¨n cán l håc sinh cõa

Lời giải.

a) Lấy ε > 0 bất kỳ. Dãy (sn) hội tụ nên ta cóVới

ε

2> 0 luôn tồn tại N > 0 sao cho

∀n > m > N ⇒ |sn − sm| <ε

2(tính chất của dãy Cauchy) (4)

hay khi đó

|um+1 + um+2 + · · ·+ un| <ε

2.

Cố định m = N + 1.

Do dãy giảm nên suy ra (n−m) un < ε, hay

nun <ε

2+ mun. (5)

Lại do dãy (sn) hội tụ nên tiếp tục áp dụng (4) cho n = m + 1, ta chứng minhđược (un)→ 0.

Khi đó, vớiε

2m> 0, ∃N′ : ∀n > N′ ⇒ |un| <

ε

2m

mun <ε

2(6)

Từ (5) và (6) suy ra ∀n > max {N, N′} thì nun < ε. Vậy ta có đpcm.b) (Kỹ thuật tách tương tự định lý Cesaro). Giả sử (bn) bị chặn bởi c. Ta có

1nun− (c + 1) =

1unn− (c + 1) =

[(1

un− 1

un−1

)+ · · ·+

(1u1− 1

u0

)]+

1u0

n− (c + 1)

=

n−1∑

i=0bi

n− (c + 1)

+

1nu0

≤ cnn− (c + 1) +

1nu0

=1

nu0− 1, với mọi n.

Suy ra1

nun≤ (c + 1), khi n đủ lớn thoả mãn

1nu0

< 1.

Điều này mâu thuẫn câu 1. Do đó điều giả sử là sai và ta có đpcm.

Bài toán kết thúc.

Nhận xét. Trong bài toán này, câu a chính là một bước quan trọng để xử lý câu b. Tuynhiên, ta có thể chứng minh trực tiếp kết quả ở b như sau.

144

Page 145: I HÅC QUÈC GIA TP. HCM · 2020. 10. 18. · Ngo i b i vi¸t cõa anh Ph¤m Tu§n Huy tø n«m 2013, c¡c b¤n s³ th§y r§t nhi·u b i vi¸t m t¡c gi£ v¨n cán l håc sinh cõa

Giả sử ngược lại rằng dãy (bn) bị chặn, tức là tồn tại M > 0 để bn < M với mọi n.Khi đó ta có

1un

=1u1

+n−1

∑k=1

(1

uk+1− 1

uk

)≤ 1

u1+ (n− 1)M.

Suy ra

un ≥u1

1 + (n− 1)u1M=

11u1

+ (n− 1)M.

Ta thấy rằng với n đủ lớn, giả sử rằng n ≥ N0 thì (n − 1)M >1u1

nên khi đó

un >1

2(n− 1)M. Khi đó, tổng

sn = a1 + a2 + . . . + an >1

2M

n

∑k=N0

1k− 1

.

Chú ý rằng tổngn∑

k=1

1k→ +∞ nên tổng

n∑

k=N0

1k− 1

cũng thế. Điều này cho thấy

sn → +∞, mâu thuẫn.Vậy dãy bn không bị chặn.

Bài 2. Gọi S là tập con của {1, 2, . . . , 2017} sao cho S không chứa hai phần tử mà phần tửnày chia hết cho phần tử kia và cũng không chứa hai phần tử nguyên tố cùng nhau. Hỏi Schứa nhiều nhất bao nhiêu phần tử?

Lời giải. Xét tập hợp các số S = {1010, 1012, 1014, . . . , 2016} thì rõ ràng tất cả đềuchẵn nên không có hai số nào nguyên tố cùng nhau. Hơn thế nữa, số lớn nhất nhỏhơn 2 lần số nhỏ nhất nên cũng không có hai số nào mà số này chia hết cho số kia.Khi đó, S thoả mãn đề bài và có 504 phần tử.

Ta sẽ chứng minh rằng đây chính là giá trị lớn nhất cần tìm. Giả sử ngược lại rằng tồntại một tập hợp S′ thoả mãn tính chất đề bài và có ít nhất 505 phần tử.

Với mỗi số chẵn i từ 1010→ 2016 với dạng i = 2α ·m và (m, 2) = 1, xét tập hợp Ai códạng Ai =

{2k ·m|0 ≤ k ≤ α

}. Rõ ràng 2017 nguyên tố nên 2017 không thuộc S (vì

nếu không thì tất cả các số còn lại đều nguyên tố cùng nhau với nó, mâu thuẫn vớitính chất của S).

Ta sẽ chỉ ra rằng mọi số nguyên dương a không vượt quá 1008 đều thuộc về ít nhấtmột tập hợp Ai nào đó nêu trên.

Thật vậy, xét các số 2a, 22a, 23a, . . . và giả sử rằng không có số k nào để 1010 ≤ k ≤2016. Khi đó, phải có số m để 2ma < 1010 < 2016 < 2m+1a , suy ra

{2ma ≤ 1008

2m+1a ≥ 2018⇔{

2ma ≤ 1008

2ma ≥ 1009.

145

Page 146: I HÅC QUÈC GIA TP. HCM · 2020. 10. 18. · Ngo i b i vi¸t cõa anh Ph¤m Tu§n Huy tø n«m 2013, c¡c b¤n s³ th§y r§t nhi·u b i vi¸t m t¡c gi£ v¨n cán l håc sinh cõa

Điều vô lý này cho thấy nhận xét được chứng minh.

Tiếp theo, ta xét 504 tập A1009 ∪ {1011}, A1012 ∪ {1011}, . . . , A2016 ∪ {2015} thì các tậphợp này chứa tất cả các số từ 1 đến 2016.

Vì S′ có ít nhất 505 phần tử nên phải có ít nhất hai phần tử a < b thuộc cùng một tậpnào đó trong 504 tập hợp ở trên. Ta có hai trường hợp:

• Nếu b = i + 1 và a ∈ Ai thì rõ ràng (i, i + 1) = 1 nên b nguyên tố cùng nhau vớitất cả các số thuộc Ai, dẫn đến (a, b) = 1, không thoả.• Nếu a, b ∈ Ai thì theo cách xây dựng tập Ai, ta sẽ có a|b, cũng không thoả.

Vậy không tồn tại tập S′ nên giá trị lớn nhất cần tìm là 504.

Nhận xét. Ta biết các kết quả quen thuộc sau:

a) Nếu chọn n + 1 số nguyên dương bất kỳ từ 2n số nguyên dương đầu tiên thìluôn có số này chia hết cho số kia (chứng minh bằng cách xét các lũy thừa của2). Giá trị n + 1 cũng là tốt nhất vì nếu chỉ chọn n số, ta có thể chọn các số từn + 1, n + 2, . . . , 2n và không có hai số nào chia hết cho nhau.

b) Nếu chọn n + 1 số nguyên dương bất kỳ từ 2n số nguyên dương đầu tiên thìluôn có hai số nguyên tố cùng nhau (chứng minh bằng cách chia n nhóm cáccặp số lẻ và chẵn liên tiếp). Giá trị n + 1 cũng là tốt nhất vì nếu chỉ chọn n, ta cóthể chọn tất cả các số chẵn thì không có hai số nào nguyên tố cùng nhau.

Bài toán và lời giải ở trên có thể nói là một kết hợp rất thú vị, độc đáo của hai bài toánrất đơn giản và quen thuộc vừa nêu.

Bài 3. Cho n là số tự nhiên lớn hơn 2 và X = {1, 2, 3, . . . , n}. Với mỗi song ánh f : X → X,gọi A f là tập hợp tất cả các bộ số (i; j) sao cho i < j và f (i) > f (j).

a) Có bao nhiêu song ánh f thoả |A f | = 1?b) Giả sử f là một song ánh mà A f = k > 0. Chứng minh rằng tồn tại một song ánh

g : X → X sao cho |Ag| = k− 1 vàn∑

i=1| f (i)− i| >

n∑

i=1|g (i)− i|.

(|X| chỉ số phần tử của tập hợp X).

Lời giải.

a) Với mỗi song ánh f , đặt ai = f (i) thì rõ ràng (a1, a2, . . . , an) chính là hoán vị của(1, 2, . . . , n).

Gọi Sn là số song ánh f có∣∣A f

∣∣ = 1 ứng với tập hợp n số nguyên dương đầutiên.

Dễ thấy với n = 3, ta có hai hoán vị thoả mãn là 1, 3, 2 và 2, 1, 3 nên S3 = 2.

Xét quan hệ giữa Sn+1, Sn thông qua hoán vị của dãy (1, 2, . . . , n, n + 1).

146

Page 147: I HÅC QUÈC GIA TP. HCM · 2020. 10. 18. · Ngo i b i vi¸t cõa anh Ph¤m Tu§n Huy tø n«m 2013, c¡c b¤n s³ th§y r§t nhi·u b i vi¸t m t¡c gi£ v¨n cán l håc sinh cõa

• Nếu an+1 = n + 1 thì loại n + 1 ra, ta có một hoán vị thoả mãn đề bài của nsố nguyên dương đầu tiên, có tất cả Sn hoán vị như thế.• Nếu an+1 6= n + 1 thì rõ ràng an = n + 1 vì nếu ai = n + 1 với i < n thì∣∣A f

∣∣ ≥ 2, không thoả. Khi đó, phải có an+1 = n, an = n + 1 và ai = i với1 ≤ i ≤ n− 1. Nghĩa là có thêm đúng 1 hoán vị nữa.

Từ đó suy ra Sn+1 = Sn + 1 và có Sn = n− 1 với mọi n. Số song ánh cần tìm làn− 1.

b) Trong song ánh f đang xét, gọi j là số lớn nhất sao cho aj = t < j. Rõ ràng j luôntồn tại vì

∣∣A f∣∣ = k > 0. Khi đó, ta cũng có ai = i với mọi j + 1 ≤ i ≤ n.

Xét số m sao cho am = t + 1 ≤ j thì rõ ràng 1 ≤ m ≤ j. Khi đổi chỗ hai số (aj, am)

này sẽ làm giảm giá trị∣∣A f

∣∣ đi đúng một đơn vị. Khi đó, giả sử ta có song ánhg mới tương ứng với việc đổi chỗ này mà

∣∣Ag∣∣ = k− 1 và đặt bi = g(i). Ta sẽ

chứng minh rằng T =n∑

i=1|bi − i| −

n∑

i=1|ai − i| ≤ 0.

Thật vậy,T =

∣∣bj −m∣∣+ |bm − j| −

∣∣aj − j∣∣− |am −m|

= |t + 1− j|+ |t−m| − |t− j| − |t + 1−m|Vì t + 1 ≤ j nên ta có

T = j− (t + 1)− (j− t) + |t−m| − |t + 1−m| = |t−m| − |t + 1−m| − 1

Vì t−m và t + 1−m là hai số nguyên liên tiếp nên chênh lệch của chúng đúngbằng 1, suy ra T ≤ 1− 1 = 0.

Do đó, hoán vị g như trên thoả mãn điều kiện đề bài. Ta có đpcm.

Bài toán kết thúc.

Nhận xét. Ta biết rằng số lượng A f được nêu trong bài chính là số nghịch thế. Nếugọi Sk là "độ lệch" nhỏ nhất của song ánh có k nghịch thế thì theo kết quả ở câu b, tacó dãy Sk là một dãy không giảm.

Từ đó suy ra rằng độ lệch ở trên lớn nhất khi số nghịch thế là lớn nhất, điều này đạtđược khi hai nửa của các số 1, 2, . . . , n được đổi chỗ cho nhau.

Ở câu a, nếu yêu cầu tính số song ánh f để∣∣A f

∣∣ = 2 thì bài toán sẽ thú vị hơn nhiều.Khi đó, vẫn bằng cách truy hồi tương tự, ta sẽ có được công thức là Sn = Sn−1 + n− 2

và S3 = 1 hay Sn =(n− 1)(n− 2)

2.

Bài 4. Cho tam giác ABC nhọn nội tiếp đường tròn (O) và điểm D di động trên cung BC chứađiểm A (D 6= A). Trên AB, AC lần lượt lấy các điểm M, N sao cho MD = MB, NC = ND.

a) Chứng minh đường cao DH trong tam giác DMN luôn đi qua một điểm cố định.

147

Page 148: I HÅC QUÈC GIA TP. HCM · 2020. 10. 18. · Ngo i b i vi¸t cõa anh Ph¤m Tu§n Huy tø n«m 2013, c¡c b¤n s³ th§y r§t nhi·u b i vi¸t m t¡c gi£ v¨n cán l håc sinh cõa

b) DM, DN theo thứ tự cắt (O) tại E, F ( E, F khác D). Chứng minh các đường trònngoại tiếp các tam giác EMB, FNC cắt nhau tại điểm K thuộc đường thẳng BC vàđường cao KI của tam giác KMN luôn qua một điểm cố định.

Lời giải.

a) Cách 1. Giả sử DH cắt (O) tại X. Ta có

∠AMD = 2∠ABD = 2∠ACD = ∠AND,

suy ra tứ giác AMND nội tiếp. Suy ra ∠DMN = ∠DAC = ∠DBC. Ta có

∠BDX = ∠HDM−∠MDB = 90◦ −∠HMD−∠MDB

= 90◦ −∠MBD−∠DBC = 90◦ −∠ABC.

Suy ra AX⊥BC nên X cố định. Ta có đpcm.

Cách 2. Gọi Q là giao điểm của (M; MB) và (N; NC). Ta có

∠MQB = ∠MBQ,∠NQC = ∠NCQ,∠MQN = ∠MDN = ∠MAN.

Suy ra ∠MQB +∠NQC +∠MQN = 180◦ nên B, Q, C thẳng hàng.

148

Page 149: I HÅC QUÈC GIA TP. HCM · 2020. 10. 18. · Ngo i b i vi¸t cõa anh Ph¤m Tu§n Huy tø n«m 2013, c¡c b¤n s³ th§y r§t nhi·u b i vi¸t m t¡c gi£ v¨n cán l håc sinh cõa

Giả sử DQ cắt (O) tại X. Khi đó

XBXC

=sin∠BDXsin∠CDX

=sin

∠BMQ2

sinCNQ

2

=cos∠ABCcos∠ACB

không đổi, nên X cố định.b) Ta có ∠MEB = 180◦ −∠BCD = 180◦ −∠BOM, suy ra OMEB nội tiếp.

Tương tự CNOF nội tiếp. Khi đó (OMB) và (OCN) cắt nhau tại K thuộc BC.

Ta có

∠OMK = ∠OBC,∠∠MKN = ∠MKO +∠NKO = ∠OBA +∠OCA = ∠BAC.

Suy ra ∠OMK + ∠MKN = ∠BAC + ∠OBC = 90◦, suy ra MO⊥KN. Chứngminh tương tự thì ON⊥KM. Do đó O là trực tâm tam giác KMN.

Vậy KO⊥MN, từ đó KI qua O cố định.

Bài toán kết thúc.

Bài 5. Với mỗi số nguyên n /∈ {−1, 0, 1}, ký hiệu p(n) là ước nguyên tố lớn nhất của n. GọiF là tập hợp tất cả các đa thức f (x) có hệ số nguyên và thoả mãn

f (n + p(n)) = n + p( f (n))

với mọi số nguyên n > 2017 và f (n) /∈ {−1, 0, 1}.

a) Tìm tất cả các đa thức bậc nhất thuộc F.b) F có bao nhiêu phần tử?

Lời giải. Trước hết, ta thấy rằng các đa thức hằng f (n) ≡ 0, f (n) ≡ 1, f (n) ≡ −1thoả mãn bài toán vì không có ràng buộc ở đề bài cho các giá trị này.

Ta thấy rằng có hữu hạn giá trị n > 2017 để f (n) ∈ {0, 1,−1} nên có vô hạn giá trịn > 2017 để f (n) /∈ {0, 1,−1}, vì thế nên trong lập luận bên dưới, ta chỉ xét các giátrị n để f (n) /∈ {0, 1,−1}.Ta có nhận xét rằng với mọi n nguyên và |n| ≥ 2 thì 2 ≤ p(n) ≤ |n| và đẳng thức ởbất đẳng thức thứ hai xảy ra khi và chỉ khi |n| nguyên tố.

Trong đẳng thức đã cho, thay n = q > 2017 là số nguyên tố, ta có

f (q + p(q)) = q + p( f (q)) hay f (2q) = q + p( f (q)). (∗)

149

Page 150: I HÅC QUÈC GIA TP. HCM · 2020. 10. 18. · Ngo i b i vi¸t cõa anh Ph¤m Tu§n Huy tø n«m 2013, c¡c b¤n s³ th§y r§t nhi·u b i vi¸t m t¡c gi£ v¨n cán l håc sinh cõa

Suy ra f (2q) > q với mọi q > 2017 là số nguyên tố. Do đó, f (n) không thể là đa thứchằng (khác 0, 1,−1) và trong trường hợp deg f > 0, bậc cao nhất của f cũng khôngthể âm (nếu không thì tồn tại q đủ lớn để f (2q) < 0, mâu thuẫn.

Cũng từ đẳng thức trên, ta có f (2q) ≤ q + f (q) với mọi q nguyên tố và q > 2017.

• Nếu deg f = 1, đặt f (n) = an + b với a 6= 0. Bất đẳng thức ở trên đưa về

a(2q) + b ≤ q + aq + b⇔ aq ≤ q⇔ a ≤ 1.

Theo nhận xét ở trên thì a > 0 nên a = 1. Thay vào (∗), ta có

2q + b = q + p(q + b)⇔ q + b = p(q + b).

Từ đây suy ra q + b là số nguyên tố. Chú ý rằng ta chọn được q đủ lớn đểq + b > 2017 nên áp dụng lập luận trên khi thay q bởi q + b, ta có q + 2b cũngphải là số nguyên tố.

Tương tự như thế, suy ra q + kb là số nguyên tố với k nguyên dương bất kỳ.Chọn k = q, ta có q + qb = q(b + 1) phải là số nguyên tố. Điều này chỉ xảy rakhi b = 0.

Do đó, f (n) = n. Thử lại ta thấy thoả mãn.

• Nếu deg f = k ≥ 2, theo chứng minh ở trên thì f (2q) ≤ q + f (q) và hệ số bậccao nhất a > 0. Chia hai vế cho (2q)k rồi tính lim q → +∞, ta thấy vế trái tiếntới a trong khi vế phải tiến tới 0.

Suy ra a ≤ 0, điều mâu thuẫn này cho thấy không tồn tại đa thức bậc cao hơn 1thoả mãn đề bài.

Vậy có tất cả 4 đa thức thoả mãn đề bài là f (n) ≡ 0, 1,−1 và f (n) = n thoả mãn nên|F| = 4.

Nhận xét. Một bài toán tương tự với bài toán trên nhưng xét ước nguyên tố nhỏ nhất:Gọi d(n) là ước nguyên tố nhỏ nhất của số nguyên n /∈ {0, 1,−1}. Xác định tất cả các đathức P(x) hệ số nguyên sao cho

P(n + d(n)) = n + d(P(n)).

Việc thêm yêu cầu với mọi số nguyên n > 2017 vừa là một ràng buộc, vừa là mộtgợi ý khi đòi hỏi học sinh chỉ xét các số nguyên đủ lớn. Một bài toán đa thức sốhọc có ý tưởng tương tự khi xét đa thức bậc nhất và chứng minh đa thức bậc từ 2trở lên không thoả có xuất hiện trong đề VN TST 2014 như sau: Cho dãy số nguyênu1 = 1, un+1 = P(un), n ≥ 1 với P(x) là một đa thức có hệ số nguyên. Biết rằng mọi sốnguyên dương là ước của một số hạng nào đó của dãy. Chứng minh rằng P(n) = n + 1.

150

Page 151: I HÅC QUÈC GIA TP. HCM · 2020. 10. 18. · Ngo i b i vi¸t cõa anh Ph¤m Tu§n Huy tø n«m 2013, c¡c b¤n s³ th§y r§t nhi·u b i vi¸t m t¡c gi£ v¨n cán l håc sinh cõa

Bài 6. Với mỗi số tự nhiên n, ký hiệu T(1 + n, 3 + n, 4 + n) là tập hợp tất cả các bộ (a, b, c)với a, b, c là các số tự nhiên thoả 1 ≤ a ≤ 1 + n, a + 1 ≤ b ≤ 3 + n, b + 1 ≤ c ≤ 4 + n. Gọian là số phần tử của T(1 + n, 3 + n, 4 + n).

a) Tính a4.b) Tìm tất cả n sao cho an chia hết cho 3.

Lời giải.

a) Xét một bộ (a, b, c) gồm các số nguyên dương thoả mãn 1 ≤ a < b < c ≤ n + 4.Rõ ràng có tất cả C3

n+4 bộ như thế.

Ta có hai trường hợp:

• Nếu a = n + 2 thì rõ ràng b = n + 3, c = n + 4 và chỉ có duy nhất một bộ(a, b, c) như thế. Bộ này không thoả mãn điều kiện của bài toán.• Nếu a ≤ n + 1, dĩ nhiên ta cũng có b ≤ n + 3, c ≤ n + 4 và đây là tất cả các

bộ thoả mãn đề bài.

Do đó, số các bộ thoả mãn đề bài là an = C3n+4 − 1.

Suy ra a4 = C38 − 1 = 55.

b) Ta có

an =(n + 4)(n + 3)(n + 2)

6− 1 =

(n + 1)(n2 + 8n + 18)6

=n(n + 1)(n + 8)

6+ 3(n+ 1).

Vì 3|3(n + 1) nên để an chia hết cho 3 thì n(n + 1)(n + 8) phải chia hết cho 9. Tathấy rằng không có hai số nào trong ba số n, n + 1, n + 8 có thể cùng chia hếtcho 3 nên điều kiện trên chỉ xảy ra khi 9|n hoặc 9|n + 1 hoặc 9|n + 8.

Vậy tất cả các số n cần tìm để 3|an là n chia 9 dư 0, 1, 8.

Bài toán kết thúc.

Nhận xét. Nếu nhận ra được sự tương ứng giữa T(1 + n, 3 + n, 4 + n) trong đề bàivới tổ hợp các số nguyên dương phân biệt không vượt quá n + 4 thì bài toán có thểgiải quyết đơn giản như trên. Nếu không, ta có thể dùng nguyên lý bù trừ, đưa vềcông thức truy hồi dạng

a0 = 3, an = an−1 +(n + 2)(n + 3)

2, n ≥ 0.

Bài toán tương tự đã từng xuất hiện trong đề chọn đội tuyển của PTNK năm trước:Với các số nguyên a, b, c, d thoả 1 ≤ a < b < c < d, ký hiệu

T(a, b, c, d) = {(x, y, z, t) ∈ N | 1 ≤ x < y < z < t, x ≤ a, y ≤ b, z ≤ c, t ≤ d} .

151

Page 152: I HÅC QUÈC GIA TP. HCM · 2020. 10. 18. · Ngo i b i vi¸t cõa anh Ph¤m Tu§n Huy tø n«m 2013, c¡c b¤n s³ th§y r§t nhi·u b i vi¸t m t¡c gi£ v¨n cán l håc sinh cõa

a) Tính số phần tử của T(1, 4, 6, 7).b) Cho a = 1 và b ≥ 4. Gọi d1 là số phần tử của T(a, b, c, d) chứa 1 nhưng không chứa

2, d2 là số phần tử chứa 1, 2 nhưng không chứa 3, d3 là số phần tử chứa 1, 2, 3 nhưngkhông chứa 4. Chứng minh rằng d1 ≥ 2d2 − d3. Dấu đẳng thức xảy ra khi nào?

Bài 7. An và Bình luân phiên đánh dấu các ô vuông của hình vuông 101× 101 ô. An làngười bắt đầu. Một ô sẽ không thể được đánh dấu nếu trên cùng hàng với nó hoặc cùng cộtvới nó đã có ít nhất 2 ô được đánh dấu. Ai không đi được nữa sẽ thua. Hãy xác định ai làngười có chiến thuật thắng.

Lời giải.Cách 1. Để đơn giản, ta gọi hai người chơi là A và B thay vì An và Bình. Ta sẽ chứngminh rằng B có chiến thuật để thắng. Điều này cũng đúng khi thay 101 bằng một sốnguyên dương n ≥ 2 bất kỳ.

Rõ ràng theo luật chơi thì có không quá 2n ô được đánh dấu. Vì thế nên chiến thuật ởđây là B sẽ tìm cách đánh được ô cuối cùng.

Chiến thuật là: người đi trước đánh ô nào thì người sau sẽ đánh một ô bất kỳ cùngdòng với nó sao cho số cột được đánh là nhiều nhất có thể.

Đặc điểm của chiến thuật này là:

• Sau mỗi lượt của A, B thì có thêm một hàng có hai ô được đánh; không có hàngnào chứa một ô.• Sau mỗi lượt của A thì trên hàng mà A vừa đánh, còn đúng n− 1 ô chưa được

đánh và B sẽ đánh tuỳ ý vào ô thuộc cột chưa có ô nào được đánh; nếu như tấtcả các cột đều có ô được đánh thì B sẽ chọn cột tuỳ ý mà chỉ có 1 ô được đánhtrên đó.

Bằng cách đó, trong n− 1 lượt đầu tiên, một khi A còn đi được thì B vẫn đi được vìvẫn luôn còn hàng trống và các cột vẫn chưa đầy 2 ô. Do đó, sau n− 1 lượt của A, B,bảng còn lại sẽ có đặc điểm là:

• Chỉ còn một hàng duy nhất mà chưa có ô nào được đánh, n− 1 hàng kia đều cóô đã được đánh.• Tất cả các cột đều có ô được đánh (có cột có 1 ô, có cột có 2 ô).

Tổng số ô đã đánh là 2n− 2 nên gọi a, b lần lượt là số cột có 1 ô, 2 ô được đánh thì

a + b = n và a + 2b = 2n− 2.

Suy ra a = 2, b = n− 2, nghĩa là có đúng hai cột mà trên đó có 1 ô được đánh. Hai ônằm ở vị trí giao giữa hai cột đó và dòng chưa được đánh là hai vị trí cuối cùng cóthể đánh. A, B thay phiên đánh vào hai ô đó và B là người đánh cuối cùng nên chiếnthắng.

Cách 2. Ta có nhận xét rằng:

152

Page 153: I HÅC QUÈC GIA TP. HCM · 2020. 10. 18. · Ngo i b i vi¸t cõa anh Ph¤m Tu§n Huy tø n«m 2013, c¡c b¤n s³ th§y r§t nhi·u b i vi¸t m t¡c gi£ v¨n cán l håc sinh cõa

a) Nếu ở lượt đầu tiên, An đánh vào ô (1, 51) giữa hàng 1 thì Bình sẽ đánh vàoô (101, 51) giữa hàng cuối. Khi đó, hai bạn sẽ không thể điền vào ô trung tâmnữa. Tiếp theo, mỗi khi An đánh dấu ô nào thì Bình sẽ đánh dấu vào ô đối xứngvới ô đó qua tâm. Mỗi khi An điền được thì Bình cũng điền được, vậy Bình sẽthắng.

b) Nếu ở lượt đầu tiên, An đánh vào ô (a, b) thì ta có thể đưa về nhận xét 1 bằngcách xây dựng một bảng tương ứng cùng kích thước nhưng hàng 1 và hàng a,cột 51 và cột b đổi chỗ cho nhau. Khi đó, An đánh vào ô nào thì Bình sẽ đánhvào ô ở chỉ số hàng/cột tương ứng ở bảng đối chiếu; Bình sẽ đánh vào ô đốixứng như ở chiến lược 1 rồi đánh vào ô có cùng chỉ số hàng/cột ở bảng gốc. Dễthấy rằng đánh dấu được của mỗi hàng và cột ở hai bảng là giống nhau.

Hình minh họa cho trường hợp 5× 5 khi An đánh vào ô (5, 5) trong nước đi đầu tiên.

Từ hai nhận xét trên, ta thấy Bình là người có chiến lược thắng trò chơi.

Nhận xét. Rõ ràng theo cách thứ nhất thì bài toán không chỉ đúng với số 101 hay cácsố lẻ mà còn đúng với bảng vuông có kích thước bất kỳ không nhỏ hơn 2. Để cảmnhận và đoán được ai là người có chiến lược thắng cuộc, ta có thể thử với trường hợpn = 2, n = 3. Khi n = 2, ta thấy tất cả ô vuông đều có thể được đánh dấu nên Bìnhhiển nhiên thắng. Khi n = 3, ta thấy ô ở trung tâm có vai trò rất quan trọng cho chiếnlược chơi. Chú ý rằng vẫn còn nhiều chiến lược khác thú vị hơn cho Bình để thắng tròchơi này.

Bài 8. Đường tròn (O) nội tiếp tứ giác ABCD và tiếp xúc với các cạnh AB, BC, CD, DAlần lượt tại E, F, G, H. Gọi I, J là trung điểm của AC, BD và IB, ID, JA, JC theo thứ tự cắtEF, GH, HE, FG tại M, N, P, Q.

a) Chứng minh rằng I J, MN, PQ đồng quy (tại điểm S).b) Các tia đối của các tia JA, IB, JC, ID lần lượt cắt (O) tại các điểm A′, B′, C′, D′. Giả

sử A′C′, B′D′ lần lượt cắt PQ, MN tại U, V. Gọi K là hình chiếu của S trên UV.Chứng minh rằng ∠AKB = ∠CKD.

Lời giải.

a) Trước hết, sử dụng định lý Menelaus trong các tam giác ABC và ADC, ta thấyrằng các đường thẳng GH và EF cùng cắt AC tại một điểm, đặt là U. Tương tự,các đường thẳng EH, FG cùng cắt BD tại một điểm, đặt là V.

153

Page 154: I HÅC QUÈC GIA TP. HCM · 2020. 10. 18. · Ngo i b i vi¸t cõa anh Ph¤m Tu§n Huy tø n«m 2013, c¡c b¤n s³ th§y r§t nhi·u b i vi¸t m t¡c gi£ v¨n cán l håc sinh cõa

Tiếp theo, áp dụng định lý Desargue cho hai tam giác AHP và CGQ ta cóGH; AC; PQ đồng quy tại U. Tương tự thì HE, MN, BD đồng quy tại V.

Áp dụng định lý Menelaus cho tam giác AIU thì

DNIN· IU

AU· AH

DH= 1 nên

DNIN

=AUIU· DH

AH.

Gọi S′ = MN ∩ I J thì

IS′

JS′· V J

VD· ND

NI= 1 nên

IS′

JS′=

NIND· VD

V J=

HAHD· UI

UA· VD

V J(7)

Tương tự, ta cũng cóAPPJ

=VDV J· AH

HD. Gọi S′′ là giao điểm của PQ và I J thì

IS′′

JS′′=

UIUS· AP

PJ=

UIUA· VD

V J· AH

HD(8)

Từ (7) và (8) suy ra S′ ≡ S′′.

154

Page 155: I HÅC QUÈC GIA TP. HCM · 2020. 10. 18. · Ngo i b i vi¸t cõa anh Ph¤m Tu§n Huy tø n«m 2013, c¡c b¤n s³ th§y r§t nhi·u b i vi¸t m t¡c gi£ v¨n cán l håc sinh cõa

Vậy PQ, MN, I J đồng quy tại một điểm đặt là S. Ta có đpcm.b) Bước 1. Chứng minh S trùng O. Theo câu a thì

ISJS

=UIUA

.VDV J

.AHHD

(9)

Theo định lý đường thẳng Newton, ta có I, O, J thẳng hàng và

IOOJ

=CF + AEBE + DH

.

Ta cóAEBE· BF

CF· UC

UA= 1, suy ra CF =

AE ·UCUA

. Suy ra

AE + CF = AE(1 +UCUA

) =AE(UC + UA)

UA= 2

AE ·UIUA

.

Tương tự thì BF + DH =2DH ·V J

VD. Khi đó

OIOJ

=2 · AE ·UI

UA:

2 · DH ·V JVD

=UIUA

.VDV J

.AHDH

(10)

Bước 2. A′C′, PQ, AC đồng quy tại U và B′D′; MN; BD đồng quy tại V.

Thật vậy, xét trong đường tròn (O), vì GH là đường đối cực của D và đi qua Unên đường đối cực của U đi qua D. Tương tự thì đường đối cực của U đi quaB nên BD là đường đối cực của U. Suy ra J(RU, C′A′) = −1, suy ra U, C′, A′

thẳng hàng.

Do đó A′C′, PQ, AC đồng quy. Tương tự với B′D′; MN; BD.

Ta cũng chú ý rằng HF, GE, AC, BD đồng quy tại R (theo định lý Brianchon suybiến từ lục giác ngoại tiếp thành tứ giác ngoại tiếp).

Bước 3. Theo định lý Brocard thì OR⊥UV tại K.

Ta có V(UR, VF) = −1, suy ra B(UR, EF) = −1, suy ra (U, R, A, C) = −1. Dođó K(UR, AC) = −1.

Mà KU⊥KR nên KR là phân giác góc ∠AKC. Chứng minh tương tự, ta có KRlà phân giác ∠BKD.

Vậy ∠AKB = ∠CKD.

Bài toán được giải quyết hoàn toàn.

Nhận xét. Câu a của bài toán có thể chứng minh không dùng tính toán mà bằng cực -đối cực như sau: Qua A dựng đường thẳng song song với BD thì A(DB, Jd) = −1

155

Page 156: I HÅC QUÈC GIA TP. HCM · 2020. 10. 18. · Ngo i b i vi¸t cõa anh Ph¤m Tu§n Huy tø n«m 2013, c¡c b¤n s³ th§y r§t nhi·u b i vi¸t m t¡c gi£ v¨n cán l håc sinh cõa

(vì J là trung điểm BD) nên A(HE, Pd) = −1. Gọi X là giao điểm của HE và d thì(HE, PX) = −1 nên X thuộc đường đối cực của P đối với (O).

Vì P thuộc đối cực của A nên A thuộc đối cực của P, đó chính là d. Suy ra OP ⊥ d hayOP ⊥ BD, vì thế nên U, P, O thẳng hàng. Tương tự thì U, Q, O cũng thẳng hàng nênPQ đi qua O. Chứng minh tương tự ta cũng có MN đi qua O nên MN, PQ, I J đồngquy tại O.

Một điều thú vị của câu a là nếu các điểm I, J thay đổi tuỳ ý trên các đoạn AC, BDvà các điểm M, N, P, Q được định nghĩa tương tự thì bài toán vẫn đúng. Tuy nhiên,điểm đồng quy đó không hẵn là O. Câu b là kết hợp của mô hình tứ giác ngoại tiếpcùng tứ giác toàn phần nội tiếp EFGHUV. Tuy nhiên, nó đòi hỏi phải xử lý khéo léocác chùm điều hòa nên thực sự là một thử thách đáng kể. Đây có thể nói là bài toánkhó nhất trong kỳ thi chọn đội tuyển này.

2.3. Đề thi năm 2018

Bài 1. Cho số nguyên dương a > 1. Tìm giá trị lớn nhất của số thực d sao cho tồn tại mộtcấp số cộng có công sai d và số hạng đầu là a và có đúng hai số trong các số a2, a3, a4, a5 làcác số hạng của cấp số cộng đó.

Lời giải. Do cần tìm giá trị lớn nhất của d nên ta chỉ cần quan tâm trường hợp d > 0.Rõ ràng b thuộc cấp số cộng công sai d và số hạng đầu là u1 = a khi và chỉ khib− a

d∈ Z+.

Điều này có nghĩa là cần tìm d sao cho trong các số a2 − a, a3 − a, a4 − a, a5 − a chỉ cóđúng hai số chia cho d thu được số nguyên.Chú ý rằng a2 − a là ước của các số còn lại nên số này không được thuộc cấp số cộng.Ngoài ra, a3 − a | a5 − a nên hai số xuất hiện không thể là a3 − a, a4 − a. Ta có haitrường hợp:

• Nếu a3 − a, a5 − a thuộc cấp số cộng. Khi đó d ≤ a3a và với d = a3 − a thì

a4 − ad

=a4 − aa3 − a

=a2 + a + 1

a + 1= a +

1a + 1

/∈ Z.

Do đó, trong trường hợp này, dmax = a3 − a.• Nếu a4 − a, a5 − a thuộc cấp số cộng. Chú ý rằng a5 − a = a(a4 − a) + (a2 − a)

nêna2 − a

d=

a5 − ad

+a(a4 − a)

d∈ Z,

kéo theo a2 − a cũng thuộc cấp số cộng, mâu thuẫn với điều kiện đề bài.

Vậy giá trị lớn nhất cần tìm của d là a3 − a.

Nhận xét. Ở bài này, nhận xét đầu tiên mang tính quyết định cho bài toán. Nếu lanman vào việc xét trường hợp ngay từ đầu thì rất khó đi đến kết quả. Bài toán chỉ dùngcác kết quả nhẹ nhàng về chia hết nhưng cách đặt câu hỏi rất hay.

156

Page 157: I HÅC QUÈC GIA TP. HCM · 2020. 10. 18. · Ngo i b i vi¸t cõa anh Ph¤m Tu§n Huy tø n«m 2013, c¡c b¤n s³ th§y r§t nhi·u b i vi¸t m t¡c gi£ v¨n cán l håc sinh cõa

Một số bài toán tương tự:

a) (Trường xuân 2013) Một cấp số cộng gồm các số nguyên dương được gọi là“chuẩn” nếu nó có ít nhất ba số hạng và tích các số hạng của nó là ước số củamột số có dạng n2 + 1.

a) Chứng minh rằng không tồn tại cấp số cộng chuẩn có công sai là d =

10, d = 11.b) Chứng minh rằng tồn tại cấp số cộng có d = 12. Hỏi cấp số đó có nhiều

nhất mấy số hạng?

b) Chứng minh rằng√

2,√

5,√

7 không thể là các số hạng của cùng một cấp sốcộng.

c) (KHTN 2011) Chứng minh rằng không tồn tại các số nguyên dương thoả b2 −a2 = c2 − b2 = d2 − c2.

Bài 2. Cho n số thực x1, x2, ..., xn. Với mỗi i ∈ {1, 2, 3, . . . , n}, gọi ai là số các chỉ số j saocho

∣∣xi − xj∣∣ ≤ 1 và bi là số các chỉ số j sao cho

∣∣xi − xj∣∣ ≤ 2 (các số i, j có thể bằng nhau).

a) Chứng minh rằng tồn tại i mà bi ≤ 3ai.b) Gọi A là số cặp (i, j) có thứ tự mà

∣∣xi − xj∣∣ ≤ 1 và B là số cặp (i, j) có thứ tự mà∣∣xi − xj

∣∣ ≤ 2 (các số i, j có thể bằng nhau). Chứng minh rằng B ≤ 3A.

Lời giải.

a) Không mất tính tổng quát, giả sử các số đã cho được sắp xếp tăng dần. Xétk = max {a1, a2, . . . , an} và giả sử ai = k, khi đó tồn tại k số trong dãy là

xu ≤ xu+1 ≤ . . . ≤ xi ≤ . . . ≤ xv với |xu − xi| , |xv − xi| ≤ 1.

Ngoài ra vì tính lớn nhất của k nên |xu−1 − xi| > 1, |xv+1 − xi| > 1.

Trong [xu, xv], có đúng k số j để∣∣xj − xi

∣∣ ≤ 1 < 2; còn trước xu, xét hai số xr, xs

trong đó sao cho xr ≤ xs và |xr − xi| ≤ 2, |xs − xi| ≤ 2 thì

|xr − xs| = xs − xr = (xi − xr)− (xi − xs) < 2− 1 = 1

nên sẽ có không quá k số j để∣∣xj − xi

∣∣ ≤ 2 vì nếu ngược lại, sẽ có nhiều hơn ksố liên tiếp trong dãy cách nhau không quá 1 đơn vị, mâu thuẫn với tính lớnnhất của k. Tương tự với các số sau xv, vì thế nên bi ≤ 3k⇒ bi ≤ 3ai.

b) Ta sẽ chứng minh bằng quy nạp theo n.

Với n = 1 rõ ràng A = B = 1 nên khẳng định hiển nhiên đúng.

Giả sử kết quả đúng với n ≥ 1, ta sẽ chứng minh nó cũng đúng với n + 1.

Xét dãy số thực T = (x1, x2, . . . , xn+1) bất kỳ và giả sử x1 ≤ x2 ≤ . . . ≤ xn+1, kýhiệu AT, BT là số cặp có thứ tự các chỉ số (i, j) tương ứng với định nghĩa của đề

157

Page 158: I HÅC QUÈC GIA TP. HCM · 2020. 10. 18. · Ngo i b i vi¸t cõa anh Ph¤m Tu§n Huy tø n«m 2013, c¡c b¤n s³ th§y r§t nhi·u b i vi¸t m t¡c gi£ v¨n cán l håc sinh cõa

bài. Giả sử k ≥ 1 là số lượng lớn nhất các số của T được chứa trong một đoạnđộ dài bằng 2 nào đó.

Gọi xi là số cuối cùng của dãy mà trong đoạn [xi − 1, xi + 1] có chứa đúng k số(kể cả xi). Gọi T′ là dãy mới sau khi bỏ xi đi. Khi đó, số lượng các số thuộc T′ cótrong [xi − 1, xi + 1] là k− 1, ngoài ra xi đã bị bỏ đi thuộc về đúng 2k− 1 cặpcủa AT (gồm k− 1 số ∗ thuộc đoạn trên tạo thành các cặp có dạng (xi, ∗), (∗, xi)

cùng với (xi, xi)). Do đóAT = AT′ + 2k− 1.

Ta viết [xi − 2; xi + 2] = [xi − 2; xi − 1] ∪ [xi − 1; xi + 1] ∪ [xi + 1; xi + 2], trừđoạn ở giữa thì hai đoạn đầu và cuối chứa tối đa k phần tử của T. Hơn nữa,do định nghĩa số xi nên trong đoạn [xi + 1; xi + 2] có tối đa k− 1 phần tử củaT. Suy ra có tối đa 2(k− 1) + k = 3k− 2 phần tử của T (không tính xi) thuộc[xi − 2; xi + 2], suy ra

BT ≤ 2(3k− 2) + 1 + BT′ = 3(2k− 1) + BT′ .

Áp dụng giả thiết quy nạp, ta có BT′ < 3AT′ nên từ các điều trên, suy ra

BT ≤ 3(2k− 1) + BT′ < 3(2k− 1) + 3AT′ = 3(AT′ + 2k− 1) = 3AT.

Theo nguyên lý quy nạp, ta có đpcm.

Bài toán kết thúc.

Nhận xét. Ta thử xét một số ước lượng giữa A, B khi các số x1, x2, . . . , xn đặc biệt.

• Nếu n số bằng nhau thì ai = bi = n nên A = B.• Nếu n số chẵn liên tiếp thì ai = 1, ∀i = 1, 2, . . . , n; còn b1 = bn = 2, bi = 3, ∀i =

2, . . . , n− 1 nên A = n, còn B = 3n− 2 và rõ ràngBA

= 3− 2n→ 3 chứng tỏ 3 là

hằng số tốt nhất.

Bằng phương pháp tương tự trên, ta cũng có thể chứng minh được rằng với 0 <

a < b thì đặt số cặp có thứ tự (i, j) thoả∣∣xi − xj

∣∣ ≤ a và số cặp có thứ tự (i, j) thoả∣∣xi − xj∣∣ ≤ b lần lượt là A, B thì sẽ có A < (2k− 1) B với k = db/ae .

Bài 3. Cho p là số tự nhiên. Xét phương trình x3 + x + p = y2 trên tập số nguyên dương.

a) Tìm số nguyên tố p nhỏ nhất dạng 4k + 1 sao cho phương trình có nghiệm.b) Chứng minh rằng nếu p là số chính phương thì phương trình luôn có nghiệm.

Lời giải.

a) Các số nguyên tố có dạng 4k + 1 là 5, 13, 17, ...

Trước hết, ta thấy với p = 13 thì x3 + x + 13 = y2 có nghiệm là (x; y) = (4; 9).

Ta cần chứng minh x3 + x + 13 = y2 không có nghiệm nguyên. Xét (mod 4).

158

Page 159: I HÅC QUÈC GIA TP. HCM · 2020. 10. 18. · Ngo i b i vi¸t cõa anh Ph¤m Tu§n Huy tø n«m 2013, c¡c b¤n s³ th§y r§t nhi·u b i vi¸t m t¡c gi£ v¨n cán l håc sinh cõa

• Khi x chia 4 dư 0, 1, 2, 3, vế trái chia 4 dư 1, 3, 3, 3.• Khi y chia 4 dư 0, 1, 2, 3, vế phải chia 4 dư 0, 1, 0, 1.

Do đó, y phải lẻ và x phải chia hết cho 4.

Ta viết biểu thức đã cho thành x3 + x+ 30 = y2 + 25 hay (x+ 3)(x2− 3x+ 10) =y2 + 52.

Do x + 3 chia 4 dư 3 nên nó có ước nguyên tố dạng q = 4k + 3. Ta biết rằng vớia, b ∈ Z thì a2 + b2 chia hết cho số nguyên tố q = 4k + 3 khi và chỉ khi q | a, q | b,nên từ trên ta suy được q | 5, vô lý. Vậy nên phương trình trên vô nghiệm vàp = 13 là số nguyên tố nhỏ nhất cần tìm.

b) Trước hết xin phát biểu không chứng minh bổ đề sau:

Bổ đề. Với các số nguyên dương a, b, c, d thoả mãn ab = cd thì tồn tại các số nguyênx, y, z, t sao cho a = xy, b = zt, c = xz, d = yt.

Quay lại bài toán. Do p là số chính phương nên đặt p = a2, a ∈ Z. Ta viết lạiphương trình thành

x3 + x + a2 = y2 ⇔ x(x2 + 1) = (y− a)(y + a).

Áp dụng bổ đề vào bài toán, ta thấy tồn tại các số m, n, p, q nguyên dương đểx = mn; x2 + 1 = pq; y + a = mp; y− a = nq. Suy ra

(mn)2 + 1 = pq và mp− nq = 2a.

Ta biết rằng dãy số sai phân tuyến tính có dạng un+2 = αun+1 + βun, khiđặt vn = u2

n+1 − unun+2 thì vn là cấp số nhân công bội −β (thật vậy, vn =

un+1(αun + βun−1)− un(αun+1 + βun) = −β(u2n − un+1un−1) = −βvn−1).

Ta xét dãy tuyến tính u0 = 0, u1 = 1, un+2 = αun+1 + un thì rõ ràng với mọi nthì

u2n − un+1un−1 = (−1)n−1(u2

1 − u2u0) = (−1)n−1.

Khi đó, với mọi n chẵn thì u2n − un+1un−1 = −1 nên có thể chọn mn = u2k của

dãy. Ta có

u2 = α, u3 = α2 + 1, u4 = α(α2 + 2), u5 = α4 + 3α2 + 1.

Chọn p = u3, q = u5, mn = u4 thì rõ ràng (mn)2 + 1 = pq.

Ta cần có

mu3 − nu5 = 2a⇔ m(α2 + 1)− n(α4 + 3α2 + 1) = 2a.

159

Page 160: I HÅC QUÈC GIA TP. HCM · 2020. 10. 18. · Ngo i b i vi¸t cõa anh Ph¤m Tu§n Huy tø n«m 2013, c¡c b¤n s³ th§y r§t nhi·u b i vi¸t m t¡c gi£ v¨n cán l håc sinh cõa

Chọn tiếp α = 4a2 và viết m = 2a(α2 + 2), n = 2a(α4 + 3α2 + 1) thì đẳng thứctrên đúng vì

(α2 + 1)(α2 + 2)− (α4 + 3α2 + 1) = 1.

Từ đó ta chỉ được nghiệm cụ thể của phương trình là

(x; y) =(

4a2(16a4 + 2); 2a(16a4 + 2)(16a4 + 1)− a)

với a =√

p ∈ Z+.

Bài toán kết thúc.

Nhận xét. Ở câu a, việc ứng dụng bổ đề trên là khá tự nhiên. Bổ đề là hệ quả quenthuộc của định lý Fermat nhỏ. Một kết quả tương tự đối với số nguyên tố có dạng3k + 2 là a2 + ab + b2 chia hết cho p khi và chỉ khi a, b cùng chia hết cho p. Một số bàitoán tương tự:

a) (Euler) Chứng minh rằng phương trình sau không có nghiệm nguyên: y2 =

x3 + 7.b) (USA TST 2008) Chứng minh rằng phương trình x2 = n7 + 7 không có nghiệm

nguyên.c) A = 3p + p− 4 với p là số nguyên tố thì A không phải là số chính phương, cũng

không là tích của hai số tự nhiên liên tiếp.

Ở câu b, việc dùng bổ đề giúp ta chuyển từ bài toán “tồn tại hai số” sang “tồn tại bốnsố”, khả năng chọn được sẽ cao hơn. Một số bài toán tương tự:

a) (Hàn Quốc TST, 2012) Cho số nguyên dương n. Chứng minh rằng tồn tại vôhạn bộ ba các số nguyên dương (x, y, z) sao cho nx2 + y3 = z3 và x, y, z đôi mộtnguyên tố cùng nhau.

b) (Ấn Độ) Tìm tất cả các nghiệm nguyên của phương trình x3 + (x + 4)2 = y2.c) Chứng minh rằng với mọi số nguyên dương m thì luôn tồn tại x, y ∈ Z+ sao cho

x3 + 4x + m2 = y2.

Bài 4. Cho tam giác ABC nhọn nội tiếp trong đường tròn có B, C cố định và A di động trên(O). Gọi D là trung điểm BC. Trên AB lấy điểm M, P và trên AC lấy các điểm N, Q sao choDA = DP = DQ, DM⊥AC, DN⊥AB.

a) Chứng minh rằng các điểm M, N, P, Q cùng thuộc một đường tròn, đặt là (C) vàđường tròn này luôn đi qua một điểm cố định.

b) Chứng minh rằng tâm của (C) luôn di chuyển trên một đường tròn cố định.

Lời giải.

a) Dễ thấy tam giác AMQ cân tại M nên

∠DMQ = ∠DMA = 90◦ −∠A =180◦ − 2∠A

2=

180◦ −∠PDQ2

= ∠DPQ,

160

Page 161: I HÅC QUÈC GIA TP. HCM · 2020. 10. 18. · Ngo i b i vi¸t cõa anh Ph¤m Tu§n Huy tø n«m 2013, c¡c b¤n s³ th§y r§t nhi·u b i vi¸t m t¡c gi£ v¨n cán l håc sinh cõa

suy ra tứ giác MPDQ nội tiếp. Chứng minh tương tự, ta có tứ giác QNDP nộitiếp nên suy ra M, N, P, Q cùng nằm trên một đường tròn đi qua điểm D cốđịnh.

b) Gọi I là tâm MNPQ và Z, T lần lượt là giao điểm của DN, DM với AB, AC. Giảsử OD cắt MN ở J ta sẽ chứng minh rằng J là trung điểm MN. Thật vậy, gọiR là điểm đối xứng của N qua D thì MN ⊥ DR và BRCN là hình bình hành,kéo theo PR ‖ AC mà DM ⊥ AC nên BR ⊥ DM. Suy ra B là trực tâm tam giácDMR. Do đó BD⊥MR⇒ MR ‖ DJ, kéo theo J là trung điểm MN. Lại có

DJ =12

RM =12· (BD · tan∠MDR) = BD · tan A =

12

BC · tan A

nên suy ra J là điểm cố định.

Gọi X, Y lần lượt là trung điểm AD, DJ thì rõ ràng Y cũng là điểm cố định. Tasẽ chứng minh rằng DXI J là hình bình hành.

Dễ thấy J là trung điểm MN cũng là tâm đường tròn ngoại tiếp tứ giác MZTN;còn X là tâm (AZT) nên JX ⊥ ZT mà ∠ATZ = ∠ANM = ∠APQ nên ZT ‖PQ, dẫn đến JX ⊥ PQ.

Lại có D là tâm (APQ) và I là tâm (DPQ) nên DI ⊥ PQ, kéo theo JX ‖ ID.Ngoài ra, vì D là trực tâm tam giác AMN nên AD ⊥ MN, mà I J ⊥ MN suyra I J ‖ AD. Từ đó ta có tứ giác DXJI là hình bình hành, dẫn đến X, I đối xứngnhau qua Y. Mà điểm X chạy trên đường tròn (ω) là ảnh của (O) qua phép vị

tự tâm D tỉ số12

cố định nên I cũng chạy trên đường tròn cố định là ảnh của

(ω) qua phép đối xứng tâm I.

Bài toán kết thúc.

Nhận xét. Bài toán trên còn nhiều cách xử lý khác, nhưng ở đây giới một cách thuầntúy nhất. Chẳng hạn, để có J là trung điểm MN ta có thể dùng định lý con bướm chođường tròn (MNZT). Ta cũng nhận thấy AJ đẳng giác với AD trong góc ∠BAC nênAJ đi qua giao điểm T của hai tiếp tuyến kẻ từ B, C của (O). Khi đó AMTN nội tiếpđường tròn đường kính AT với DMTN là hình bình hành,. . . Nói chung các mô hìnhliên kết nhau rất thú vị.Thực ra ở điểm này, ta có thể thấy bài toán có liên hệ với bài 4 của đề Sharygin khối 9năm 2018: Cho tam giác ABC nội tiếp đường tròn (O) có BC cố định và A di động.Gọi H là trực tâm tam giác và D ∈ AB, E ∈ AC sao cho H là trung điểm DE Chứngminh rằng tâm của (ADE) thuộc một đường tròn cố định khi A di động.Bài toán 4 là phát biểu ngược lại của bài toán Sharygin trên. Cuối cùng, chúng tôixin giới thiệu lời giải ngắn gọn của BTC Sharygin để bạn đọc tham khảo: Giả sử P, Qlà trung điểm AD, AE và trung trực AD, AE cắt BC ở R, S. Gọi O′ là tâm (ADE) thì

∠RO′S = 180◦ −∠A cố định. Vì ∠BHP = 90◦ ⇒ BP = BH · 1cos A

, mà H di chuyển

trên đường tròn (ω) cố định nên P cũng di chuyển trên đường tròn cố định (ω)′,

161

Page 162: I HÅC QUÈC GIA TP. HCM · 2020. 10. 18. · Ngo i b i vi¸t cõa anh Ph¤m Tu§n Huy tø n«m 2013, c¡c b¤n s³ th§y r§t nhi·u b i vi¸t m t¡c gi£ v¨n cán l håc sinh cõa

là ảnh của (ω) qua phép vị tự tâm B tỷ số1

cos A. Mà ∠BPR = 90◦ nên suy ra BR là

đường kính của (ω)′, kéo theo R cố định. Tương tự thì S cũng cố định nên O′ thuộccung chứa góc 180◦ −∠A dựng trên RS.

Bài 5. Cho số thực a 6= 0 và dãy (un) thoả mãn u1 = 0, un+1(un + a) = a + 1 với mọin ∈ Z+. Tìm giới hạn của dãy (un).

Lời giải. Đặt un =xn

ynvới n ≥ 1. Theo giả thiết thì un+1 =

a + 1un + a

⇒ xn+1

yn+1=

(a + 1)yn

xn + ayn.

Xét quan hệ truy hồi của hai dãy là xn+1 = (a + 1)yn và yn+1 = xn + ayn thì

yn+2 = xn+1 + ayn+1 = ayn+1 + (a + 1)yn.

Vì u1 = 0, u2 =a + 1

anên chọn y1 = 1, y2 = a; từ đó, ta có yn =

(a + 1)n − (−1)n

a + 2với

n ≥ 1.Công thức trên chỉ xác định với a 6= −2 nên xét trường hợp a = −2, ta có dãy

u1 = 0,

un+1 =1

2− un, n ≥ 1

.

Bằng quy nạp, ta chứng minh được rằng un ∈ [0; 1) nên un+1 − un =1

2− un− un =

(un − 1)2

2− un> 0 nghĩa là dãy (un) tăng, và dãy bị chặn trên bởi 1 nên có giới hạn, đặt là

L ∈ (0, 1) thì giải ra ta có L =1

2− L, tức là L = 1.

Tiếp theo, xét a 6= −2 ta có un =xn

yn=

(a + 1)yn−1

yn=

(a + 1)n + (a + 1)(−1)n

(a + 1)n − (−1)n với mọi n.

Đặt −(a + 1) = b ∈ {−1; 1}, ta viết lại un =bn − bbn − 1

với mọi n ≥ 1.

Nếu b > 1 hoặc b < −1, tương ứng là a < −2 hoặc a > 0, thì lim un = 1.Nếu −1 < b < 1, tương ứng là −2 < a < 0, thì lim un = b = −(a + 1).Vậy ta có kết luận sau:

• Nếu a ∈ (−2; 0) thì lim un = −(a + 1).• Nếu a /∈ (−2; 0) thì lim un = −1.

Bài toán kết thúc.

Nhận xét. Cách giải trên hiệu quả và triệt để vì khi đã tìm được công thức tổng quátcủa dãy thì mọi thứ sẽ rõ ràng hơn nhiều.

162

Page 163: I HÅC QUÈC GIA TP. HCM · 2020. 10. 18. · Ngo i b i vi¸t cõa anh Ph¤m Tu§n Huy tø n«m 2013, c¡c b¤n s³ th§y r§t nhi·u b i vi¸t m t¡c gi£ v¨n cán l håc sinh cõa

Dạng toán này không mới, tuy nhiên điểm khó ở đây là đề bài xét a 6= 0 chứ khôngphải Và nếu như đã có a > 0 thì un > 0 nên bài toán sẽ trở thành dạng un+1 = f (un)

kinh điển, ở đây vì chưa biết a > 0 hay a < 0 nên nếu ngay từ đầu, ta xét f (x) =a + 1x + a

thì chưa xác định được miền cụ thể của x; và cần phải làm rõ tại sao dãy số un luônxác định, tức là un 6= −a, ∀n. Trong lời giải trên, ta cũng thấy rằng số hạng đầu u1 = 0cũng quan trọng và ảnh hưởng nhiều đến việc biện luận sau đó.Một số bài toán tương tự:

a) Chứng minh rằng để dãy số xác định bởi

{u1 = b,un+1 = u2

n − (2a− 1)un + a2 , n ≥ 1

hội tụ thì điều kiện của a, b là a− 1 ≤ b ≤ a.b) (VMO 2000) Cho dãy số (xn) xác định bởi

{x0 ∈ (0; c),xn+1 =

√c−√c + xn

, n ≥ 0.

Tìm c để dãy số xác định với mọi n và tồn tại giới hạn hữu hạn lim xn.

Bài 6. Tìm tất cả các hàm số f : R→R+ thoả mãn điều kiện

f(

x f (y2)− y f (x2))= (y− x) f (xy) ∀ y > x > 0.

Lời giải. Để dễ theo dõi, ta chia lời giải thành các bước sau:Bước 1. Hàm số f (x) đồng biến trên R+.Theo giả thiết thì với mọi y > x > 0 ta đều có

x f (y2)− y f (x2) > 0⇒ f (y2)

f (x2)>

yx> 1.

Do đó, y2 > x2 ⇔ y > x ⇔ f (y2) > f (x2) nên hàm f đã cho đồng biến trên R+.Bước 2. f (1) = 1.Trong đề bài, thay y = x + 1, ta có

f(

x f ((x + 1)2)− (x + 1) f (x2))= f (x(x + 1))

hayx f ((x + 1)2)− (x + 1) f (x2) = x(x + 1)

⇔ f ((x + 1)2)

x + 1=

f (x2)

x+ 1, ∀x > 0

163

Page 164: I HÅC QUÈC GIA TP. HCM · 2020. 10. 18. · Ngo i b i vi¸t cõa anh Ph¤m Tu§n Huy tø n«m 2013, c¡c b¤n s³ th§y r§t nhi·u b i vi¸t m t¡c gi£ v¨n cán l håc sinh cõa

Suy raf (n2)

n=

f ((n− 1)2)

n− 1+ 1 = · · · = f (1)

1+ (n− 1) = a + n− 1

hayf (n2) = n(a + n− 1) với mọi số nguyên dương n.

Trong đề bài, thay y = m2, x = n2 với m, n ∈ Z+ sao cho m2 − n2 = p2 là số chínhphương (rõ ràng có vô hạn bộ như thế là các bộ ba Pythagoras), ta có

f(

n2 f (m4)−m2 f (n4))= (m2 − n2) f (m2n2)

⇔ f (m2n2p2) = p2mn(a + mn− 1)

⇔ mnp(a + mnp− 1) = p2mn(a + mn− 1)

⇔ (p− 1)(a− 1) = 0

Đẳng thức cuối phải luôn đúng với mọi p được chọn như trên nên a = 1. Suy raf (1) = 1.Bước 3. f (x) = x ∀ x ∈ Q.Tiếp theo, cũng từ đẳng thức

f ((x + 1)2)

x + 1=

f (x2)

x+ 1, ∀x > 0,

ta suy raf ((x + n)2)

x + n=

f (x2)

x+ n, ∀x > 0, n ∈ Z+

hayx f ((x + n)2)− (x + n) f (x2) = nx(x + n).

Trong đề bài, thay y = x + n, ta có

f(

x f ((x + n)2)− (x + n) f (x2))= n f (x(x + n))

⇔ f (nx(x + n)) = n f (x(x + n))

Với mọi n ∈ Z+, y > 0, ta luôn chọn được x > 0 để x(x + n) = y nên ta có

f (ny) = n f (y), ∀ n ∈ Z+, y ∈ R+.

Với mọi n ∈ Z+ cho y =1n

, suy ra f (1) = n f(

1n

)⇒ f

(1n

)=

1n

. Suy ra

f( n

m

)= n f

(1m

)=

nm

, ∀m, n ∈ Z+

164

Page 165: I HÅC QUÈC GIA TP. HCM · 2020. 10. 18. · Ngo i b i vi¸t cõa anh Ph¤m Tu§n Huy tø n«m 2013, c¡c b¤n s³ th§y r§t nhi·u b i vi¸t m t¡c gi£ v¨n cán l håc sinh cõa

hay f (x) = x, ∀x ∈ Q+.Bước 4. f (x) = x ∀ x ∈ R+.Với mọi số thực x0 > 0, giả sử f (x0) > x0 thì chọn hai dãy số hữu tỷ (an, bn) saocho an < x0 < bn và lim an = lim bn = x0. Rõ rang f (an) < f (x0) < f (bn) ⇒ an <

f (x0) < bn, cho n→ ∞, ta có f (x0) = x0. Do đó, với mọi số thực x > 0 thì f (x) = x.Thử lại ta thấy thoả vì

f(

x f (y2)− y f (x2))= f (xy2 − yx2) = xy2 − x2y = (y− x)xy = (y− x) f (xy).

Vậy tất cả các hàm số cần tìm là f (x) = x, ∀ x > 0.

Nhận xét. Bài toán dùng các kỹ thuật quen thuộc nhưng đòi hỏi nhiều bước nên thísinh cần chắc các phương trình hàm dạng này thì mới xử lý triệt để được. Hàm xácđịnh trên R+ và cho điều kiện các biến phân biệt cũng gây không ít khó khăn chỉ nắmcác phép thế trong phương trình hàm.Hai mấu chốt quan trọng để giải quyết bài toán là:

• Hàm số f (x) đơn điệu thì f (x) = f (y)⇒ x = y.• Hàm số đơn điệu và cộng tính trên Q thì f (x) = ax ∀ x ∈ R.

Dưới đây là một số bài tương tự:

a) Tìm tất cả các hàm số f : R+ → R+ sao cho

f (x + f (y)) = f (x)− x + f (x + y) ∀ x, y > 0.

b) Tìm tất cả các hàm số f : R+ → R+ thoả

f ( f (x) + 2y) = f (2x + y) + 2y ∀ x, y > 0.

c) Cho hai hàm số f , g : R+ → R+ thoả mãn và với mọi x, y > 0 thì

f (g(x) + y) = f (x) + g(y)g ( f (x) + y) = g(x) + f (y)

.

Chứng minh rằng f (x) = g(x), ∀x > 0.

Bài 7. Cho n = 2018.2019. Gọi A là tập hợp tất cả các bộ số (a1; a2; . . . ; an−1; an) có tính

thứ tự sao cho ai ∈ {0; 1} ∀i = 1, 2, ..., n vàn

∑i=1

ai = 20182. Có bao nhiêu bộ số như thế để

thoả mãnk

∑i=1

ai ≥k2

vàk

∑i=1

an−k+1 ≥k2

với k = 1, 2, ..., n?

Lời giải. Ta giải bài toán tổng quát khi thay 2018 bởi m ∈ Z+. Bài toán đã cho tươngđương với bài toán sau:

165

Page 166: I HÅC QUÈC GIA TP. HCM · 2020. 10. 18. · Ngo i b i vi¸t cõa anh Ph¤m Tu§n Huy tø n«m 2013, c¡c b¤n s³ th§y r§t nhi·u b i vi¸t m t¡c gi£ v¨n cán l håc sinh cõa

Trong hệ trục tọa độ Oxy xét lưới điểm nguyên trong hình chữ nhật có đỉnhdưới bên trái là O(0, 0) và đỉnh trên bên phải là A(m2; m). Đặt B(m; m) vàC(m2 −m; 0), hỏi có bao nhiêu đường đi từ O→ A sao cho mỗi bước, ta đisang phải hoặc lên trên 1 đơn vị, gọi là đường đi đơn, và không vượt lên trênOB cũng như không xuống dưới AC?

Ở đây, các số 0; 1 tương ứng với các bước đi lên trên, các bước đi sang phải; còn điều

kiện tổng k số đầu và tổng k số cuối không nhỏ hơnk2

tương ứng với số lượng bước

đi lên không vượt quá số lượng bước đi sang phải.Để thuận tiện, ta gọi đường đi cắt d nếu nó có các phần nằm về cả hai phía của d.Trước hết, ta sẽ chứng minh bổ đề sau:

Bổ đề. Số đường đi đơn từ O→ A(m; n), có cắt đường thẳng y = x, là Cm+1m+n.

ml

Trong hình trên, đường cũ là đứt nét, còn đường mới là liền nét.

Thật vậy, xét đường thẳng (d) : y = x + 1, rõ ràng các đường đi đơn cắt y = x đềusẽ có điểm chung với đường thẳng (d) này. Tại các điểm chung đó, ta thực hiện đốixứng trục để được một đường đi mới xuất phát từ O→ A′(n− 1, m + 1).Rõ ràng phép đối xứng trục trên là song ánh, biến các đường cần tìm (cắt y = x),thành các đường từ O→ A′; do đó, số lượng đường cần tìm là Cn−1

m+n.Trở lại bài toán. Số đường đi đơn từ O→ A(m2; m) là Cm

m2+m vì nó bằng số cách chọnm lần đi lên trong tổng số m2 + m lần di chuyển, trong đó số đường đi cắt OB bằngsố đường đi cắt AC và bằng Cm−1

m2+m (theo bổ đề).Do đó, ta chỉ cần tìm số đường đi cắt cả OB, AC với ý tưởng đối xứng hai lần đã dùngđể chứng minh bổ đề.Đầu tiên, ta thực hiện đối xứng qua đường thẳng y = x + 1; khi đó, các đường đi đơnsẽ xuất phát từ O→ A′(m− 1; m2 + 1). Do các đường ban đầu còn vượt qua AC nêncác đường mới phải cắt thêm y = x + m2−m + 3. Tiếp tục đối xứng qua đường thẳngnày, ta đưa về đếm số đường đi đơn từ O→ A′′(m− 2, m2 + 2). Suy ra số đường đitrong trường hợp này là Cm−2

m2+m.

166

Page 167: I HÅC QUÈC GIA TP. HCM · 2020. 10. 18. · Ngo i b i vi¸t cõa anh Ph¤m Tu§n Huy tø n«m 2013, c¡c b¤n s³ th§y r§t nhi·u b i vi¸t m t¡c gi£ v¨n cán l håc sinh cõa

Vậy theo nguyên lý bù trừ, kết quả cần tìm sẽ là Cm−2m2+m. Thay m = 2018, ta có số lượng

đường đi, cũng chính là số bộ thoả mãn đề bài.

Nhận xét. Bài toán thoạt nhìn có vẻ có thể xử lý được bằng truy hồi hoặc bù trừ trựctiếp, nhưng quả thật không dễ. Việc tiếp cận theo hướng dùng “lưới nguyên” đòihỏi ít nhiều kinh nghiệm và các kỹ thuật liên quan, vì bài toán này sau khi mô hìnhhóa xong còn phải thêm bước “đối xứng hai lần” mới có thể giải quyết triệt để được.Đường đi trong bài toán còn gọi là đường đi Dyck hoặc Catalan, liên quan đến các bàitoán nổi tiếng như: bỏ phiếu bầu cho các ứng viên sao cho ứng viên này luôn thắngứng viên kia tại mọi thời điểm, mua vé với tiền 1 đồng, 2 đồng sao cho không có aicần phải chờ tiền thối lại,. . .Bài toán cũng có thể mô phỏng theo dãy các đường đi chéo, lên hoặc xuống 1 đơn vị,từ (0, 0) đến (n2 + n; n2− n) sao cho đường đi không xuống dưới trục hoành và cũngkhông vượt lên trên y = n2. Thực ra nếu trình bày theo hướng này thì việc lấy đốixứng sáng sủa hơn.Ứng với n = 1, 2, 3, 4 ta có các giá trị 0, 4, 100, 2755 là một dãy số không quen thuộcnên chúng tôi cũng không rút gọn đáp số trên.Dưới đây là một số kết quả tương tự về đường đi đơn trong đề bài:

a) Số đường đi đơn từ (0; 0) → (m; n) mà không có điểm chung với y = x làm− nm + n

Cmm+n.

b) Số đường đi đơn từ (0; 0)→ (m; n) mà không vượt qua y = x là Cnm+n − Cn−1

m+n.c) Số đường đi gồm n bước mà không vượt qua y = x là

n∑

i=n/2

n!(2i + 1− n)(i + 1)!(n− i)!

= C[n/2]n .

d) Số đường đi đơn từ (0; 0) → (m; n) mà không có điểm chung với y = x + t làCn

m+n − Cm−tm+n.

e) (Việt Nam TST 2003) Tính số cặp đường đi đơn (0; 0)→ A(m, n) và B(p; 0)→C(m; q) với p < m; q < n sao cho chúng không có điểm chung.

Bài 8. Cho đường tròn (C) có tâm I nội tiếp tam giác ABC và tiếp xúc với các cạnh AB, ACtại E, F. Gọi AM, AN là phân giác trong và ngoài của tam giác với M, N ∈ BC. Gọi dM, dN

lần lượt là tiếp tuyến khác BC của (C) và đi qua M, N.

a) Chứng minh rằng dM, dN và EF đồng quy. Gọi điểm đồng quy đó là D.b) Trên AB, AC lần lượt lấy P, Q thoả mãn DP ‖ AC, DQ ‖ AB. Gọi R, S lần lượt là

trung điểm DE, DF. Chứng minh rằng I thuộc đường thẳng qua trực tâm của các tamgiác DPS, DQR.

Lời giải.

a) Gọi X, Y lần lượt là tiếp điểm của tiếp tuyến thứ hai kẻ từ M đến I và D′ là tiếpđiểm của (I) trên BC. Gọi K là trung điểm EF.

167

Page 168: I HÅC QUÈC GIA TP. HCM · 2020. 10. 18. · Ngo i b i vi¸t cõa anh Ph¤m Tu§n Huy tø n«m 2013, c¡c b¤n s³ th§y r§t nhi·u b i vi¸t m t¡c gi£ v¨n cán l håc sinh cõa

Xét trong đường tròn (I) thì EF là đường đối cực của A và K ∈ EF nên đối cựccủa K sẽ đi qua A, mà NA ⊥ IA nên NA chính là đường đối cực của K.

Đường đối cực của K đi qua N nên đối cực của N là D′Y sẽ đi qua K. Dễ thấy rằngAM là trục đối xứng của tứ giác D′EF nên suy ra D′X ⊥ EF. Xét D′(EF, XY),ta có D′Y đi qua trung điểm của EF và D′X ⊥ EF nên D′(EF, XY) = −1 hay tứgiác EXFY điều hòa. Suy ra MX, NY, EF đồng quy. Ngoài ra ta cũng có X, Y, Athẳng hàng.

b) Dễ thấy các tam giác PED và DQF là các tam giác cân. Gọi H1, H2 lần lượt làtrực tâm của tam giác DPS, DQR. Ta có ∠PH1S = ∠PDF = ∠AFE = ∠PESnên EPSH1 là tứ giác nội tiếp. Suy ra

RH1 · RP = RS · RE.

Ngoài ra, KA · KI = KE · KF nên

RPKA· RH1

KI=

REKE· RS

KF.

Theo định lý Thales thìRPKA

=REKE

nênRH1

KI=

RSKF

, mà

RS = RD− SD =DE− DF

2=

EF2

= KF

nên RH1 = KI, mà RH1 ‖ KI (do cùng vuông góc với EF) nên IKRH1 là hìnhchữ nhật, kéo theo IH1 ‖ EF. Một cách tương tự, ta có IH2 ‖ EF nên H1H2 điqua I.

Bài toán kết thúc.

Nhận xét. Câu a của bài toán đơn giản nhưng nhiều kỹ thuật, câu b thì phức tạpnhưng lại giải được nhẹ nhàng. Bài toán có nhiều ý mới, đẹp và thú vị. Mô hình trêncòn có nhiều ý để khai thác:

• MX là trục đẳng phương của (I) và đường tròn bàng tiếp góc A là (Ia).• Đường trung bình của tam giác DMN đi qua K.• Điểm I thuộc cả hai đường tròn (PES), (QFR).

Chú ý rằng câu a của bài toán vẫn đúng khi thay M, N bởi cặp điểm liên hợp điềuhòa tuỳ ý với B, C ; không nhất thiết phải là chân các đường phân giác.Ngoài cách dùng đối cực, ta có thể dùng bổ đề: Tam giác ABC có (I) nội tiếp tiếp xúcvới AB, AC ở D, E thì BI, CI cắt DE tại các điểm thuộc đường tròn đường kính BC.Khi đó, bài toán sẽ được giải quyết nhẹ nhàng hơn nhiều.

168

Page 169: I HÅC QUÈC GIA TP. HCM · 2020. 10. 18. · Ngo i b i vi¸t cõa anh Ph¤m Tu§n Huy tø n«m 2013, c¡c b¤n s³ th§y r§t nhi·u b i vi¸t m t¡c gi£ v¨n cán l håc sinh cõa

GIỚI THIỆU MỘT SỐ BÀI TOÁN HAY

Ban biên tập

GIỚI THIỆU. Trong phần này, chúng tôi giới thiệu một số bài toán sơ cấp hay vàthú vị để bạn đọc tự luyện tập. Các bài toán được giới thiệu trong chuyên đề nàyđến từ đủ 4 phân môn chính là Đại số, Hình học, Số học và Tổ hợp. Một số bài toánđược tham khảo từ những tài liệu khác đều được trích dẫn nguồn gốc rõ ràng.

1. ĐẠI SỐ

Bài toán 1 (PTNK 2016). Tìm số nguyên dương k nhỏ nhất để bất đẳng thức sau đúng:

xkykzk(x3 + y3 + z3) ≤ 3 với mọi x, y, z > 0 và x + y + z = 3.

Bài toán 2 (IMO Shortlist 2016). Tìm tất cả các hàm số f : R+ → R+ thoả mãn:

x f (x2) f ( f (y)) + f (y f (x)) = f (xy)(

f ( f (x2)) + f ( f (y2)))∀x, y ∈ R+

Bài toán 3 (ELMO Shortlist 2017). Tìm tất cả các hàm số f : R→ R thoả mãn đồng thờicác điều kiện sau:

• Nếu a + b + c ≥ 0 thì f (a3) + f (b3) + f (c3) ≥ f (3abc).• Nếu a + b + c ≤ 0 thì f (a3) + f (b3) + f (c3) ≤ f (3abc).

Bài toán 4 (Hàn Quốc 2017). Với mỗi số nguyên dương n, đặt cn = 2017n. Cho một hàmsố f : N→ R thoả mãn đồng thời các điều kiện sau:

• Với mọi số nguyên dương m, n thì f (m + n) ≤ 2017 f (m) f (n + 325).• Với mọi số nguyên dương n thì 0 < f (cn+1) < f (cn)2017.

Chứng minh rằng tồn tại dãy số a1, a2, ... thoả mãn: ∀n, k mà ak < n thì f (n)ck < f (ck)n.

Bài toán 5 (Russia 2014). Một tập hợp hữu hạn điểm nằm trên mặt phẳng toạ độ được gọilà thích hợp nếu chúng có hoành độ đôi một khác nhau và mỗi điểm được tô bởi một tronghai màu: xanh hoặc đỏ. Ta nói đồ thị của một đa thức phân tách tập điểm đó nếu ở phần mặtphẳng phía trên của đồ thị chỉ có các điểm thuộc cùng một màu và ở phần mặt phẳng phíadưới của đồ thị chỉ có các điểm thuộc màu còn lại. Ngay trên đồ thị có thể có các điểm thuộc cả

169

Page 170: I HÅC QUÈC GIA TP. HCM · 2020. 10. 18. · Ngo i b i vi¸t cõa anh Ph¤m Tu§n Huy tø n«m 2013, c¡c b¤n s³ th§y r§t nhi·u b i vi¸t m t¡c gi£ v¨n cán l håc sinh cõa

hai màu. Với mỗi số tự nhiên n > 1, hãy xác định số k nhỏ nhất với tính chất: với mọi bộ nđiểm thích hợp, luôn tồn tại đa thức bậc không quá k mà đồ thị của nó phân tách tập điểm này.

2. HÌNH HỌC

Bài toán 6. Cho tam giác ABC nội tiếp (O). Tiếp tuyến tại B, C của (O) cắt nhau tại X.Các điểm Y, Z xác định tương tự. Gọi A1, B1, C1 lần lượt nằm trên OX, OY, OZ sao cho:

A1OA1X

=B1OB1Y

=C1OC1Z

= k

a) Chứng minh rằng AA1, BB1, CC1 đồng quy tại N.

b) Gọi A2 là điểm nằm trên OA sao choA2OA2A

= k. Đường thẳng da qua A2 vuông góc với

OA cắt BC tại A3. Các điểm B3, C3 xác định tương tự. Chứng minh rằng A3, B3, C3

cùng nằm trên một đường thẳng vuông góc với ON.

Bài toán 7 (Nguyễn Tiến Hoàng). Cho tam giác ABC. D là điểm bất kỳ trên đường thẳngBC. X là điểm đối xứng của A qua D. Y, Z lần lượt là điểm đối xứng của D qua B, C. Gọi Tlà giao điểm thứ hai của (XBY) và (XCZ).

a) Chứng minh rằng XT luôn đi qua điểm P cố định khi D thay đổi.b) Chứng minh rằng XT là trục đẳng phương của các đường tròn (PBY) và (PCZ).

Bài toán 8 (Nguyễn Tăng Vũ). Cho tam giác ABC nội tiếp (O) có M, N là trung điểmCA, AB. BM, CN cắt lại (O) tại X, Y. Gọi (K) là đường tròn đi qua X và tiếp xúc với CAtại C. (L) là đường tròn đi qua Y và tiếp xúc với AB tại B. Gọi P là tâm đường tròn Eulercủa tam giác ABC. Chứng minh rằng (PCK) và (PBL) cắt nhau trên đường tròn (O).

Bài toán 9. Cho tam giác ABC có (ω) là đường tròn Euler. Chứng minh rằng tồn tại đúng3 điểm nằm trên đường tròn ngoại tiếp tam giác ABC sao cho đường thẳng Simson của điểmđó với tam giác ABC thì tiếp xúc với (ω).

Bài toán 10 (Trần Quang Hùng). Cho tam giác ABC có K, L là tâm bàng tiếp góc B, C.Lấy P, Q trên CK, BL sao cho LP ‖ AC, KQ ‖ AB. PQ cắt KL tại R. H là hình chiếu của Rtrên đường thẳng OI của tam giác ABC. AD là phân giác của tam giác ABC. G nằm trênAD sao cho KG ⊥ GA. HB, HC cắt (HGD) tại M, N khác H. DN, DM cắt CA, AB tạiE, F. Chứng minh rằng A, E, H, F thuộc cùng một đường tròn.

3. SỐ HỌC

Bài toán 11. Cho đa thức P(x) = a0 + a1x + ... + anxn (ai ∈ Z, i ∈ {0; 1; ...; n}). Giả sửtồn tại số nguyên dương m thoả mãn đồng thời các điều kiện sau:

• a2, a3, ..., an chia hết cho tất cả các ước nguyên tố của m.• a1 và m là hai số nguyên tố cùng nhau.

170

Page 171: I HÅC QUÈC GIA TP. HCM · 2020. 10. 18. · Ngo i b i vi¸t cõa anh Ph¤m Tu§n Huy tø n«m 2013, c¡c b¤n s³ th§y r§t nhi·u b i vi¸t m t¡c gi£ v¨n cán l håc sinh cõa

Chứng minh rằng ∀k ∈ N∗, tồn tại số nguyên dương c để P(c) là bội của mk.

Bài toán 12. Cho (an) là dãy các số nguyên dương sao cho k số hạng đầu tiên a1, ..., ak làcác số nguyên dương phân biệt. Với mỗi n > k, ta xác định an là số nguyên dương nhỏ nhấtkhông thể biểu diễn được như tổng của một vài số (có thể một) số trong a1, ..., an−1. Chứngminh rằng với mỗi n đủ lớn ta có an = 2an−1.

Bài toán 13 (IMO 2016). Cho P = A1A2...Ak là một đa giác lồi trong mặt phẳng tọa độ.Tất cả các đỉnh A1, A2, ..., Ak đều có tọa độ nguyên và cùng nằm trên một đường tròn. Kíhiệu S là diện tích của P. Cho n là một số nguyên dương lẻ mà bình phương độ dài mỗi cạnhcủa P là một số nguyên chia hết cho n. Chứng minh rằng 2S là một số nguyên chia hết cho n.

Bài toán 14 (Serbia TST 2017). Hàm số f : N→ N được gọi là tốt nếu như:

f a(b) = f (a + b− 1) ∀a, b ∈ N; a > 0

Với ký hiệu f a(b) là tác động hàm f đúng a lần. Gọi g là một hàm số tốt thoả mãn bài toán,và đồng thời giả sử rằng tồn tại số nguyên A sao cho g(A + 2018) = g(A) + 1.

a) Chứng minh rằng g(n + 20172017) = g(n) ∀n ≥ A + 2.b) Nếu như g(A + 1) 6= g(A + 1 + 20172017), hãy xác định g(n) với n < A.

Bài toán 15 (Serbia TST 2017). Với k là số nguyên dương, gọi n là số nguyên dương nhỏnhất có đúng k ước số. Giả sử rằng n là một lập phương đúng. Liệu n có ước nguyên tố dạng3j + 2 hay không ? Hãy chứng minh.

4. TỔ HỢP

Bài toán 16. Cho 2017 số thực có tổng bằng 0. Hỏi có ít nhất bao nhiêu cặp số từ 2017 sốcho trước mà có tổng là một số không âm ? Hãy chứng minh.

Bài toán 17. (China TST 2017) Có 2017 kỹ sư tham gia một hội nghị. Với 2 kỹ sư bất kỳ, khihọ trò chuyện với nhau, họ chỉ sử dụng tiếng Trung hoặc tiếng Anh. 2 kỹ sư bất kỳ thì tròchuyện với nhau không quá 1 lần. Biết rằng với 4 kỹ sư bất kỳ thì ta luôn có:

• Số cuộc trò chuyện giữa họ luôn là số chẵn.• Có ít nhất 1 cuộc trò chuyện được diễn ra bằng tiếng Trung.• Hoặc là không có cuộc trò chuyện nào bằng tiếng Anh, hoặc là số cuộc trò chuyện bằng

tiếng Anh không ít hơn số cuộc trò chuyện bằng tiếng Trung.

Chứng minh rằng tồn tại 673 kỹ sư mà 2 người bất kỳ trò chuyện với nhau bằng tiếng Trung.

Bài toán 18. Một nhà ảo thuật cùng với trợ lý của mình biểu diễn một màn ảo thuật nhưsau. Họ dùng một bộ bài tây gồm 52 lá bài. Các lá bài đều được lật ngửa. Người trợ lý bịt mắtnhà ảo thuật gia sau đó mời một khán giả chọn 13 lá bài bất kì và công bố cho tất cả mọi ngườibiết (trừ nhà ảo thuật). Người trợ lý sau đó sẽ chọn ra 13 lá trong số lá bài còn lại và đưa cho

171

Page 172: I HÅC QUÈC GIA TP. HCM · 2020. 10. 18. · Ngo i b i vi¸t cõa anh Ph¤m Tu§n Huy tø n«m 2013, c¡c b¤n s³ th§y r§t nhi·u b i vi¸t m t¡c gi£ v¨n cán l håc sinh cõa

người khán giả. Người khán giả sẽ trộn tất cả 26 lá bài này theo cách tùy ý rồi đưa cho nhà ảothuật. Thật ngạc nhiên là nhà ảo thuật nói đúng tất cả 13 lá bài mà người khán giả đã chọn.Hỏi nhà ảo thuật và người trợ lý đã làm thế nào ?

Bài toán 19. Có 2n tấm thẻ, mỗi tấm ghi một số 0. Hai bạn An và Bình cùng chơi một tròchơi như sau. Ở mỗi lượt An chia 2n tấm thẻ ra hai phần, mỗi phần gồm n tấm thẻ. Bình đượcphép chọn ra một trong hai phần, công thêm 1 vào các số được trên các tấm được chọn và trừđi một vào các số ở trên các tấm thẻ còn lại. Chứng minh rằng Bình có chiến thuật sao cho tại

mọi thời điểm thì số viết trên mỗi tấm thẻ đều không vượt quá(

2n− 2n− 1

).

Bài toán 20. Cho bảng ô vuông kích thước n× n. Ban đầu tất cả các ô vuông đơn vị được tômàu trắng. Ta tô màu đen một số ô vuông đơn vị sao cho với một ô màu trắng bất kỳ thì cóchung cạnh với ít nhất một ô màu đen. Xác định số lượng ô màu đen ít nhất trên bảng.

172